You are on page 1of 236

Pharmacology Legend Major Topic Adrenergic Anesthetics Anti-Anxiety Agents Anti-Epileptics Anti-Depressants Anti-Histamines Anti-Infectives Anti-Psychotics Aspirin/Acetaminophen/ NSAIDs

Abbreviation Adrenergic Anesthetics Anti-Epileptics Anti-Depressants Anti-Histamines Anti-Infectives Anti-Psychotics Asp/Acet/NSAIDs Major Topic Cholinergic Corticosteroids Cancer/ Chemotherapy Diuretics Hypoglycemics Miscellaneous Terms/Definitions Abbreviation Cholinergic Corticosteroids Cardiovascular Cancer/Chemo Diuretics Hypoglycemics Misc. Terms/Def

Anti-Anxiety Agents Cardiovascular

Narcotic Analgesics Narcotic Analgesics

0
f

PHARMACOLOGY

Adrenerg

Amphetamines are sympathomimetic amines that cause rapid release of:

Dopamine in the brain Serotonin in the brain Norepinephrine in the brain Acetylcholine in the brain

Copyright 2011-2012 - Denial Decks

TM

PDF Editor

Norepinephrine in the brain Amphetamines pass readily into the CNS and cause a rapid release of norepinephrine in the brain. Amphetamines increase systolic and diastolic blood pressures and act as weak bronchodilators and respiratory stimulants. They have a high potential for abuse, resulting in tolerance, psychological dependence, and severe social disability. Abuse causes extreme violence and paranoid psychotic behavior. Note: Amphetamines and ephedrine are indirectly acting sympathomimetic (sympathetic-type) drugs. These drugs demonstrate tolerance and are orally active, unlike epinephrine and norepinephrine. Therapeutic uses of amphetamines: Attention Deficit Hyperactivity Disorder (ADHD; hyperkinesis) - dextroamphetamine (Dexedrine) and a mix of dextroamphetamine with amphetamine (Adderall); (Adderall XR) sometimes used instead of methylphenidate (Ritalin). Narcolepsy - dextroamphetamine (Dexedrine) is used to prevent daytime sleepiness in these patients. Weight loss - phentermine (Ionamin) - structurally similar to dextroamphetamine.

7: norepinephrine (NE) and epinephrine. The enzyme is inhibited by metyrosine. 2. Termination of transmission by NE takes place primarily by the reuptake of NE into prejunctional nerves and secondarily into other cells. Monoamine oxidase (MAO) and catechol-O-methyl transferase (COMT) then play a role in metabolizing the NE.

1. Tyrosine hydroxylase catalyzes the rate limiting step in the synthesis of

TM

PDF Editor

^PHARMACOLOGY

Adrenergic)

The following effects result from what type of pharmacological agent: * Lower blood pressure * Vasodilation * Orthostatic hypotension

1 An alpha-adrenergic receptor blocker (alpha-blocker)

A beta-adrenergic receptor blocker (beta-blocker)

2 Copyright 6 2011-2012 - Dental Decks

^PHARMACOLOGY

Adrenergic J

A drug that reduces the actions of the sympathetic nervous system is called a:

1 Sympathomimetic 1 Sympatholytic

Sympathetic amine Adrenergic agent

3
Copyright C 2011-2012 - Dental Decks

TM

PDF Editor

An alpha-adrenergic receptor blocker (alpha-Mocker) Examples of selective alpha}-blockers used in medicine: Doxazosin (Cardura) - used to treat hypertension Prazosin (Minipress) - used to treat hypertension Terazosin (Hytrin) - manage mild/moderate hypertension; treatment of benign prostatic hyperplasia (BPH) Tamsulnsin (Flomax) - used to treat benign prostatic hyperplasia (BPH) *** These drugs selectively block a ]-type receptors but not a 2 -type. Important: Alpha) -blockers cause orthostatic hypotension, also known as postural hypotension. This is a fainting spell which occurs because of a rapid fall in blood pressure when moving from the supine to the upright position, as in getting rapidly out of the dental chair. The symptoms are similar to simple fainting, however the condition is related to positioning. Note: Other adverse effects include tachycardia, nasal congestion and dry mouth. *** Orthostatic hypotention can also result from centrally acting drugs (i.e., clonidine and methyldopa) and direct vasodilators (i.e., hydralazine and diazoxide) 1. Alpha-blockers inhibit the vasoconstrictor response to epinephrine and levonordefrin. 2. Beta-blockers increase the vasoconstrictor response to epinephrine, but reduce the tachycardia resulting from epinephrine. 3. MAO inhibitors (i.e., phenelzine and tranylcypromine) should not be used with indirectly acting sympathetic drugs (i.e., Tyramine and amphetamines) and with several other drugs such as opioids, especially meperidine. 4. Epinephrine and levonordefrin have exaggerated effects when given with neuron depleting agents like reserpine and quanethidine.

Sympatholytic *** Also called adrenergic blocking agent or adrenergic receptor blockers Four kinds of adrenergic receptor blockers: (Note: They are all used to treat hypertension) I. Beta-adrenergic blockers: Nonselective: blocks both betap and beta2-receptors Propranolol Carteolol Sotalol Penbutolol Timolol Nadolol Betapselective: Esmolol Betaxolol Atenolol Metoprolol Bisoprolol Acebutolol 2. Alpha-adrenergic blockers Nonselective: blocks both alpha]- and alpha2- receptors Phentolamine Phenoxybenzamine Alpha] -selective: Prazosin Doxazosin Terazosin Tamsulosin 3. Centrally acting alpha2-agonists: Act through stimulation of central inhibitory alpha2-adrenergic receptors.They inhibit sympathetic cardioaccelerator and vasoconstrictor centers. Stimulation of alpha-adrenergic receptors in the brainstem results in reduced sympathetic outflow from the CNS. Clonidine Methyldopa 4. Neuronal depleting agents: encompasses a wide variety of drugs having different mechanisms of action. Whatever their specific mechanism, the result is usually the same a depletion of mediator in the neuron terminal or an inability of the mediator to be released from the terminal. Reserpine - depletes granules containing NE in nerve endings, releases NE Guanethidine - blocks adrenergic nerve endings by a series of actions Metyrosine - inhibits tyrosine hydroxylase; used to treat pheochromocytoma Note: Carvedilol and labetalol are nonselective beta-blockers that also block alpha,-receptors. They are used for heart failure.

TM

PDF Editor

^PHARMACOLOGY

Adrenergic)

What is the pharmacologic category of the following drugs?

Epinephrine (Adrenalin) Phenylephrine (Neo-Synephrine) Albuterol (Proventii; Ventolin) Isoproterenol

4 Copyright 2011-2012 - Dental Decks

^PHARMACOLOGY

Adrenergic^

Adrenaline (epinephrine) stimulates:

Alpha,-receptors only Beta preceptors only Both alpha] 2 and beta] 2-receptors Both alpha] and beta,-receptors only

5 Copyright 6 2011 -2012 - Dental Decks

TM

PDF Editor

Adrenergic agonists Note: The adrenergic receptors are divided into the subtypes of: alpha, (a,) and alpha2 (a 2 ) beta, (P,)andbeta 2 (P 2 ). Some common adrenergic agonists, their main uses and receptor preference: Epinephrine: reverse anaphylaxis, vasoconstriction, bronchodilation - alpha,, alpha2, beta,, and beta2 Norepinephrine: vasoconstriction -alpha,, alpha2, and beta, Isoproterenol: bronchodilation -beta,, andbeta 2 Phenylephrine: nasal vasoconstriction - alpha,, and alpha2 Albuterol: bronchodilation - beta2 Terbutaline: bronchodilation - beta2 Clonidine: antihypertensive - alpha2 -Methyldopa: antihypertensive - alpha2 Dobutamine: cardiac stimulation - alpha,, and beta.

Both alpha,

and beta, 2 -receptors


Characteristics Neurotransmitters that Combine With Receptors

Adrenergic Receptor Type Alpha, Alpha, Beta, Beta-

Most common alpha receptor Norepinephrine or Epinephrine Less common alpha receptor Less common beta receptor (found on cells in heart) Most common beta receptor Norepinephrine or Epinephrine Norepinephrine or Epinephrine Epinephrine (Norepinephrine generally combines either weakly or not at all)

Remember: Alpha receptor responses are predominantly excitatory in nature, while beta receptor responses are excitatory in nature in the heart and inhibitory elsewhere.

Comparison of Adrenergic Receptors Receptor Alpha, Site Smooth muscle in blood vessels Stomach, intestine Kidney Liver Smooth muscle in blood vessels Cardiac Kidney Effect of Stimulation Vasoconstriction Decreased motility and tone Increased renin secretion Gluconeogenesis Vasodilation Increased rate and force of contraction Increased renin secretion

Alpha; Beta, Beta;

Bronchial, vascular, coronary Vasodilation arteriole, uterine smooth muscle, skeletal muscle Pancreas Decreased secretion Liver Gluconeogenesis

TM

PDF Editor

^PHARMACOLOGY

Adrenergic)

All of the following drugs are indirect-acting adrenergic agonists, EXCEPT one. Which one is the EXCEPTION?

Tyramine Amphetamine Epinephrine Methamphetamine Hydroxyamphetamine

6 Copyright 2011-2012 - Dental Decks

^PHARMACOLOGY

Adrenergic^

All of the following drugs are alpha-adrenergic receptor blockers EXCEPT one. Which one is the EXCEPTION?

1 Doxazosin (Cardura)

Phenoxybenzamine hydrochloride (Dibenzyline) Phentolamine hydrochloride (Regitine) Prazosin (Minipress) Propranolol (Inderal) Terazosin (Hytrin)

7 Copyright 2011 -2012 - Dental Decks

TM

PDF Editor

Epinephrine Two types of adrenergic agonists: 1. Indirect-acting: are those that cause the release of stored norepinephrine at the postganglionic nerve endings to produce their effects. Tyramine and amphetamines cause release of norepinephrine. 2. Direct-acting: are those that interact directly with the alpha or beta receptors. Direct-acting adrenergic agonists may be receptor selective or receptor non-selective: Phenylephrine = alpha, selective agonist Clonidine = alpha2 selective agonist Dobutamine = alpha, and beta, selective agonist Terbutaline = beta2 selective agonist Albuterol = beta2 selective agonist Epinephrine = alpha, 2 and beta, 2 agonist Norepinephrine = alpha, 2 and beta, agonist Isoproterenol = beta, 2 agonist Remember: Some literature refers to direct and indirect-acting adrenergic agonists as direct and indirect-acting sympathomimetic agents. These terms are almost always synonymous. They are agents that bring about tissue responses resembling those produced by stimulation of the sympathetic nervous system.

Propranolol (Inderal) *** Propranolol is a beta-blocker (specifically a non-selective beta-blocker) Alpha-blockers are medications that act by competitively inhibiting the action of catecholamines at the alpha receptor site. They act on blood vessels, causing them to relax. Alpha-blockers are commonly used to reduce high blood pressure and to treat an enlarged prostate. There are two types of alpha-blockers, which are classified according to the alpha receptor that they block (alpha, or alpha2). Selective alpha antagonists: only block alpha,-receptors and are more commonly used to treat cardiac conditions (hypertension) and benign prostatic hyperplasia. Non-selective alpha antagonists: block both types of receptors and are generally not used for cardiac conditions because blocking both receptors can cause tachycardia (rapid heart beat) and palpitations (pounding heart beat). They are used in the presurgical management of pheochromocytoma and sometimes in treating Raynaud's phenomenon. Some examples of alpha-blockers: Doxazosin - selective alpha,-blocker. Among the alpha,-blockers it is the preferred agent for hypertension due to a longer duration of action. Terazosin - selective alpha,-blocker. Used to treat benign prostatic hypertrophy. Prazosin - selective alpha,-blocker. It is rarely used to treat hypertension due to unwanted adverse reactions. Phenoxybenzamine and phen tola mine - both are nonselective alpha, and alpha2blockers. They are used in the presurgical management of pheochromocytoma. *** Aside from hypotension, which is a major effect of alpha-blockade, alpha-blockers cause relatively few adverse reactions.

TM

PDF Editor

PHARMACOLOGY

Adrenerg

The heart is generally considered to have predominantly which type of adrenergic receptors?

Alpha, (a,) receptors Beta, (6,) receptors


1 Alpha2 (a 2 )

receptors

1 Beta2 (I32) receptors

8 Copyright 2011 -2012 - Dental Decks

0
:

PHARMACOLOGY

Adrenerg
= >

Alpha adrenergic receptors are located on:

Vascular smooth muscle Presynaptic nerve terminals Blood platelets Fat cells Neurons in the CNS All of the above

9 Copyright 2011-2012 - Dental Decks

TM

PDF Editor

Beta, (B,) receptors

Receptor Types in the Sympathetic (Adrenergic) System Receptor Type Alpha, Alpha, Beta, Beta; Prominent Effector Organs Arterioles in skin, mucosa, viscera, and kidney (resistance vessels) Veins Presynaptic nerve endings Postsynaptic in CNS Heart Arterioles (and arteries in skeletal muscle) Bronchial and uterine smooth muscle Response to Receptor Activation Constriction Constriction Inhibit norepinephrine release Decreased sympathetic tone Increased heart rate Increased force of contraction Dilation Relaxation

All of the above There are two main types of adrenergic receptors: 1. Alpha (a) receptors - responsible for most of the excitatory effects such as vasoconstriction and contraction of the uterus and spleen. 2. Beta (B) receptors - responsible for most of the inhibitory effects such as vasodilation and relaxation of respiratory smooth muscle. Note: Two important EXCEPTIONS to the above: some alpha receptors mediate relaxation of gastrointestinal smooth muscle, and some beta receptors mediate increases in the force and rate of contractions of the heart. Alpha receptors fall into two groups: 1. Postjunctional alpha, adrenergic receptors are found in radial smooth muscle of the iris, arteries, arterioles, and veins; in the GI tract. 2. Prejunctional alpha, adrenergic receptors mediate the inhibition of the release of norepinephrine. Beta receptors fall into two groups: 1. Postjunctional beta, adrenergic receptors are found in the myocardium (heart), the intestinal tract smooth muscle, and adipose tissue. 2. Postjunctional beta2 adrenergic receptors are found in bronchiolar and vascular smooth muscle.

TM

PDF Editor

^PHARMACOLOGY

Adrenergic)

All of the following neurons are cholinergic EXCEPT one. Which one is the EXCEPTION?

Preganglionic sympathetic neurons


1

Preganglionic parasympathetic neurons Postganglionic sympathetic neurons Postganglionic parasympathetic neurons

10 Copyright 2011-2012 - Dental Decks

^PHARMACOLOGY

Adrenergic^

at substance produces all of the following physiologic actions?

1 Constricts arteriolar blood vessels (vasoconstriction)

Relaxes bronchial smooth muscle (bronchodilation) Decrease blood volume in nasal tissues
1

Causes a hypertensive response Produces physiological actions opposite to that of histamine

11 Copyright & 2011-2012 - Dental Decks

TM

PDF Editor

Postganglionic svmpathetic neurons


The autonomic nervous system (ANS) controls involuntary (smooth) muscle and gland activity. Blood pressure, pulse, sweating, bladder, and rectal sphincter tone are regulated by the ANS. Function of the heart, eyes, uterus, urinary bladder, and gastrointestinal tract, from the salivary glands to the anal sphincter, is governed and maintained by the ANS. The ANS is a complex set of neurons thai originate from the CNS (specifically the hypothalamus); it is composed of two antagonistic systems. The sympathetic nervous system (from thoraco-lumhar outflow) and the parasympathetic nervous system (from cranial-sacral outflow). The autonomic nervous system has cholinergic neurons that secrete acetylcholine and adrenergic neurons that secrete norepinephrine. Preganglionic neurons of both divisions (sympathetic and parasympathetic) are cholinergic, as arc the postganglionic neurons of the parasympathetic branch. Postganglionic neurons of the sympathetic branch arc usually adrenergic. One EXCEPTION is the sympathetic postganglionic neurons that innervate the sweat glands (they are cholinergic = secrete acetylcholine). Note: Adrenergic blocking agents block the effect of impulses transmitted by the adrenergic postganglionic neurons of the sympathetic branch. CNS Somatic Nervous System Striatcd (3 ^CAClT muscle Autonomic Nervous System Reproduced with permission, from Neidle EA, and Yagiela JA. Pharmacology* and Therapeutics for Dennstry. Mosby, 1989.

Sympathetic division -<AarQ _< A O r Q. < A C V ( Ad> j L medulla NE Parasympathetic division * Nicotinic site Muscarinic

Heart <NE Smooth muscle Glands


<ACht

GG-

Sweat glands Some blood vessels

OZH Hcan -<ACh 0-<ACh* Smooth muscle Glands

Fibers of the somatic and autonomic nervous systems, with the structures innervated by the different fibers and the chemical mediators responsible for transmission at the various loci. Dashed lines indicate postganglionic autonomic fibers.

Epinephrine Therapeutic indications for epinephrine: To alleviate symptoms of an acute asthma attack To treat bronchospasm associated with hypotension, as in anaphylaxis To treat hypersensitivity reactions Agent of choice for anaphylactic reactions (given sublingually or subcutaneously) It is added to local anesthetics as a vasoconstrictor to prolong the activity of the local anesthetic solutions, by decreasing the rate of diffusion and absorption from the injection site To restore cardiac activity in cardiac arrest To relieve congestion of the nose, sinuses, and throat Common side effects: headaches, agitation (anxiety), and tachycardia. Important: Epinephrine should be used with caution in patients with high blood pressure and hyperthyroidism. These patients may have an increased sensitivity to epinephrine.

TM

PDF Editor

MPHARMACOLOGY

Adrenergic)

Epinephrine is the agent of choice for treating an anaphylactic reaction because of its stimulatory effects on both alpha and beta adrenergic receptors. All of the following are desirable effects of epinephrine that make it the agent of choice for treating an anaphylactic reaction. It has vasopressor activity It has bronchodilator properties It causes increased cardiac output It has a rapid onset of action

The first statement is true; the second statement is false The first statement is false; the second statement is true Both statements are true Both statements are false
12 Copyright 0 2011-2012 - Dental Decks

MPHARMACOLOGY

Adrenergic)

"Epinephrine reversal" is a predictable result of the use of epinephrine in a patient who has received a/an:

Beta-blocker Alpha-blocker Adrenergic agonist All of the above

13 Copyright 2011-2012 - Dental Decks

TM

PDF Editor

Both statements are true Epinephrine is the agent of choice for treating an anaphylactic reaction because of its stimulatory effects on both alpha and beta adrenergic receptors. Alpha receptor stimulation results in a vasopressor response (elevation of blood pressure). Beta receptor stimulation results in air way dilation and increased cardiac output. In this way, epinephrine counteracts the vascular effects of histamine-related anaphylaxis. Note: Epinephrine is administered either intravenously, sublingually, subcutaneously or intramuscularly. There is a very rapid onset of action when given by these routes. Other therapeutic indications for the use of epinephrine: To control superficial hemorrhage As a component of local anesthetic solutions to prolong duration of anesthesia Asthma bronchodilator properties

Alpha-bloeker Explanation of answer: One of the best known effects of the alpha-receptor blocking agents (antiadrenergic) is their ability to reverse the pressor action of adrenaline (epinephrine). In the absence of blocking agents, epinephrine and norepinephrine both cause the blood pressure to rise. After the alpha-receptors have been blocked by an alphablocker, the pressor effect of norepinephrine is reduced or abolished while epinephrine brings about a fall in blood pressure. This is because epinephrine stimulates both alpha and beta-receptors in the cardiovascular system but norepinephrine only stimulates alpha-receptors (norepinephrine lacks beta2 effects). After blockage of the alpha-receptors, only the beta-receptors can be stimulated. Note: A pressor response (produces an increase in blood pressure) is mediated by alphareceptors and a depressor response (produces a decrease in blood pressure) is mediated by beta2 receptors.

TM

PDF Editor

MPHARMACOLOGY

Adrenergic)

Of the following, which is a clinically significant adverse reaction due to Metoprolol?

Drowsiness Hallucinations Arthralgia Dry mouth

14 Copyright 2011-2012 - Dental Decks

^PHARMACOLOGY

Adrenergic^

If norepinephrine or epinephrine were to stimulate or combine with the alpha receptors in the eye, which response would you expect?

Miosis (contraction of the pupil) Mydriasis (dilation of the pupil) Neither of the above; norepinephrine and epinephrine do not stimulate or combine with alpha receptors in the eye

15 Copyright 0 2011 -2012 - Dental Decks

TM

PDF Editor

Drowsiness Metoprolol (Lopressor, Toprol-XL) is a competitive, beta r selective adrenergic receptor blocker (cardioselective), which is most similar to Atenolol. It is used in the treatment of hypertension, acute angina pectoris and may be helpful after a heart attack. Atenolol (Tenormin) is a competitive, beta,-selective adrenergic receptor blocker (cardioselective), with a long plasma half life (long duration of action). It is used in the treatment of hypertension and chronic angina pectoris. Due to its low lipid solubility. Atenolol is renally eliminated, minimally metabolized, and has a low potential for causing CNS side effects compared to lipid soluble beta-blockers (e.g., Propranolol). *** Metoprolol and Atenolol are both longer-acting and more predictable than Propranolol in producing therapeutic plasma levels. Because they are beta, selective adrenergic receptorblockers, they are also safer to use in patients with a history of asthma or bronchitis. Acebutolol (Sectral) is a beta,-selective adrenergic receptor blocker (cardioselective). It is used to treat hypertension and to control ventricular arrhythmias. It has a low lipid solubility, which reduces its likelihood of producing adverse CNS effects. It also has mild intrinsic sympathomimetic activity (partial agonist activity at beta, receptors) similar to Pindolol. Important points: 1. With all "selective" beta-blockers, selectivity for the beta] receptor is lost at high doses. As the dose is increased they also block the beta2 receptors, thereby having effects on bronchial smooth muscle. 2. The most common side effects of beta-blockers are weakness and drowsiness.

Mydriasis (dilation of the pupil) Organ/Tissue Heart Receptor Type Response to Adrenergic Agonists Increases conduction velocity Increases contraction force Increases contraction rate Increases cardiac output Constricts cerebral arterioles Dilates skeletal muscle arterioles Contracts sphincter muscle, producing mydriasis Relaxes tracheal and bronchial muscles Decreases peristalsis Contracts sphincter Contracts trigone and sphincter muscles Relaxes detrusor muscle Excites uterine contractions Inhibits uterine contractions

P. P.
Pi Pi

Arterioles Eye Lung Intestine Urinary bladder Uterus

a, P2

a.
P2 1*1,2 P l j a.
ai

P.
a, P2

TM

PDF Editor

^PHARMACOLOGY

Adrenergic)

All of the following drugs are bronchodilators EXCEPT one. Which one is the EXCEPTION?

Epinephrine Levalbuterol (Xopenex) Albuterol (Proventil) Salmeterol (Serevent) Metaproterenol (Alupent) Histamine Aminophylline
16 Copyright 2011-2012 - Dental Decks

^PHARMACOLOGY

Adrenergic)

Pharmaceutical agents that bring about tissue responses resembling those produced by stimulation of the sympathetic nervous system are called?

Cholinomimetic Antiadrenergic Parasympathomimetic Sympathomimetic

17 Copyright 2011-2012 - Dental Decks

TM

PDF Editor

Histamine Asthma is a respiratory disorder characterized by recurring episodes of paroxysmal dyspnea, wheezing on expiration, coughing, and viscous mucoid bronchial secretions. The episodes may be precipitated by inhalation of allergens or pollutants, infection, vigorous exercise, or emotional stress. Management and prevention of an acute asthma attack: Epinephrine, levalbuterol, albuterol, salmeterol and metaproterenol are beta2 adrenergic receptor agonists. Thus they stimulate the beta receptors in the airway to cause bronchodilation. They are taken via aerosol inhalers and nebulizers. Important: Albuterol is the drug of choice as a "rescue" agent during an acute asthmatic attack. Note: Aminophylline is an example of a theophylline compound. Theophylline compounds are administered orally as bronchodilators in reversible airway obstruction due to asthma or COPD (chronic obstructive pulmonary disease). These drugs relax bronchial smooth muscle to improve airway function.

Sympathomimetic The terms sympathomimetics, sympathomimetic amines, adrenergic agonists and adrenergic agents are almost always synonymous. Epinephrine is the sympathomimetic agent used in dentistry. It is the vasoconstrictor for anesthetic solutions and the vasoconstrictor component in gingival retraction cords. As a component of local anesthetic preparations, it is used to prolong the duration of local anesthesia. In medicine, sympathomimetics are used as pressor agents to maintain blood pressure in vascular shock. They are used as bronchodilators for asthma attacks and for allergic states including anaphylactic shock. Sympathomimetics used in medicine include dopamine, epinephrine, norepinephrine, isoproterenol, and phenylephrine. Important: Epinephrine is indicated in medicine to treat bronchospasm and hypersensitivity reactions. It is the agent of choice for reversing anaphylactic reactions. It is used to restore cardiac activity in cardiac arrest.

TM

PDF Editor

MPHARMACOLOGY

Adrenergic)

The medical uses of a drug having adrenergic agonist actions would include which of the following?

1 Reverse

an anaphylactic reaction

1 Treat hypertension 1 Prevent angina pectoris

Reduce anxiety All of the above

18 Copyright 0 20! I-2012 - Dental Decks

(PHARMACOLOGY

Adrenergic)

A fear reaction activates the sympathetic division of the autonomic nervous system to result in:

Miosis Bradycardia Hypertension Increased salivation

19 Copyright O 2011-2012 - Dental Decks

TM

PDF Editor

Reverse an anaphylactic reaction The prototypical adrenergic agonist agent is epinephrine. Epinephrine stimulates both beta adrenergic receptors and alpha adrenergic receptors within the sympathetic division of the autonomic nervous system. Anaphylaxis is characterized by rapid, extreme reduction in blood pressure and bronchospasms. Epinephrine, upon injectable administration, will rapidly reverse the hypotension by causing vasoconstriction via the alpha] receptor stimulation; it will dilate the bronchial tubes via beta2 receptor stimulation; it will increase cardiac output via beta, receptor stimulation on the cardiac muscle. Epinephrine is: Ineffective in treating hypertension because of its alpha receptor stimulatory actions on the vasculature which could cause an even further elevation of blood pressure Contraindicated in angina conditions because its cardiostimulatory effects would aggravate this condition Will not reduce anxiety but will likely increase anxiety since it has central nervous system stimulatory effects

Hypertension Activation of the sympathetic portion of the autonomic nervous system will cause alpha,adrenergic receptor activation to result in arteriolar vasoconstriction with an associated elevation of blood pressure leading to hypertension. , Notes 1. Sympathetic activation of the eye would result in mydriasis (dilation) not miosis. 2. Sympathetic activation of the heart would result in tachycardia, not bradycardia. 3. Sympathetic activation of the salivary glands would result in a thick ropeytype salivary flow, not increased salivation.

Miosis (pupillary constriction), bradycardia, and increased salivation are physiological effects all resulting from activation of the parasympathetic division of the autonomic nervous system.

TM

PDF Editor

^PHARMACOLOGY

Anesthetics)

The amide local anesthetics are metabolized primarily in the:

Lungs Plasma Liver Kidney

20 Copyright 2011-2012 - Dental Decks

(PHARMACOLOGY

Anesthetics)

What are some adverse side effects of local anesthetics?

21 Copyright 2011-2012 - Dental Decks

TM

PDF Editor

The amide-type local anesthetics that are used in dentistry and metabolized by the liver include: Lidocaine (Xylocaine) Prilocaine (Citanest) Articaine (Ultracaine) Mepivacaine (Carbocaine) ' Bupivacaine (Marcaine) Etidocaine (Duranest) These anesthetics are metabolized by the hepatic microsomal enzyme system. The products formed do not have anesthetic actions and are excreted primarily in the urine as metabolites. These agents should be used with caution or not at all in patients with compromised liver function. Note: The most abundant urinary metabolite of lidocaine is 4-hydroxyxylidine. Amides are metabolized by three types of reactions: 1. Dealkylation of the amino terminus 2. Hydrolysis of the amide bond 3. Hydroxylation of the aromatic ring The amides vary in protein binding. Lidocaine and mepivacaine are bound moderately. Etidocaine and bupivacaine are highly bound. Note: Bupivacaine is more selective for sensory nerves than etidocaine. , . . . ... .
Product Carbocaine 3% (Mepivacaine) Citanest Plain 4% (Prilocaine) Citanest Forte w/epinephrine (Prilocaine 4% w/epinephrine 1:200,000) Lidocaine 2% w/epinephrine( 1:100,000) Marcaine 0.5% w/epinephrine (Bupivacaine w/epinephrine 1:200,000) Articaine 4% w/epinephrine (1:100,000) Infiltration 20 minutes 20 minutes 2.25 hours 60 minutes 60 minutes 60 minutes Inferior Alveolar Block 40 minutes 2.5 hours 3 hours 90 minutes 5-7 hours 60 minutes

Note: Bupivacaine has the longest duration of action of any dental local anesthetic presently available.

Adverse effects caused by local anesthetics are categorized into two major groups: 1. Effects resulting from toxicity 2. Effects resulting from allergy Toxicides: Too much anesthetic in the bloodstream can cause toxicities to the central nervous system (CNS) and cardiovascular system Opioids can increase the systemic toxicity of local anesthetics Esters will show greater toxicity in patients with a hereditary deficiency of plasma esterases. Remember: Esters are metabolized by the plasma enzyme plasma pseudocholinesterase, which is a plasma esterase. The CNS effects include: restlessness, stimulation, tremors, convulsive seizures followed by CNS depression, slowed respiration and even coma The cardiovascular effects include: bradycardia and reduction of cardiac output Allergy: Hypersensitivities or allergic reactions to local anesthetics, particularly from the amides, are rare. These reactions are manifested as dermatological reactions and edema at the injection site. Asthmatic wheezing syndromes have occurred in response to local anesthetic injections Allergic reactions are more prevalent with the ester-type rather than the amide-type anesthetics. They also display cross-allergenicity. Ester local anesthetic allergic manifestations include: nasolabial swelling, itching, and oral mucosal swelling Methylparaben, which was used as a perservative, can also cause allergies

TM

PDF Editor

PHARMACOLOGY

Anesthetics
>

Which statement best describes the mechanism of action of local anesthetics on the nerve axon?

Decreases sodium uptake through sodium channels of the axon Increases potassium outflow from inside to outside the nerve Increases the membrane's permeability to sodium Increases the excitability of the nerve axon

22 Copyright 20! I -2012 - Dental Decks

PHARMACOLOGY

icsj Anesthetics

Which form of a local anesthetic can readily penetrate tissue membranes?

Ionized form Nonionized free-base form Both ionized and nonionized free-base forms

23 Copyright 2011-2012 - Dental Decks

TM

PDF Editor

Decreases sodium uptake through sodium channels of the axon


When a local anesthetic solution is injected near the nerve, the solution interferes with the uptake of sodium from outside to inside the nerve. The local anesthetic blocks the specific sodium channels thereby blocking the sodium uptake. This decreases the nerve excitability below a critical level and nerve impulses fail to propagate along the axon. Since axons carry pain sensations, these sensations will not be carried and a blockage of pain results. 1. Local anesthetics reversibly block nerve impulse conduction and produce reversible loss of sensation at their administered site. 2. Local anesthetics have no effects on potassium at the nerve axon. 3. Small, nonmyelinated nerve fibers, which conduct pain and temperature sensations, arc affected first, followed by touch, proprioception (pressure), and skeletal muscle tone (motor).
Pharmacokinetics of Local Anesthetic!* Drug Short Acting Procaine Chloroprocainc Intermediate Acting Lidocaine Amide <2 min 30-60 min 64.3% B 0.5%. 1%. 2%. 4%. with and without epinephrine 1%, 1.5%, 2%. 3% 4% with and without epinephrine Ester Ester 2-5 min 6-12 min 15-60 min 30 min 5.8% C 1%,2%, 10% 1%.2%.3% Amide or ester Onset of Action Duration of Protein Pregnancy Action Bound Category How Supplied

Mepivacaine Prilocaine Long Acting Bupivacaine

Amide Amide

3-5 min <2 min

45-90 min 60-120 min

77.5% 55%

C B

Amide

5 min

2-4 hr

95.6%

0.25%. 0.5%, 0.75% with and without epinephrine 1%, 1.5% with and without epinephrine 1%.2%.3%

Etidocaine Tetracaine

Amide Ester

3-5 min 15 min

5-10hr 2-3 hr

94% 75.6%

Nonionized free-base form Local anesthetic free bases are fat-soluble (lipophilic) drugs. They are converted to their water-soluble (hydrophilic) hydrochloride salts to allow preparation of an injectable solution. In solution an equilibrium is established between the ionized and the nonionized forms of the local anesthetic. The proportion of the drug in the ionized form depends on the pKa of the drug and the pH of the solution. At the usual solution pH of 6.0 or lower, most local anesthetics are almost completely in the ionized form. This is important because only the nonionized free-base form of the drug can readily penetrate tissue membranes. Local anesthesia can only be obtained if sufficient free-base is available. The lower the pKa of the drug and the higher the pH of the solution or injected tissues, the more free-base will be available. Once the local anesthetic is injected the buffering capacity and pH of the tissues (normally!.4) shifts the equilibrium in favor of free-base formation. At physiologic pH of 7.4 approximately 5-20% of the local anesthetic is in free-base form which is enough to penetrate and cause anesthesia. If infection or inflammation is present, the pH of the tissues may be acidic and there is a significant reduction in the concentration of the free-base form. In this situation, the local anesthetic may not be effective. Key point of all this: The potential action of all local anesthetics depends on the ability of the anesthetic salt to liberate the free-base. Note: If you inject lidocaine (pKa = 7.8) into tissue that has a pH of 7.8, the lidocaine will exist in an equal mixture of ionized and nonionized forms which will be more than enough to produce anesthesia.

TM

PDF Editor

^PHARMACOLOGY

Anesthetics)

| A sedative often used in the management of anxious pediatric dental patients is:

Pentobarbital
1 Secobarbital

Chloral hydrate Meperidine

24 Copyright 2011-2012 - Dental Decks

^PHARMACOLOGY

Anesthetics)

Ester-type local anesthetics are metabolized where?

Liver Lungs Plasma Kidneys

25 Copyright 2011-2012 - Dental Decks

TM

PDF Editor

Chloral hydrate Traditionally chloral hydrate has been used orally in the preoperative management of the anxious pediatric dental patient. It has an unpleasant odor and a bitter, caustic taste, which can be partially masked in a flavored syrup. Chloral hydrate is rapidly absorbed after oral administration with an onset of 15 to 30 minutes. Its duration of action is about 4 hours. For use in children, chloral hydrate is available as a 500 mg/5 ml solution. The usual dose for a child is 50 mg/kg up to a maximum of lg. Important: Children will often enter a period of excitement and irritability before becoming sedated. It affects brain centers that control wakefulness and alertness. Chloral hydrate does not relieve pain. 1. Chloral hydrate is a prodrug and is metabolized to the active metabolite, trichloroethanol. /Notea# 2. Chloral hydrate's metabolite (trichloroethanol) may displace warfarin from its protein binding sites resulting in an increase in the hypoprothrombinemic response to warfarin. 3. Sedative effects and/or respiratory depression with chloral hydrate may be additive with other CNS depressants monitor for increased effect. These other CNS depressants include ethanol, antidepressants, narcotic analgesics, and benzodiazepines. 4. Be careful when using Chloral hydrate, serious toxicity can result if the dose is not controlled.

Ester-type local anesthetics all have an ester grouping within their chemical structure. An ester grouping is essentially a bridge or link containing the - COOCH2-configuration. Amide-type local anesthetics all have an amide grouping within their chemical structure. An amide grouping is essentially a bridge or link containing the - CONHCH2-configuration. Ester-type local anesthetics are mainly available as topical anesthetics and include benzocaine (which does not have an amino terminus and therefore does not become charged, thus it is poorly soluble in water), tetracaine, and dibucaine. They are also available as medical anesthetic preparations such as propoxycaine (Ravocaine). Ester-type local anesthetics are no longer available as dental anesthetic injectable preparations because of their relatively high incidence of allergy. Only the amides are presently available as dental injectable local anesthetic agents. These include lidocaine, mepivacaine, prilocaine, bupivacaine, and articaine. Ester-type local anesthetics are metabolized by the plasma enzyme plasma cholinesterase. Another name for this enzyme is pseudocholinesterase. This enzyme splits the ester linkage within the chemical structure rendering the anesthetic ineffective. Procaine (Novocaine) was one of the original ester-type anesthetics. When procaine is metabolized by plasma cholinesterase, a highly allergic compound called para aminobenzoic acid (PABA for short) is formed. Many patients developed an allergy to PABA. Note: PABA can decrease the effectiveness of sulfonamides (antibiotics). Note: Other chemicals that act like local anesthetics: Diphenhydramine: H| antihistamine Tetrodotoxin: found in a number of tissues of the puffer or blowfish Saxitoxin: is produced by certain strains of algae Remember: Dyclonine hydrochloride is unusual in that it has a ketone linkage between the aromatic moiety and the rest of the anesthetic molecule. It is used as a topical.

TM

PDF Editor

^PHARMACOLOGY

Anesthetics)

Which of the following is a contraindication or precaution to the use of Prilocaine?

Biliary tract disease Type II diabetes Rheumatoid arthritis Hepatic disease

26 Copyright 2011-2012 - Dental Decks

(PHARMACOLOGY

Anesthetics)

(f

Which local anesthetic has no place in the routine practice of dentistry?

Articaine Cocaine Lidocaine Bupivacaine Prilocaine

27 Copyright O 2011-2012 - Dental Decks

TM

PDF Editor

Hepatic disease *** Remember: All amides are metabolized primarily in the liver, and the metabolites are then renally excreted. Prilocaine (Citanest) is a local anesthetic of the amide class, used for nerve block, epidural and regional anesthesia. It has an intermediate duration of action and is longer acting than Lidocaine. Prilocaine produces less vasodilation than do equal amounts of Lidocaine (it is somewhat less potent than Lidocaine). It is available as a 4% solution with or without epinephrine, which prolongs the anesthetic effect. Prilocaine is metabolized to orthotoluidine, a product that can produce methemoglobinemia, a condition that is characterized by increased levels of methemoglobin in the blood. Note: This methemoglobin is less effective than hemoglobin in carrying oxygen in the blood. Prilocaine is about one-half as toxic as Lidocaine, but since methemoglobinemia is a possible reaction, Prilocaine is not used for patients with hypoxic conditions of any kind. Note: EMLA Cream is a eutectic mixture of lidocaine 2.5% and prilocaine 2.5% formulated as an oil in water emulsion. In this eutectic mixture, both anesthetics are liquid at room temperature and the penetration and subsequent systemic absorption of both prilocaine and lidocaine are enhanced over that which would be seen if each component in crystalline form was applied separately as a 2.5% topical cream.

Cocaine Cocaine is a naturally occurring ester of benzoic acid and was the first local anesthetic used in dentistry and medicine. It is potent and extremely toxic, and it is the only local anesthetic that causes definite vasoconstriction (all other local anesthetics are vasodilators). Cocaine is commercially available in a variety of forms and is applied to mucous membranes of the oral, laryngeal, and nasal cavities for use as a topical anesthetic. Cocaine causes significant euphoria (most likely due to its blockade of reuptake of dopamine in the brain), and abuse can lead to a physical dependence. Despite being an excellent local anesthetic, the risk of abuse and the intense local vasoconstriction it produces prevent cocaine from being more widely used clinically. Important: Cocaine has no place in the routine practice of dentistry. The pharmacology of cocaine is unique among local anesthetics in that the drug inhibits the uptake of catecholamines by adrenergic nerve terminals. It therefore potentiates the action of endogenously released and exogenously administered sympathomimetic amines such as dobutamine, dopamine, or epinephrine. It increases the risk of developing cardiac arrhythmias and hypertension (i.e., cocaine increases the pressor activity of these sympathomimetic amines).

TM

PDF Editor

PHARMACOLOGY (J

icsl Anesthetics

Local anesthetics theoretically should be less effective in acutely inflamed tissue than in normal tissue because in inflamed tissue what happens?

The pH rises, thus inactivating the anesthetic The pH rises, thus decreasing available free base The pH decreases, thus decreasing available free base The pH remains the same, the extracellular fluid dilutes the anesthetic

28 Copyright 2011-2012 - Dental Decks

QPHARMACOLOGY

ics) Anesthetics

A dental anesthetic carpule contains 1.8 ml of a 2% solution of lidocaine with 1:100,000 epinephrine. How much lidocaine and epinephrine does the carpule contain?

3.6 mg lidocaine and 0.18 mg epinephrine 3.6 mg lidocaine and 0.018 mg epinephrine 36 mg lidocaine and 0.18 mg epinephrine 36 mg lidocaine and 0.018 mg epinephrine

29
Copyright 2011-2012 - Dental Decks

TM

PDF Editor

The pH decreases, thus decreasing available free base At body pH (7.4), a local anesthetic when infiltrated, will chemically exist as a portion which is ionized (has a proton attached) and as a portion which is non-ionized (has no proton attached). Note: A proton is nothing more than a hydrogen (H+) atom. That portion which is ionized has difficulty penetrating the nerve and will not be effective. That portion which is non-ionized will penetrate the nerve to cause anesthesia. That portion which is non-ionized is also known as the free base. The more proportion of the anesthetic which is in the free base form, the more effective it will be. When tissue conditions are normal (pH 7.4), approximately 10-20% portion of an infiltrated local anesthetic is in the form of the free base (non-ionized form). This is enough to penetrate the nerve to cause anesthesia. When tissues are acidic, as in the case of tissue infection, less free base portion exists and more ionized portion is present. There is not enough free base form to penetrate the nerve to cause anesthesia. Therefore, the local anesthetic when infiltrated to the tissue site is not effective at the normal anesthetic doses.

36 mg lidocaine and 0.018 mg epinephrine Key to question: 1 ml of a 2% solution of lidocaine with 1:100,000 epinephrine contains 20 mg of lidocaine and 0.010 mg of epinephrine. A dental carpule contains 1.8 ml solution. Therefore, 1.8 ml of 2% solution of lidocaine with 1:100,000 epinephrine contains 36 mg of lidocaine and 0.018 mg epinephrine. Epinephrine (vasoconstrictor) is included in local anesthetics for the following reasons: It prolongs the duration of action It reduces toxicity It reduces the rate of vascular absorption It provides a hemostatic effect to reduce bleeding at injection site

TM

PDF Editor

(PHARMACOLOGY

Anesthetics)

The maximal recommended adult dose of Lidocaine is 300 mg. How many milliliters of 2% Lidocaine need to be given to reach this level?

7.5 milliliters 10 milliliters 15 milliliters 20 milliliters

30
Copyright 2011-2012 - Dental Decks

(PHARMACOLOGY

Anesthetics)

The maximum recommended dose of a local anesthetic that can be administered to a child < 10 years of age is determined by:

Age Weight Height Gender

31 Copyright 2011-2012 - Dental Decks

TM

PDF Editor

15 milliliters Important information to solve questions of this type: There are 20 mg of Lidocaine in every milliliter of 2% Lidocaine 300 mg (maximal recommended dose) + 20 mg (in every mL) = 15 mLs 20 mg x 1.8 mL (in every carpule) = 36 mg / 1 carpule 300 mg + 36 mg = 8.3 carpules (8 carpules of 2% Lidocaine can be used)
Maximum Recommended Doses of Local Anesthetic Drug Maximal Recommended Dose mg ml 300 300 400 40 15 10 10 18 Concentration Used (%) 2 3 4 0.5 Approximate Number of Carpules 8.3 5.6 5.6 10

Lidocaine (Xylocaine) Mepivacaine (Carbocaine) Prilocaine (Citanest) Bupivacaine (Marcaine)

Remember: Lidocaine has serious drug interactions with beta-blockers and cimetidine that decrease lidocaine clearance 30% or more. Propranolol, metoprolol, and nadolol are reported to reduce lidocaine clearance due to the decrease in cardiac output caused by the beta-blockers. Decreased cardiac output results in reduced liver blood flow which explains the decline in lidocaine clearance caused by these drugs. Cimetidine also decreases lidocaine clearance, but the mechanism of the interaction is different. Because cimetidine does not change liver blood flow, it is believed that cimetidine decreases lidocaine clearance by inhibiting hepatic microsomal enzymes. Lidocaine clearance may be accelerated by concomitant use of phenobarbital or phenytoin. Both of these agents are known to be hepatic drug metabolizing drug inducers, and this is the probable mechanism of their drug interaction with lidocaine.

Weight(mg/kg) Example: For Lidocaine (2%) with epinephrine, a dosage of 4.4 mg/kg should not be exceeded (maximum is 300 mg).

Maximum Recommended Doses of Lidocaine for Children < 10 Years Old Patient Weight (kg/lb) 10/23 15/34.5 20/46 25/57.5 Mg 44 66 88 100 Number of Carpules 1.2 1.8 2.4 2.6

Note: 1 kg = 2.3 lbs 2% = 20 mg / mL x 1.8 mL /I carpule = 36 mg/1 carpule

TM

PDF Editor

(PHARMACOLOGY

Anesthetics)

Local anesthetics depress which of the following nerve fibers first?

Large myelinated fibers Small unmyelinated fibers Small myelinated fibers Large unmyelinated fibers

32 Copyright 2011 -2012 - Dental Decks

(PHARMACOLOGY

Anesthetics J

The maximum dose of articaine (Septocaine) that is recommended in one appointment is expressed as mg per kilogram body weight rather than total milligrams. Which dose is the maximum recommended dose for articaine in children and adults:

7 mg/kg 100 mg/kg 1 mg/kg 300 mg/kg

33 Copyright 2011-2012 - Dental Decks

TM

PDF Editor

Small unmyelinated fibers *** These conduct pain and temperature Important: They depress the large myelinated fibers last. The small nerves have the greater surface-volume ratio (this accounts for the rapid onset of action). Clinically the general order of loss of function is as follows: l.Pain 2. Temperature 3. Touch 4. Proprioception 5. Skeletal muscle tone Note: Pain threshold refers to the lowest level of pain a patient will detect. Remember: Local anesthetics reversibly block nerve impulse conduction and produce the reversible loss of sensation at their administration site. They do not produce a loss of consciousness. They appear to become incorporated within the nerve membrane or to bind to specific membrane sodium ion channels, restricting sodium permeability in response to partial depolarization.

7 mg/kg In a typical 70 kg adult male, the dose of 7 mg/kg would equate to a total of 490 mgs. Thus, the maximum recommended amount of articaine that could be given to a 70 kg adult in one appointment is 490 mgs. The table below shows the following number of dental cartridges containing 1.7 ml volume of solution to provide the indicated amounts of articaine hydrochloride 4% and epinephrine 1:100,000. Number of Cartridges (1.7 ml) 1 2 3 4 5 6 7 8 Articaine HCL (4%) (mg) 68 136 204 272 340 408 476 544 Epinephrine 1:100,000 (mg) 0.017 0.034 0.051 0.068 0.085 0.102 0.119 0.136

TM

PDF Editor

(PHARMACOLOGY

Anesthetics)

Which is the most frequently utilized route of administration for sedation in pediatric patients?

Oral Inhalation IV IM

34 Copyright 2011-2012 - Dental Decks

(PHARMACOLOGY

Anesthetics)

All of the following are advantages of using nitrous oxide analgesia EXCEPT one. Which one is the EXCEPTION? Rapid onset of action Lowers pain threshold Produces euphoria Pleasant induction Titra table Rapid and complete recovery Virtually no adverse effects in absence of hypoxia Therapeutic sedative for many medically compromised patients Suitable for all ages
35 Copyright 2011 -2012 - Dental Decks

TM

PDF Editor

Inhalation *** The agent used most frequently is nitrous oxide. Nitrous oxide is a slight sweet smelling, colorless, inert gas. It must always be coupled with no less than 20% oxygen. Nitrous oxide is quickly absorbed from the lungs and is physically dissolved in the blood. There is no biotransformation, and the gas is rapidly excreted by the lungs when the concentration gradient is reversed. It is recommended that the patient be maintained on oxygen for 5 to 10 minutes after the sedation period. Dose response for nitrous oxide (always given in mixture with oxygen): 10%) - 2 0 % : tingling of hands, feet, body warmth 20%o - 40%: mild sleepiness, relaxation, some analgesia, mind dissociation, heightened auditory perception Above 50%: this is too much nitrous nausea, sweating *** Rapid onset: 5 minutes *** Rapid recovery: 5 minutes Note: Nitrous oxide does not provide enough analgesic effect to preclude the use of local anesthesia in dental surgery or restorative procedures. Local anesthetics must be used along with nitrous oxide. Remember: Nitrous oxide has minimal depressant effects on the cardiovascular system and no skeletal muscle relaxant properties.

Lowers pain threshold *** Nitrous oxide elevates pain threshold Important points about nitrous oxide: The main therapeutic effect of nitrous oxide is relaxation/sedation (it is used in conscious sedation). Mild analgesia is a secondary effect. It is not a respiratory depressant. The first symptom of nitrous oxide onset is tingling of the hands. Nitrous oxide has no local anesthetic properties. Therefore, the addition of local anesthesia is necessary in procedures in which pain is anticipated. Long term exposure to low doses of nitrous oxide has been shown to increase the incidence of spontaneous abortions. Environmental contamination by nitrous oxide can be kept to a minimum by employing a scavenger system. Nitrous oxide is stored under pressure (750 psi) in steel cylinders (in a liquid state) painted blue. Oxygen is stored in green tanks. Nitrous oxide delivery machines come pre-equipped with a failsafe mechanism that will not allow less than 20% oxygen to be delivered to the patient. It is a nonirritating, colorless gas with a slightly sweet odor and tasteless. It is very stable and inert chemically at room temperatures. It has a rapid onset (blood.gas solubility coefficient = 0.47) and termination. It is 1.5 times heavier than air. Nitrous oxide will diffuse into air-containing cavities within the body faster than nitrogen diffuses out. This results in a temporary increase in either the pressure and/or volume of the cavity depending upon the distensibility of its walls. This is most noticeable in the bowel. Nitrous oxide interacts with vitamin B ) 2 synthesis in the human body by interfering with the enzyme methionine synthase, depleting the body of vitamin B| 2 . When nitrous oxide is used heavily and over an extended period of time, vitamin B12 depletion will probably become a major problem, as it can cause brain and nerve damage.

TM

PDF Editor

(PHARMACOLOGY

Anesthetics)

AH the following are considered to be what type of anesthetic?

1 Halothane

Enflurane Isoflurane Sevoflurane Desflurane

36
Copyright 2011-2012 - Dental Decks

(PHARMACOLOGY

Anesthetics)

f List the three reasons why epinephrine is included in dental local anesthetics.

37 Copyright 2011-2012 - Dental Decks

TM

PDF Editor

Inhalation anesthetics
Inhalation anesthetics arc drugs which arc vaporized from the liquid form and inhaled to produce general anesthesia. These diverse drugs arc relatively simple lipophilic molecules ranging from ethers such as halogenated hydrocarbons such as halothanc and halogenated ethers such as isofluranc. cnfluranc, scvofluranc, and dcsfluranc. Note: Vaporization of these liquids occurs in a vaporizer and delivery to the patient occurs through an anesthesia machine via a nasal mask. Essential Components of Anesthesia: Analgesia - perception of pain eliminated Hypnosis - unconsciousness Depression of spinal motor reflexes Muscle relation Hypotheses of General Anesthesia: Lipid Theory: based on the fact that anesthetic action is correlated with the oil/gas coefficients. The higher the solubility of anesthetics is in oil, the greater is the anesthetic potency. Protein (Receptor) Theory: based on the fact that anesthetic potency is correlated with the ability of anesthetics to inhibit enzymes activity of a pure, soluble protein. Also, attempts to explain the GABA A receptor is a potential target of anesthetics acton. Binding theory: anesthetics bind to hydrophobic portion of the ion channel. Pharmacokinetics of Inhaled Anesthetics: Amount that reaches the brain - Indicated by oil:gas ratio {lipidsolubility) Partial Pressure of anesthetics - $% anesthetics = 38 mmHg Solubility of gas into blood (hlood:gas solubility coefficient) - The lower the blood:gas ratio, the more anesthetics will arrive at the brain Cardiac Output - Increased CO = greater induction time MAC Imean alveolar concentration): A measure of potency 1 MAC is the concentration necessary to prevent responding in 50% of population Values of MAC are additive: - Avoid cardiovascular depressive concentration of potent agents

Epinephrine, a vasoconstrictor, is included in dental local anesthetic preparations for the following three reasons: 1. It prolongs the duration of local anesthesia. 2. To provide a hemostasis such that local bleeding is controlled or reduced. 3. To delay the absorption of anesthetic into the systemic circulation thus reducing the chance of systemic toxicities.

TM

PDF Editor

(PHARMACOLOGY

Anesthetics)

Which stage of general anesthesia begins with unconsciousness?

Stage I Stage II Stage III Stage IV

38 Copyright C 2011-2012 - Dental Decks

(PHARMACOLOGY

Anesthetics)

Nitrous oxide is used as a single agent to produce general anesthesia. Nitrous oxide is used to produce sedation and mild analgesia.

The first statement is true; the second statement is false The first statement is false; the second statement is true Both statements are true Both statements are false

39 Copyright 2011-2012 - Dental Decks

TM

PDF Editor

Stage II
Four stages of general anesthesia: Stage I: Amnesia/analgesia Stage II: Delerium: begins with unconsciousness and ends with loss of eyelid reflex, purposeless movements and hyper-reaction, dilated pupils, reflex vomiting, tachycardia, and hypertension Stage III: Surgical anesthesia: four planes with progressive loss of reflexes and muscle control Stage IV: Medullary paralysis: cessation of respiration, ending with death without proper treatment Note: These four stages of anesthesia apply to the inhalants and not to the intravenous general anesthetics. Agents used for general anesthesia: Inhalation agents: volatile halogen-containing liquids - Haiothane: poses a risk with epinephrine; associated with hepatitis: poor skeletal muscle relaxant. However, it is one of the more widely used anesthetics for general surgical anesthesia. - Desflurane, sevoflurane, isoflurane and enflurane: pose less risk with epinephrine; not associated with hepatitis: good skeletal muscle relaxants. Note: Nitrous oxide (N20) is not considered a general anesthetic since hypoxic levels are required to produce anesthesia. It is considered a sedative. It is used alone to produce sedation or in combination with the above agents to supplement the anesthetic response. It is a gas at room temperature and pressure. Intravenous agents: Ultrashort-acting barbiturates: thiopental (Pentothal) and methohexital (Brevital) Benzodiazepines: diazepam (Valium), midazolam (Versed), and lorazepam (Ativan) Neuroleptic-opioid combinations which are called neuroroleptanalgesics combine fentanyl (opioid) and droperidol (a phenothiazine) Other agents: Propofol (Diprivan): a short-acting hypnotic agent; vasodilator Ketamine (Ketalar): is considered a dissociative anesthetic. It blocks /V-methyl-D-asparate (NMDA) (glutamate) recpetors. Note: In some circumstances, ketamine has been known to produce illusions or hallucinations that are enhanced by environmental stimuli upon emergence from anesthesia. Diazepam is given with ketamine to avoid this. Etomidate (Amidate): is a carboxylated imidazole derivative which acts similar to the ultrashortacting barbiturates.

The first statement is false; the second statement is true Nitrous oxide is unable to produce general anesthesia except if it is given at concentrations greater than 80%. At these concentrations, the lack of oxygen would cause hypoxia in the patient. Inhalant anesthetics such as haiothane and isoflurane can produce general anesthesia at concentrations approximating 3-5%. As such they are very useful in anesthesia. Nitrous is used to produce sedation and mild analgesia. It is usually used in concentrations of 3050% along with pure oxygen. It is a colorless, nonirritating gas at room temperature and pressure. and is not flammable nor explosive. The onset of sedation is within 5 minutes and the recovery is just as rapid. It is excreted unchanged by the lungs. The most common complaint from patients taking nitrous oxide is mild nausea. Note: Always give patient 100% oxygen at the end of the procedure to prevent diffusion hypoxia. Adverse effects of nitrous oxide: Decreased mental performance, audiovisual ability, and manual dexterity At high doses and/or high exposure: reduced fertility, spontaneous abortions, neurological and kidney disease as well as bone marrow suppression Nitrous oxide is contraindicated in patients: Head injury With bowel obstruction Pneumothorax Middle ear and sinus disease With upper respiratory infections COPD (emphysema or bronchitis) In the first trimester of pregnancy With whom communication is difficult (i.e., autistic patients) Having a contagious disease since it is difficult to sterilize entire tubes Important: Prolonged exposure (e.g., more than 24 hours) causes bone marrow depression.

TM

PDF Editor

(PHARMACOLOGY

Anesthetics)

Which component of a lidocaine local anesthetic solution causes an allergy?

Water Bisulfites Lidocaine Epinephrine (vasoconstrictor)

40 Copyright 2011-2012 - Dental Decks

^PHARMACOLOGY

Anesthetics)

Of the amide-type local anesthetics, which is the only one that is metabolized in the bloodstream rather than the liver?

Articaine (Septocaine, Zorcaine) Mepivacaine (Carbocaine) Lidocaine (Xylocaine) Prilocaine (Citanest) Bupivacaine (Marcaine)

41 Copyright 2011-2012 - Dental Decks

TM

PDF Editor

Bisulfites Patients may exhibit hypersensitivity to sulfites contained in some anesthetic solutions. Sodium metabisulfite prevents the oxidation (deterioration) of the epinephrine vasoconstrictor in those commercial preparations containing epinephrine. Most of the patients reacting to bisulfites have a history of asthma and the airway is hyperactive to the sulfites. Allergic reaction usually results in an asthmatic syndrome of wheezing and bronchial constriction. Bisulfites are present in only those commercial preparations containing vasoconstrictor. Preparations without vasoconstrictor such as mepivacaine 3% (Carbocaine 3%) do not contain bisulfites. Important: All local anesthetics except cocaine are vasodilators, however, mepivacaine has less of a vasodilator effect compared to the others and, therefore, is the drug chosen when a vasoconstrictor is not used with the local anesthetic. Note: Hypersensitivity or allergic reactions to local anesthetics, particularly the amides, are much more rare than allergic reactions to the bisulfites.

Articaine (Septocaine, Zorcaine) Articaine (Septocaine, Zorcaine) is an amide-type local anesthetic. However, it is chemically unique in that it has an ester group attached to its molecule which can be acted upon by plasma cholinesterase to render it ineffective. Therefore, it is the only amide which is metabolized in the bloodstream and not the liver. Rapid metabolism of this ester bond gives it a short half life. Articaine is supplied as articaine HCL 4% solution with epinephrine 1:100,000 and as articaine HCL 4% solution with epinephrine 1:200,000. It is indicated for local, infiltrative, or conductive anesthesia in both simple and complex dental and periodontal procedures. The onset of anesthesia following administration of articaine has been shown to be 1 to 6 minutes after injection. Complete anesthesia lasts approximately 1 hour. Articaine is contraindicated in patients with hypersensitivity to local anesthetics of the amide type or to sodium bisulfite.

TM

PDF Editor

(PHARMACOLOGY

Anesthetics)

When phentolamine mesylate (OraVerse) is given after a dental procedure to reverse the local anesthetics effects, approximately how fast is the return to normal nerve sensation compared to the normal time for the anesthetic to wear off?

10% decrease in the time for normal sensation to return 25% decrease in the time for normal sensation to return 50% decrease in the time for normal sensation to return 80% decrease in the time for normal sensation to return

42 Copyright 2011-2012 - Dental Decks

(PHARMACOLOGY

Anesthetics)

Phentolamine mesylate (OraVerse) is a drug used to reverse soft tissue anesthesia and the associated functional deficits resulting from a local dental anesthetic containing a vasoconstrictor. Which statement best describes its mechansim of action?

Prevents the influx of sodium into the neuron by blocking neuronal channels Causes vasodilation and increased blood flow in injection area Prevents the efflux of sodium out of the neuron Causes vasoconstriction and decreased blood flow in injection area

43 Copyright 2011 -2012 - Dental Decks

TM

PDF Editor

50% decrease in the time for normal sensation to return Thus if a local anesthetic such as lidocaine with epinephrine normally takes about 180 minutes to wear off, after phentolamine injection, the anesthetic will wear off in approximately 90 minutes. This represents a 50% decrease in time for normal sensation to be retored. Note: OraVerse dose is based upon the number of cartridges of local anesthetic with vasoconstrictor administered. 1/2 a cartridge (0.2mg) of OraVerse if 1/2 cartridge of anaesthetic was administered 1 cartridge (0.4mg) of OraVerse if 1 cartridge of anesthetic was administered 2 cartridges (0.8mg) of OraVerse if 2 cartridges of anesthetic were administered Note: OraVerse (solution for injection/dental cartridge) is administered as a submucosal injection.

Causes vasodilation and increased blood flow in injection area Phentolamine mesylate (OraVerse) is an alpha,-adrenergic receptor blocker. Phentolamine mesylate competitively blocks alpha-adrenergic receptors to produce brief antagonism of circulating epinephrine and norepinephrine and antagonism of the vasoconstrictor in the anesthetic preparation. This results in vasodilation of the blood vessels. The local anesthetic is thus carried away at a more rapid rate from the injection site resulting in a more rapid return to normal nerve sensation.

TM

PDF Editor

(PHARMACOLOGY

Anti-Anxiety AgentsJ

All of the following drugs are benzodiazepines EXCEPT one. Which one is the EXCEPTION?
Chlordiazepoxide (Librium) Diazepam (Valium) Flurazepam (Dalmane) Midazolam (Versed) Alprazolam (Xanax) Zolpidem (Ambien) Triazolam (Halcion) Oxazepam (Serax) Temazepam (Restoril) Clorazepate (Tranxene)
44 Copyright 2011-2012 - Dental Decks

(PHARMACOLOGY

Anti-Anxiety Agents J

The most common adverse effects associated with the benzodiazepines include | all of the following EXCEPT one. Which one is the EXCEPTION? J

CNS depression (drowsiness and sedation) GI disturbances (nausea, vomiting and diarrhea) Confusion Respiratory depression Disorientation Ataxia

45 Copyright 2011-2012 - Dental Decks

TM

PDF Editor

Zolpidem

(Ambien)

The benzodiazepines are used as oral preparations to alleviate anxiety and to induce sleep, and intravenously to cause conscious-sedation for outpatient surgery. Benzodiazepines act by potentiating the action of GABA, an amino acid and an inhibitory neurotransmitter, which results in increased neuronal inhibition and CNS depression. Tolerance and physical dependence can occur with prolonged high dosage. They seem to be much safer than barbiturates. Other useful properties include being an anti-convulsant and a skeletal muscle relaxant. Pharmacological effects of benzodiazepines: antianxiety, sedation, anticonvulsant, amnesia, and skeletal muscle relaxation. Adverse effects: CNS effects - fatigue, slurred speech, drowsiness/sleepiness and confusion (do not drive a car) GI effects - dry mouth, nausea Other effects - hypotension, muscle relaxation Benzodiazepines used orally as tranquilizers: chlordiazepoxide (Librium), diazepam (Valium), alprazolam (Xanax) and lorazepam (Ativan). Those used as hypnotics to overcome insomnia: flurazepam (Dalmane) and triazolam (Halcion). Diazepam (Valium) and Clonazepam (Klonopin) can be used as anticonvulsants. Midazolam (Versed) comes as a liquid for preoperative sedation in children and as injectable for IV conscious sedation. Note: Benzodiazepines should never be taken with any form of alcohol. Serious potentiation of the sedative effects of each will occur leading to unexpected inebriation and respiratory depression. Important: Zolpidem (Ambien), eszopiclone (Lunesta), and zaleplon (Sonata) are anti-insomnia agents (called GABA-BZ Agonists). These agents appear to act through the potentiation of GABA on benzodiazepine receptors, especially the omega-1 subunit. Note: They have shown no potential for causing addiction.

Respiratory depression The benzodiazepines have clinically useful antianxiety, sedative-hypnotic, anticonvulsant and skeletal muscle relaxant properties. They exert their main effect on central GABA-nergic neurons. With normal dosing, the benzodiazepines have little effect on respiratory systems in healthy individuals. The benzodiazepines (especially diazepam, lorazepam and midazolam) are important adjuncts in the practice of anesthesiology. They may be used as preoperative sedatives and induction agents, as well as supplemental agents for the maintenance of anesthesia. Important: The benzodiazepines, barbiturates and narcotic analgesics all produce sedation and have the ability to produce dependence. The pharmacokinetics of individual benzodiazepines differ, and therefore there is a wide range in speed of onset and duration of action among these compounds. Following oral administration, most of the benzodiazepines are rapidly absorbed and highly bound to plasma protein. In general, most of the metabolites are conjugated with glucuronic acid and excreted in the urine and feces. Note: a-Hydroxylation is a rapid route of metabolism that is unique to triazolam, midazolam, and alprazolam. This accounts for the very rapid metabolism and short sedative actions of these drugs. 1. The term tranquilizer refers to a drug that promotes tranquility by calming, soothing, quieting or pacifying without sedating or depressant effects. Antipsychotic agents are considered to be major tranquilizers and antianxiety agents (benzodiazepines) are considered to be minor tranquilizers. 2. For the treatment of insomnia due to anxiety the benzodiazepines flurazepam, temazepam and triazolam are usually prescribed. 3. Important: Flumazenil (Mazicon), a benzodiazepine antagonist, may be used to reverse the residual effects of benzodiazepines in the event of an overdose. 4. If the benzodiazepines are ineffective in helping with insomnia, Zolpidem (Ambien), eszopiclone (Lunesta), or zaleplon (Sonata) are often useful. These are anti-insomnia agents (called GABA-BZ Agonists).

TM

PDF Editor

(PHARMACOLOGY

Anti-Anxiety Agents)

Buspirone (BuSpar) is a partial agonist at a specific:

1 GABA (gamma-aminobutyric acid) receptor 1 Norepinephrine receptor

Dopamine receptor Serotonin receptor

46 Copyright 2011-2012 - Dental Decks

(PHARMACOLOGY

Anti-Anxiety AgentsJ

Which of the following side effects are most common with the use of Diazepam (Valium)?

Rash, itch Mouth, throat ulcers Drowsiness, fatigue Difficulty with urination

47 Copyright 0 2011 -2012 - Dental Decks

TM

PDF Editor

Serotonin receptor Buspirone (BuSpar) is an orally administered antianxiety agent with a short half-life (24 hours). It is not chemically related to the benzodiazepines, or any other anxiolytic agent. Buspirone also differs from other antianxiety agents in that it does not possess anticonvulsant or muscle relaxant properties, does not impair psychomotor function and does not cause sedation (lacks CNS depressant activity) or physical dependence. The exact mechanism of action of buspirone is unknown. However, it has a higher affinity for serotonin receptors in the CNS and a lesser affinity for the benzodiazepineGABA receptors. Important: The antianxiety effect is achieved via a partial agonist effect on CNS serotonin 5 -HT l a receptors that occurs without affecting the benzodiazepine receptors or causing CNS depression. Remember: GABA-BZ agonists have a chemical structure that is dissimilar from those of the benzodiazepines and on the sedative hypnotics. These agents appear to act through the potentiation of GABAon benzodiazepine receptors, especially the omega-1 subunit. They are used primarily for insomnia. They have exhibited some anxiolytic action but they have little effect on skeletal muscle or seizure thresholds. They appear to have minimal disruptive action on the normal sleep cycle, thus preserving deep sleep. Note: They have shown no potential for causing addiction. Examples include Zolpidem (Ambien), eszopiclone (Lunesta), and zaleplon (Sonata). These drugs have short half-lives (1 to 2 hours).

Drowsiness, fatigue Diazepam is prescribed in the treatment of anxiety, nervous tension, muscle spasm, and as an anticonvulsant. Contraindication to use: Acute narrow angle glaucoma. It is used intravenously as the agent of choice to reverse status epilepticus induced by a local anesthetic overdose. Diazepam affects the limbic system of the brain (controls emotion). It has a high therapeutic index, produces some degree of amnesia, and can be locally irritating to tissue and may produce local thrombophlebitis when given intravenously. Note: Propylene glycol, which is in the TV mixture, is the main cause of thrombophlebitis.

TM

PDF Editor

(PHARMACOLOGY

Anti-Anxiety Agents)

The barbiturates Phenobarbital (Luminal), Mephobarbital (Mebaral), and Primidone (Mysoline) are classified as what type of barbiturate?

Ultrashort-acting Short-acting Intermediate-acting Long-acting

48 Copyright C 2011-2012 - Dental Decks

(PHARMACOLOGY

Anti-Anxiety Agents J

The brief duration of general anesthetic action of an ultra-short-acting barbiturate is due to what factor?

Rapid rate of metabolism in the liver Low lipid solubility, resulting in a minimal concentration in the brain High degree of binding to plasma proteins Rapid rate of redistribution from the brain to peripheral tissues Slow rate of excretion by the kidneys

49 Copyright 2011-2012 - Dental Decks

TM

PDF Editor

Long-acting The length of action can be related to the lipid solubility with the ultrashort being the most lipid soluble and the long acting having the least lipid solubility. They are metabolized in the liver. These drugs possess serious drug dependence potential. They do not possess significant analgesic properties. The length of hypnotic action of the four different classes after a single dose are as follows: Ultrashort-acting: 5 to 20 minutes Short-acting: I to 3 hours Intermediate-acting: 3 to 6 hours Long-acting: 6 to 10 hours Agents: Ultrashort-acting: for induction of general anesthesia. Examples include thiopental (Pentothal) and methohexital (Brevital). Short-acting: for treating insomnia. Examples include secobarbital (Seconal) and pentobarbital (Nembutal). Intermediate-acting: for treating insomnia. Examples include amobarbital (Amytal) and butabarbital (Butisol). Long-acting: for treating certain types of seizures. Examples include phenobarbital (Luminal), mephobarbital (Mebaral) and primidone (Mysoline). Remember: Barbiturates do not possess analgesic properties. Important concept for anticonvulsant effect of long-acting barbiturates (i.e. phenobarbital): Barbiturates work by inhibiting the depolarization of neurons by binding to the GABA receptors, which enhances the transmission of chloride ions. They also increase the threshold for electrical stimulation of the motor cortex. Remember: Benzodiazepines work similarly to barbiturates, but they also increase the number of chloride channels while facilitating the transmission of chloride ions. This suppresses the spread of seizure activity but does not abolish abnormal discharge from a focus. Note: The cause of death from acute barbiturate poisoning or overdosage is respiratory failure. Other adverse reactions include CNS depression, euphoria, and habituation. The most important therapeutic measure to be taken in a case of barbiturate poisoning is to assure adequate respiration.

Rapid rate of redistribution from the brain to peripheral tissues Remember: These agents will maintain anesthesia only while in the brain. Because of their high lipid solubility, these agents will rapidly leave the brain for other tissues; thus the patient wakes up within a few minutes of administration. Examples of ultra-short-acting barbiturates include thiopental (Pentothal) and methohexital (Brevital). These agents are administered by intravenous injection. Contraindications to the use of ultra-short-acting barbiturates for general anesthesia: Porphyria Liver dysfunction (they are metabolized in the liver) Emphysema Previous addiction to sedative hypnotic drugs

Remember: These drugs have the ability to produce dependence.

TM

PDF Editor

(PHARMACOLOGY

Anti-Anxiety Agents)

All of the following statements concerning barbiturates are true EXCEPT one. Which one is the EXCEPTION?

' Barbiturates may increase the half-lives of drugs metabolized by the liver
1

Barbiturates depress neuronal activity by increasing membrane ion conductance (primarily chloride), reducing glutamate-induced depolarizations and potentiating the inhibitory effects of GAB A Compared with benzodiazepines, the barbiturates exhibit a steeper dose-response relationship

Barbiturates may precipitate acute porphyria in susceptible patients

50 Copyright 2011-2012 - Dental Decks

MPHARMACOLOGY

Anti-EpilepticsJ

All the following are classified as antiepileptics EXCEPT one. Which one is the EXCEPTION?

Gabapentin (Neurontin) Carbamazepine (Tegretol) Phenytoin (Dilantin) Propafenone (Rythmol) Phenobarbital (Luminal) Diazepam (Valium) Valproic acid (Depakene) Ethosuximide (Zarontin) Primidone (Mysoline)
51 Copyright 0 2011-2012 - Dental Decks

TM

PDF Editor

Barbiturates may increase the half-lives of drugs metabolized by the liver *** This is false; barbiturates may decrease the half-lives of drugs metabolized by the liver. Barbiturates induce the formation of the liver microsomal enzymes that metabolize drugs. This leads to an increased clearance of the affected drugs and possibly leads to a decrease in the drugs effectiveness. The uses of the barbiturates are determined by their duration of action: 1. The ultrashort-acting agents are used intravenously for the induction of general anesthesia. For extensive procedures, they are used to induce stage III surgical anesthesia. For very brief procedures, they may be used alone. 2. The short-acting agents can be used orally for their hypnotic, calming effect. These agents can be given preoperatively, before a dental appointment, to allay anxiety. 3. The intermediate-acting agents can also be prescribed to relieve anxiety before a dental appointment, although their effects will last longer than those of the short-acting agents. These agents are used for daytime sedation and for the treatment of insomnia (they suppress REM sleep). 4. The long-acting barbiturates are used primarily for daytime sedation and the treatment of epilepsy Generally the primary pharmacological effects of the barbiturates involve the brain and spinal cord (CNS depression of). At sedative doses barbiturates do not effect or have little effect on the cardiovascular and respiratory systems. Barbiturates are metabolized in the liver. The chronic use of the barbiturates causes an increase in liver microsomal enzyme activity that appears to be the result of increased synthesis of enzyme. These drugs possess serious drug dependence potential. They do not possess significant analgesic properties.

Propafenone (Rythmol)
*** Propafenone (Rythmol) is an antiarrhythmic agent used to treat both ventricular arrhythmias and supraventricular tachycardias. A seizure is an alteration in behavior, function, and/or consciousness that results from an abnormal electrical discharge of neurons in the brain. Epilepsy, or the term seizure disorder, is used to describe chronic unprovoked recurrent seizures. Seizures as classified as either partial or generalized, based on how the abnormal brain activity begins. Partial seizures: when seizures appear to result from abnormal activity in just one part of the brain. Generalized seizures: seizures that seem to involve all of the brain. Four types of generalized seizures exist: absence (also called petit mal). myoclonic, atonic and tonic-clonic seizures (also called grand mat). For most antiepileptic (anticonvulsant) drugs, the exact mechanism for reducing seizure activity is not clearly understood. All increase the threshold of the CNS to convulsive stimuli or inhibit the spread of seizure activity. The antiepileptic drugs (AEDs) can be grouped into the following classes: 1. Hydantoins: Phenytoin (Dilantin) is used in the treatment of tonic-clonic (grand mal) seizures. It prolongs the effective refractory period by blocking neuronal sodium channels. Phcnytoin-induccd gingival hyperplasia is common and may partially or totally obscure the crowns of teeth. The rate of development of this condition can be diminished by proper oral hygiene. 2. GABA analogs: The exact mechanism of action of valproic acid (Depakene) and the newer GABA analogs is not clearly understood. All of them increase the actions of GABA. an inhibitory neurotransmitter. They may inhibit the voltage-dependent sodium channel, (hereby stabilizing the neuronal membranes. Examples of the newer GABA analogs include: gabapentin (Neurontin). divalproex sodium (Depakote), fclbamatc (Felbatol), lamotriginc (Lamictal), prcgabalin (Lyrica), tiagabinc (Gabitril). and topiramate (Topamax). 3. Succinimides: Ethosuximide (Zarontin): used in the treatment of absence seizures. 4. Barbiturates: Phenobarbital (Luminal) works by inhibiting depolarization of neurons by binding lo the GABA receptors, which enhances the transmission of chloride ions. Primidone (Mysoline) is metabolized to phenobarbital and phcnylcthylmalonamidc (PEMA). which have anticonvulsant activity. 5. Benzodiazepines: work similarly lo barbiturates, but they also increase the number of chloride channels while facilitating the transmission of chloride ions. Examples include diazepam (Valium), clonazepam (Klonopin), and lorazepam (Ativan). Note: Diazepam (Valium) is used for status cpilcpticas and in emergency treatment of seizures. May cause drowsiness, dizziness, and ataxia. 6. Miscellaneous: oxcarbazepinc (Trileptal), levctiracetam (Keppra), and carbamazepine (Tregretol).Carbamazepine is used as prophylaxis for partial seizures with complex symptomatology including psychomotor and temporal lobe seizures. It is also used to treat tonicclonic seizures (grand mat) and trigeminal neuralgia. It rarely causes aplastic anemia.

TM

PDF Editor

(PHARMACOLOGY

Anti-Depressants)

The most widely used tricyclic antidepressant (TCA) drug is:

' Imipramine (Tofranil) ' Amitriptyline (Elavil) Desipramine (Norpramin) < Nortriptyline (Pamelor) Doxepin (Sinequan)

52 Copyright 2011-2012 - Dental Decks

^PHARMACOLOGY

Anti-Depressants)

Which drug is the current drug of choice for the treatment of the manic phase of bipolar disorder (or manic-depressive syndrome)?

Phenobarbital
1 Imipramine

Lithium Haloperidol

53 Copyright 2011-2012 - Dental Decks

TM

PDF Editor

Amitriptvline (Elavil) The tricyclic antidepressants are generally considered to be the drugs of first choice for the treatment of unipolar disorder (depression). These drugs inhibit the neuronal re-uptake of norepinephrine and serotonin (5-HT) in the brain. This results in the potentiation of their neurotransmitter actions at postsynaptic receptors. Important: Amitriptyline displays the greatest anticholinergics effects (especially xerostomia), desipramine the least. Drowsiness is the most frequent CNS adverse reaction. Anticholinergic adverse side effects include: dry mouth, constipation, blurred vision and tachycardia. The selective serotonin reuptake inhibitors (SSRIs) have revolutionized the treatment of depression. The most important clinical distinction of the SSRIs from all other antidepressants is their very high specificity for blocking the reuptake of serotonin. Fluoxetine (Prozac) is the prototype and has the longest half-life. Paroxetine (Paxil), Sertraline (Zoloft) and Fluvoxamine (Luvox) have shorter half-lives and can be given once daily. Citalopram (Celexa) and escitalopram (Lexapro) are used for generalized anxiety disorders. These drugs are also effective for treating panic attacks. Side effects include nausea, headaches, anxiety, agitation, insomnia, and sexual dysfunction.

Lithium Bipolar disorder (or manic-depressive syndrome) is characterized by cyclical changes in affective state between the manic and depressive phases of behavior. Bipolar patients cycle between the two affected states. Lithium salts are by far the most important drugs for suppressing mania in patients with affective disorders. It is not useful for the acute manic episodes. Lithium can prevent the occurrence of both the depressive as well as the manic episodes in some but not all the patients. Antidepressants are often administered with lithium to manage the depressive phase of the illness if lithium alone is not sufficient. Note: Lithium works inside the cell to prevent the formation of inositol triphosphate and diacylglycerol. These substances serve as second messengers within the CNS and may have widespread influence on neuronal function. Approximately 25% of patients who suffer from mania do not respond to lithium. Carbamazepine (an anticonvulsant) and valproic acid (also an anticonvulsant) may be effective in some refractory cases. Note: Carbamazepine blocks sodium channels and valproic acid blocks both sodium and calcium channels. Neuroleptic agents (also referred to as anti-psychotic agents or major tranquilizers) are used in the acute manic episodes. Chlorpromazine, which is a phenothiazine, and Haloperidol, which is not a phenothiazine but acts in a similar fashion, are effective in quelling the extreme mania and psychotic behavior. 1. The most common side effects of lithium, including GI irritation, fine hand tremor, muscular weakness, polyuria, thirst, sleepiness, and a sluggish feeling, are often associated with initial therapy and usually fade within 1 to 2 weeks. 2. Severe intoxication results in vomiting, diarrhea, unconsciousness, and convulsions. 3. Sodium restriction leads to greater retention of lithium in the kidney. 4. Diuretics and some NSAIDs reduce lithium excretion and may cause lithium toxicity.

TM

PDF Editor

QPHARMACOLOGY

Anti-Depressants

i)

Monoamine Oxidase (MAO) inhibitors are often used as third-line agents in cases of refractory and atypical depression. Which Two drugs below are MAO inhibitors?

Doxepin (Sinequan) Tranylcypromine (Parnate) Imipramine (Tofranil) Phenelzine (Nardil)

54 Copyright 2011-2012 - Dental Decks

QPHARMACOLOGY

Anti-Depressants

The vasoconstrictor epinephrine in local anesthetic injections must be used cautiously in patients taking all of the following antidepressant drugs EXCEPT one in order to avoid transient and significant increases in blood pressure. Which one is the EXCEPTION?

Tricyclic antidepressants (i.e., Elavil) Selective serotonin reuptake inhibitors (i.e., Prozac and Paxil) Serotonin and norepinephrine reuptake inhibitors (i.e., Effexor and Cymbalta)

55 Copyright 2011-2012 - Dental Decks

TM

PDF Editor

Tranylcypromine (Parnate) Phenelzine (Nardil) MAO inhibitors are used are often used as third-line agents in cases of refractory and atypical depression. MAO inhibitors antagonize the action of monoamine oxidase (MAO) responsible for the degradation of the naturally occurring monoamines epinephrine, norepinephrine, dopamine, and serotonin. It is theorized that the increased level of monoamines in the brain is responsible for the antidepressant effect of the MAO inhibitors. Note: Local anesthetics containing epinephrine are generally contraindicated in patients who are taking MAO inhibitors. The major limitation for the widespread use of the MAO inhibitors for the treatment of depression has been the potential for serious adverse side effects, including hypertensive crisis, which can be fatal. A hypertensive crisis can occur within several hours after ingestion of a substance that contains tyramine. Tyramine releases norepinephrine and other sympathomimetic amines, thereby raising blood pressure. Early symptoms include occipital headache, palpitations, stiff neck, nausea, vomiting, and sweating. Among the drugs that interact with MAO inhibitors are meperidine (Demerol), epinephrine and ephedrine. 1. Miscellaneous antidepressants include: Mirtazapine (Remeron): alpha2 -noradrenergic antagonnist Bupropion (Wellbutrin SL): norepinephrine / dopamine reuptake inhibitor Trazodone (Desyrel): mechanism not clear; possible serotonin reuptake inhibitor

Selective serotonin reuptake inhibitors (i.e., Prozac and Paxil) These two categories (tricyclic antidepressants and serotonin and norepinephrine reuptake inhibitors) of antidepressant drugs significantly increase norepinephrine levels in tissues. In the presence of a vasoconstrictor administered via a local anesthetic injection, the patient could experience significant elevation of blood pressure due to the vasopressor actions of the combination. Note: The selective serotonin reuptake inhibitors (i.e., Prozac and Paxil) have no such effect on norepinephrine in tissues and interaction with a vasoconstrictor like epinephrine is not an issue.

TM

PDF Editor

(PHARMACOLOGY

Anti-Depressants)

Which TWO groups of antidepressant drugs has the highest incidence of dry mouth (xerostomia)?

Tricyclic antidepressants (i.e., Elavil) Selective serotonin reuptake inhibitors (i.e., Prozac and Paxil) Serotonin and norepinephrine reuptake inhibitors (i.e., Effexor and Cymbalta) Monoamine oxidase inhibitors (i.e., Nardil)

56 Copyright 2011-2012 - Dental Decks

(PHARMACOLOGY

Anti-Depressants)

All of the following anti-depressant drugs are serotonin and norepinephrine reuptake inhibitors EXCEPT one. Which one is the EXCEPTION?

Venlafaxine (Effexor) Nortriptyline (Pamelor) Desipramine (Norpramin) Desvenlafaxine (Pristiq) Sertraline (Zoloft) Duloxetine (Cymbalta)

57 Copyright 2011-2012 - Dental Decks

TM

PDF Editor

Tricyclic antidepressants (i.e., Elavil) Serotonin and norepinephrine reuptake inhibitors (i.e., Effexor and Cymbalta) These two categories of antidepressant drugs induce significant dry mouth in up to 75% of patients taking these medications. These effects are due to the secondary anticholinergic nature of these agents. Note: Drug-induced dry mouth must be treated palliatively with artificial salivary substitutes. The selective serotonin reuptake inhibitors (i.e., Prozac, Paxil) and monoamine oxidase inhibitors have no secondary anticholinergic effects and therefore do not cause any significant dry mouth.

Sertraline (Zoloft) Five major categories of antidepressant drugs: Tricyclic antidepressants: Amitriptyline (Elavil) Doxepin (Sinequan) Imipramine (Tofranil) Serotonin and norepinephrine reuptake inhibitors: Venlafaxine (Effexor) Desvenlafaxine (Pristiq) Nortriptyline (Pamelor) Duloxetine (Cymbalta) Desipramine (Norpramin) Selective serotonin reuptake inhibitors: Citalopram (Celexa) Escitalopram (Lexapro) Fluoxetine (Prozac) Bupropion (Wellbutrin) Trazodone (Desyrel) Nefazodone (Serzone) Mirtazipine (Remeron) Paroxetine (Paxil) Sertraline (Zoloft)

Second generation miscellaneous:

Monoamine oxidase inhibitors: Phenelzine (Nardil) Tranylcypromine (Parnate) Isocarboxazide (Marplan)

TM

PDF Editor

(PHARMACOLOGY

Anti-Histamines)

All of the following are Hj-receptor blockers EXCEPT one. Which one is the EXCEPTION?

Diphenhydramine HCL (Benadryl) Chlorpheniramine maleate (Chlor-Trimeton) Cimetidine (Tagamet) Loratadine (Claritin) Desloratadine (Clarinex)

58 Copyright 2011-2012 - Dental Decks

^PHARMACOLOGY

Anti-Histamines)

For which of the following conditions can Ranitidine be used?

Prostatitis GERD (heartburn) Toxic-shock syndrome Renal failure

59
Copyright 2011-2012 - Dental Decks

TM

PDF Editor

Cimetidine (Tagamet) *** Cimetidine is an antihistamine H2-receptor blocker. Antihistamines are antagonizing agents that compete for receptor sites with natural histamine, which is stored preformed in cytoplasmic granules of tissue mast cells and blood basophils. It is released in response to IgE-mediated (immediate) allergic reactions and plays an important role in hay fever, urticaria and angioneurotic edema. Note: Histamine also plays an important role in the control of acid secretion (HCL) in the stomach. There are two types of histamine receptors, H,-receptors, which play an extremely important role in allergic reactions and H2-receptors, which are important in gastric acid secretion. The antihistamines are divided into H,and H2-receptor blockers depending on the histamine receptor they compete for. H| receptor blockers include: 1. First-generation (classical) agents: Diphenhydramine HCL (Benadryl) and chlorpheniramine maleate (Chlor-Trimetron) *** These agents have a broad spectrum of action which includes antihistaminic, anticholinergic, antiserotonergic, antibradykinin and sedative properties. 2. Second-generation agents: Cetirizine HCL (Zyrtec), fexofenadine HCL (Allegra), loratadine (Claritin) and desloratadine (Clarinex) *** Most of these agents, because of their poor CNS penetration (do not cross the bloodbrain barrier), cause less sedation and drowsiness than the first-generation agents. Important points: All of the H,-receptor blockers do not prevent the release of histamine but rather compete with free histamine for binding at H,-receptor sites. In general, the binding is competitive, however, some second-generation agents bind non-competitively at higher doses. Common side effects include drowsiness, dizziness, anticholinergic effects (dry mouth, nose and throat), and nausea. They can both stimulate and depress the CNS.

GERD (heartburn) H2-receptor antagonists compete with histamine at the Hi-receptor. Histamine produces a wide variety of physiologic actions in many tissues. While H2-receptors are located in the GI tract and in vascular and bronchial smooth muscle, H2-receptor antagonists compete with histamine only in the GI tract. Inhibition of histamine at the parietal cell interferes with one of several mediators for signaling the parietal cell to secrete acid. Hi-blockers include: cimetidine (Tagamet), ranitidine (Zantac), famotidine (Pepcid) and nizatidine (Axid). These agents are all reversible competitive antagonists of the actions of histamine on H2-receptors. These agents are used to treat acid-peptic disease, especially duodenal ulcers and occasionally gastric peptic ulcers. These drugs are also used to treat Zollinger-Ellison syndrome (a hypersecretory disease) and gastroesophageal reflux disease (GERD). Note: Cimetidine has an antiandrogen effect (can lead to impotence, loss of libido, and gynomastia). It also inhibits liver metabolism which can lead to an increase in activity of other drugs such as warfarin and carbamazepine.

TM

PDF Editor

(PHARMACOLOGY

Anti-HistaminesJ

Diphenhydramine HCL (Benadryl), chlorpheniramine maleate (Chlor-Trimeton), loratadine (Claritin), and desloratadine (Clarinex) are antihistamines at which histamine receptor site listed below?

H preceptor site H2-receptor site

60 Copyright 2011-2012 - Dental Decks

(PHARMACOLOGY

Anti-Infectives)

Antibiotic regimens for patients with prosthetic implants include all of the following drugs as standard therapy of first choice in patients not allergic to penicillin EXCEPT one. Which one is the EXCEPTION?

Cephalexin Cephradine .Amoxicillin Tetracycline

61 Copyright C 2011-2012 - Dental Decks

TM

PDF Editor

H preceptor site Antihistamines are antagonizing agents that compete for receptor sites with natural histamine. The two types of histamine antagonists are: 1. Hj-receptor antagonists: competitively block H |-receptors blocking the effects of histamine at these receptors. They block the vasodilation, they block the constriction of the bronchi and they block capillary permeability which histamine ordinarily causes. The blockade of these effects of histamine overcomes the symptoms of seasonal allergies. Actions < l II, antihistamines: - Blocks pain and itch, vasodilation and bronchoconstriction due to histamine - Reduces motion sickness: first-generation only (i.e.. Diphenhydramine HCL and chlorpheniramine maleate) - Promotes sleep: first-generation only Remember: The second generation U]-receptor antagonists have the following characteristics: Longer half-lives than first generation (12-24 hours as opposed to 3-6 hours for first generation), they do not cross the blood-brain barrier, they produce little or no sedation, and have a higher risk of cardiac arrhythmias (long QT effect). Examples include Cetirizine HCL (Zyrtec), fexofenadine HCL (Allegra), loratadine (Claritin) and desloratadine (Clarinex). 2. H2-receptor antagonists: competitively block H2-receptors, blocking the effects of histamine at these receptors. Examples of these drugs include: cimetidine (Tagamet), ranitidine (Zantac), famotidine (Pepcid) and nizatidine (Axid). They block secretion of stomach acid and are used in the treatment of duodenal ulcers. Note: Histamine is found in all tissues, particularly in mast cells and blood basophils. It is released in allergic and inflammatory reactions.

Tetracycline From the "Advisory Statement, Antibiotic Prophylaxis for Dental Patients with Total Joint Replacement" published by the American Dental Association and the American Academy of Orthopedic Surgeons, in the J Amer Dent Assoc 1997; 128(7): 1004-8 and J Amer Dent Assoc 2003;134:895-898 either cephalexin, or cephradine or amoxicillin can be used as the drugs of choice for standard prophylaxis medication in the patient with a total joint replacement. Cephalexin (Keflex) is a 1st generation cephalosporin; prophylaxis dose is 2 grams orally 1 hour prior to the dental procedure. Cephradine (Velosef) is a 1st generation cephalosporin; prophylaxis dose is 2 grams orally 1 hour prior to the dental procedure. Amoxicillin is a member of the penicillin family; prophylaxis dose is 2 grams orally 1 hour prior to the dental procedure. Note: For patients not allergic to penicillin and unable to take oral medications the suggested medications are cefazolin (a 1st generation cephalosporin) or ampicillin (member of the penicillin family). The regimen is as follows: Cefazolin (Ancef) 1 g or ampicillin 2 g intramuscularly or intravenously 1 hour prior to the dental procedure

TM

PDF Editor

(PHARMACOLOGY

Anti-Infectives)

Clindamycin has which mode of action on the bacteria cell?

Affects cell membrane Interferes with protein synthesis Affects cell wall Interferes with normal biosynthetic pathways

62
Copyright 2011-2012 - Dental Decks

(PHARMACOLOGY

Anti-Infectives)

Mefloquine (Lariam) belongs to a class of drugs which is useful in treating which of the following conditions?

Malaria AIDS Hepatitis Cancer Chlamydia

63 Copyright 2011 -2012 - Dental Decks

TM

PDF Editor

Interferes with protein synthesis Mechanisms of action of antibiotics on the bacteria cell: Agents affecting cell - Penicillins - Cephalosporins - Bacitracin wall: - Vancomycin - Cycloserine - Aztreonam - Imipenem

Agents interfering with protein synthesis: - Tetracycline - Aminoglycosides - Chloramphenicol - Erythromycin - Lincomycin - Clindamycin - Azithromycin - Clarithromycin

Agents interfering with normal biosynthetic pathways: - Sulfonamides - Trimethoprim - Fluoroquinolones (i.e., ciprofloxacin, norfloxacin, levofloxacin, moxifloxacin, and gemifloxacin) Note: The most common clinical cause of bacterial resistance is the use of antibiotics when they are not indicated.

Malaria Malaria is a devastating parasitic disease transmitted through the bite of infected Anopheles mosquitoes. Endemic to tropical and subtropical areas of Asia, North and South America, the Middle East, North Africa, and the South Pacific. Plasmodium vivax is the most common of four human malaria species (P. falciparum, malariae, ovale, and vivax). P. vivax causes up to 65% of malaria in India and is becoming increasingly resistant to malaria drugs. By contrast, P. falciparum is the most deadly species and the subject of most malaria-related research and literature. Mefloquine (Lariam) has been shown to be effective in treating malaria caused by all four malaria species. Other antimalarial drugs are: Chloroquin (Aralen) Combination of atovaquone and proguanil (Malarone) Halofantrine Quinine Combination of sulfadoxine and pyrimethamine (Fansidar)

TM

PDF Editor

(PHARMACOLOGY

Anti-Infectives)

Which of the following antibiotics is considered a broad-spectrum antibiotic?

Penicillin VK
1 Cefaclor (Ceclor)

Penicillin G Erythromycin

64 Copyright 2011-2012 - Dental Decks

(PHARMACOLOGY

Anti-Infectives)

Tetracyclines are the drugs of first choice in the treatment of all of the following EXCEPT one. Which one is the EXCEPTION?

Mycoplasma pneumonia Chlamydia infections Rickettsial infections Staphylococcal infections

65 Copyright 2011-2012 - Dental Decks

TM

PDF Editor

Cefaclor (Ceclor) Cefaclor is a member of the cephalosporin family of antibiotics. The cephalosporins are penicillin-like in action against bacteria. They are bactericidal antibiotics and act like the penicillins and interfere with cell wall synthesis through inhibition of the synthesis of the peptidoglycan in the cell wall. The antibiotic binds to the enzymes (i.e., transpeptidase) that build/maintain the cell wall. This makes the cell wall osmotically unstable. Bacteria eventually lyse, resulting in death of the cell. Cephalosporins act against a wide range of gram-positive and gram-negative bacteria. Currently there are four generations of cephalosporins. Each newer generation has significantly greater gram-negative antimicrobial properties than the preceding one, and a decreased maximum activity against gram-positive organisms. Important cephalosporins within each generation: First: cephalexin (Keflex), cephradine (Velosef), cefadroxil (Duricef), cefazolin (Ancef). Most active against S. aureus, Group A Beta-hemolytic streptococci, and Pneumococcus. Second: cefaclor (Ceclor), cefuroxime (Ceftin), cefoxitin (Mefoxin), cefprozil (Cefzil), cefpodoxime (Vantin). Still have efficacy against gram-postivie organisms but also possess good activity against E. coli and H. influenzae. Third: cefixime (Suprax), cefoperazone (Cefobid), ceftriaxone (Rocephin). Have the broadest spectrum of activity of all cephalosporins and are extremely effective against gramnegative organisms. They are more active against E. coli, Klebsiella pneumoniae, Enterobacter. Salmonella, and Shigella. Fourth: cefepime (Maxipime). Effective against Pseudomas aeruginosa. Note: Approximately 10% of individuals expressing allergy to the penicillin family of antibiotics will have cross allergenicity to the cephalosporins. Remember: Antibiotics containing this beta-lactam ring are referred to collectively as betalactam antibiotics and include the penicillins, cephalosporins, and the two newer groups of agents, the carbapenems and the monobactams.

Staphylococcal infections Popular Tetracyclines include: Tetracycline: used to treat acne, gonorrhea and syphilis in patients allergic to penicillin, exacerbations of chronic bronchitis, Mycoplasma infections and Chlamydia infections, and Rickettsia infections. Minocycline (Minocin): used to treat acne, anthrax and meningococcal prophylaxis. Doxycycline (Vibramycin): used to treat infections caused by Rickettsia, Chlamydia and Mycoplasma; alternative to mefloquine for malaria prophylaxis and treatment of svphilis.

TM

PDF Editor

(PHARMACOLOGY

Anti-Infectives)

Azithromycin (Z-Pak, Zithromax) is an antibiotic of the:

Macrolide class of antibiotics Cephalosporin class of antibiotics Quinolone class of antibiotics Glycopeptide class of antibiotics

66 Copyright 2011-2012 - Dental Decks

(PHARMACOLOGY

Anti-Infectives)

Erythromycin is well known to cause adverse:

CNS effects GI effects Hematologic effects Renal effects

67 Copyright 2011-2012 - Dental Decks

TM

PDF Editor

Macrolide class of antibiotics Azithromycin (Z-Pak, Zithromax) and clarithromycin (Biaxin) are members of the macrolide class of antibiotics in which erythromycin is the prototype agent. The bacterial spectrums of activity of azithromycin and clarithromycin are similar to that of erythromycin but possess greater intrinsic activity against H. influenzae and Helicobacter pylori. These two macrolides concentrate within macrophages, making them useful against organisms that are taken up by macrophages such as Mycobacterium avium intracellulare. The significant tissue penetration of both agents and the prolonged elimination half-life of azithromycin (11-14 hours) allows for once-daily dosing for azithromycin and twicedaily dosing for clarithromycin. Note: Macrolide antibiotics are generally used to treat infections caused by streptococcal bacteria and respiratory infections generally. They are also active against syphilis, Lyme disease and leprosy and tuberculosis. The macrolides can have a toxic effect on the liver, causing liver toxicity and jaundice and so should not be given to people who have an underlying liver problem such as an infection with hepatitis C. Macrolides are bacteriostatic. The mechanism of action involves inhibition of protein synthesis that results from binding specifically to the 50s ribosomal subunit. This causes the RNA to dissociate from the ribosome and prevents protein synthesis. Remember: Intrinsic activity is a measure of the ability of a drug once bound to the receptor to generate an effect activating stimulus and producing a change in cellular activity.

GI effects Adverse GI effects are reported for approximately 21% of patients receiving erythromycin, about 10% of patients receiving clarithromycin, and less than 5% for azithromycin. In general, oral bioavailability of erythromycin is poor. It is readily inactivated by stomach acid, and several salts have been developed to overcome this drawback. Erythromycin stearate (Eiythrocin) Erythromycin ethylsuccinate (E.E.S.) *** Since they are destroyed by stomach acid, erythromycins are usually enteric coated. This is a term designating a special coating applied to tablets or capsules which prevents their release and absorption of their contents until they reach the intestines. Note: Erythromycin is metabolized in the liver, excretion is mainly via the bile.

TM

PDF Editor

(PHARMACOLOGY

Anti-Infectives)

The most commonly used aminoglycoside is:

Neomycin Streptomycin Tobramycin Gentamicin

68 Copyright 2011-2012 - Dental Decks

(PHARMACOLOGY

Anti-Infectives)

Sulfonamides (also known as sulfa drugs) are structurally similar to:

Penicillins Erythromycins Acetylsalicyclic acid (Aspirin) Para-aminobenzoic acid (PABA)

Copyright 2011-2012 - Dental Decks

TM

PDF Editor

Gentamicin

Aminoglycosides are potent bactericidal antibiotics that act by creating fissures in the outer membrane of the bacterial cell. They are particularly active against aerobic, gram-negative bacteria and act synergistically against certain gram-positive organisms. Gentamicin is the most commonly used aminoglycoside, but amikacin may be particularly effective against resistant organisms. Aminoglycosides are used in the treatment of severe infections of the abdomen and urinary tract, as well as bacteremia and endocarditis. They are also used for prophylaxis, especially against endocarditis. Resistance is rare but increasing in frequency. Avoiding prolonged use, volume depletion and concomitant administration of other potentially nephrotoxic agents decreases the risk of toxicity. Single daily dosing of aminoglycosides is possible because of their rapid concentration-dependent killing and post-antibiotic effect and has the potential for decreased toxicity.
Aminoglycosides:

Gentamicin (Garamycin), Amikacin (Amiken), and Tobramycin (Nebcin). These are effective against serious infections caused by aerobic gram-negative bacteria, including E. Coli, Enterobacter, Klebsiella, Proteus, Pseuodomonas aeruginosa, and Serratia. Streptomycin: The first aminoglycoside and was shown to be effective in the treatment of tuberculosis. Is seldom used today. Neomycin (Mycifradin) and Kanamycin (Kantrex): Due to its toxic potential neomycin is used only topically or locally (e.g., in the GI tract). Kanamycin is rarely used because of its marked tendency to cause ototoxicity. 1. Aminoglycosides may cause severe neuromuscular weakness lasting hours to days because of their potential curare-like effect. Aminoglycosides may aggravate muscle weakness in patients with muscular disorders such as myasthenia gravis, infant botulism, or parkinsonism. 2. Aminoglycosides bind irreversibly to the 30s ribosomal subunit of bacteria, subsequently inhibiting protein synthesis. 3. Two well-known adverse effects are ototoxicity and nephrotoxicity.

Para-aminobenzoic acid (PABA)

Sulfonamides are often referred to as "sulfa drugs" because their molecules contain sulfur atoms. They have a different antibacterial mechanism from that of the antibiotics. The sulfonamides are structurally similar to PABA and this similarity is the basis for their antibacterial actions. PABA is needed by bacteria for the synthesis of folic acid. In turn, folic acid is needed for the synthesis of cellular components within the bacteria to allow for cell growth. Because of structural similarities between sulfonamides and PABA, the sulfonamides compete with PABA and are able to inhibit the actions of PABA. With PABA inhibited, folic acid is not synthesized within the bacteria, and bacterial cellular growth is inhibited. Sulfonamides are predominantly bacteriostatic agents. Examples of sulfonamides: Sulfacetamide, Sulfadiazine, Sulfadoxine, Sulfamefhizole, Sulfamethoxazole, Sulfanilamide, Sulfasalazine, and Sulfisoxazole. 1. Sulfonamides are not used for treatment of dental infections because of a low degree of effectiveness against oral pathogens. 2. Sulfonamides are used in medicine primarily for the treatment of urinary tract infections. 3. Bactrim is the brand name for the combination of trimethoprim and sulfamethoxazole. It is considered the drug of choice for many urinary tract infections. Bacteria susceptible to Bactrim include E. coli, Pneumocystis carinii, Klebsiella pneumoniae, Haemophilus influenzae and Salmonella species. 4. The trimethoprim component is an antimicrobial and the sulfamethoxazole is one of the sulfonamides. 5. Hypersensitivity reactions are common. Although blood dyscrasias are relatively rare, they can be fatal.

TM

PDF Editor

(PHARMACOLOGY

Anti-Infectives)

Which antibiotic is not only effective against most staphylococci, aerobic and anaerobic streptococci, but is most effective in treating infections due to bacteroides species?

Penicillin VK Erythromycin Tetracycline Cephalexin (Keflex) Clindamycin (Cleocin) Vancomycin


70 Copyright 2011-2012 - Dental Decks

(PHARMACOLOGY

Anti-Infectives)

Which antibiotic may cause bone marrow disturbances (aplastic anemia) and has limited use due to the side effects?

Penicillin Tetracycline Chloramphenicol Doxycycline

71 Copyright 2011-2012 - Dental Decks

TM

PDF Editor

Clindamycin (Cleocin) Clindamycin binds to the 50s ribosomal subunit, blocking bacterial protein synthesis. Its use is restricted by its side effects such as severe diarrhea and pseudomembranous colitis. These side effects are caused by the overgrowth of the bacterium known as Clostridium difficile. Clindamycin is bacteriostatic and is active against most gram-positive (i.e.. Streptococcus pneumoniae, viridans, and pyogenes as well as Staphylococcus aureus) and many anaerobic organisms, including the anaerobic gram-negative bacteria Bacteroides fragilis.

In dentistry, clindamycin is an alternate antibiotic in the following situations: When amoxicillin cannot be used for the standard regimen for prevention of bacterial endocarditis in patients undergoing dental procedures For treatment of common oral-facial infections caused by aerobic gram-positive cocci and susceptible anaerobes For prophylaxis for dental patients with total joint replacement Important: Clindamycin can be given to patients allergic to penicillins since there is no cross allergenicity between penicillins and clindamycin. Note: Vancomycin is given IV and is used most often in serious or life-threatening staphylococcal or streptoccocal infections. It remains the drug of choice for severe cases of Clostridium difficile. Adverse effects include ototoxicity which may be permanent and the "red man" syndrome which is characterized by a sudden and profound fall in blood pressure with or without a maculopapular rash over the face, neck, upper chest, and extremities.

Chloramphenicol Chloramphenicol is a broad-spectrum antibiotic effective against gram-positive and gram-negative bacteria and against anaerobes. It is used as a second or third line drug in medicine to treat serious infections due to organisms resistant to other less toxic antibiotics. For example it can be used to treat the following: Typhoid Fever, Bacterial Meningitis. Anaerobic Infections, Rickettsial Diseases, and Brucellosis. Chloramphenicol inhibits protein synthesis in bacteria and, to a lesser extent, in eukaryotic cells. The drug readily penetrates bacterial cells, probably by facilitated diffusion. Chloramphenicol acts primarily by binding reversibly to the 50s ribosomal subunit. The most important adverse effect of chloramphenicol is on the bone marrow. Chloramphenicol affects the hematopoietic system in two ways: by an non-dose-related idiosyncratic response manifested by aplastic anemia, leading in many cases to fatal pancytopenia by a dose-related toxic effect that presents as anemia, leukopenia, or thrombocytopenia Important: The risk of aplastic anemia does not contraindicate the use of chloramphenicol in situations in which it is necessary; however, it emphasizes that the drug should never be employed in undefined situations or in diseases readily, safely, and effectively treatable with other antimicrobial agents. Note: Fatal chloramphenicol toxicity may develop in neonates, especially premature babies, when they are exposed to excessive doses of the drug. The illness, the gray baby syndrome, usually begins 2 to 9 days after treatment is started.

TM

PDF Editor

(PHARMACOLOGY

Anti-Infectives)

Nitazoxanide (Alinia) is an oral antiprotozoal agent used to treat which of the following conditions?

1 Leprosy 1 Malaria

AIDS
1

Diarrhea caused by Clostridium difficile

Diarrhea caused by Giardia lamblia

72 Copyright 2011-2012 - Dental Decks

(PHARMACOLOGY

Anti-Infectives)

All of the following drugs are useful for treating what disease?

Isoniazid Streptomycin Rifampin Ethambutol Pyrazinamide

73 Copyright 2011-2012 - Dental Decks

TM

PDF Editor

Diarrhea caused by Giardia lamblia *** This type of diarrhea is an intestinal infection also known as Giardiasis, and is the most common protozoan infection in the United States. Antiprotozoal Agents: Nitazoxanide (Alinia): is an antiprotozoal agent which is used in the treatment of Giardia, and those protozoal infections caused by Cryptosporidium parvum. Its mechanism is to interfere with the electron transfer reaction within the protozoa essential to anaerobic metabolism. Atovaquone (Mepron): is an antiprotozoal agent used to treat Pneumocystitis carinii pneumonia (PCP) in patients who are intolerant to co-trimoxazole. Eflornithine (Vaniqa): has orphan drug status for the treatment of meningoencephalitic stage of Trypanosoma brucei gambiense infection (sleeping sickness). This drug is also indicated to be used as a cream to reduce unwanted hair from face and adjacent areas under the chin. Furazolidone (Furoxone): is a antiprotozoal agent used to treat diarrhea caused by susceptible Giardias lamblia and Vibrio cholerae. Metronidazole (Flagyl): is a synthetic antibacterial and antiprotozoal agent that is effective against Trichomonas vaginalis, which causes trichomoniasis. In addition, it is one of the most effective drugs available against anaerobic bacterial infections. These include infections caused by Helicobacter pylori, Clostridium difficile, Bacteroides species and Fusobacterium species. Note: Alcohol should be avoided because metronidazole and alcohol together can cause severe nausea, vomiting, cramps, flushing, and headache. It should is not used in pregnancy because of potential adverse effects on the fetus.

Tuberculosis Tuberculosis is a bacterial infection caused by Mycobacterium tuberculosis. The antitubercular drugs either inhibit the growth of the bacteria or kill the bacteria. Because the Mycobacterium organism tends to develop resistance to any single antitubercular drug, combination drug therapy is standard in the treatment of tuberculosis. Antitubercular drugs: Isoniazid: often given in a four drug regimen along with rifampin, pyrazinamide and ethambutol Streptomycin: often given in combination with isoniazid Rifampin: usually given in combination with other agents Ethambutol: usually given in combination with other agents Pyrazinamide: popular in combination with rifampin Mechanism of action: Isoniazid: is bacteriostatic for resting organisms and bactericidal for dividing organisms. It interferes with lipid and nucleic acid biosynthesis in growing organisms. Streptomycin: is a bactericidal antibiotic. It acts by interfering with normal protein synthesis. Rifampin: inhibits DNA-dependent RNA-polymerase activity, thereby supressing RNA synthesis. It can be bacteriostatic or bactericidal and is most active against bacteria undergoing cell division. Ethambutol: impairs cellular metabolism, causing cessation of cell multiplication and cell death. It is bactericidal and is active only against Mycobacterium. Pyrazinamide: the mechanism of action is unknown. It may be bacteriostatic or bactericidal against M. tuberculosis, depending on the concentration.

TM

PDF Editor

(PHARMACOLOGY

Anti-Infectives)

The antiviral agent penciclovir (Denavir) is active against which virus?

Herpes zoster Genital herpes Herpes simplex type 1 (HSV-1) Papilloma virus

74 Copyright 2011-2012 - Dental Decks

(PHARMACOLOGY

Anti-Infectives)

The following drugs are used to treat what viral disease?

Didanosine Zidovudine Ritonavir Indinavir Delavirdine

75 Copyright 2011 -2012 - Dental Decks

TM

PDF Editor

Herpes simplex type 1 (HSV-l) Penciclovir (Denavir) is a cream formulation indicated for the treatment of recurrent herpes labialis (cold sores) in adults. This condition is caused by the herpes simplex type 1 virus. Penciclovir is not available for systemic dosing. Penciclovir (Denavir) inhibits viral action by selectively inhibiting herpes viral DNA synthesis and therefore resulting in the inhibition of viral replication. Other agents indicated for use in treating the condition of herpes labialis are: Acyclovir tablets Acyclovir cream Docosanol cream (Abreva) Lysine tablets Valacyclovir caplets (Valtrex)

AIDS (acquired immunodeficiency syndrome) AIDS has been recognized since 1981 as a unique clinical syndrome brought on by infection with the human immunodeficiency virus 1 (HIV-1) or virus 2 (HIV-2). The major cellular defect caused by infection with HIV is a depletion of T cells, primarily the subtype T-helper cells known as CD4 cells. This results in a compromised immune system, which becomes susceptible to opportunistic infections. HIV is a type of retrovirus that is responsible for the fatal illness from AIDS. A retrovirus has RNA as its nucleic acid and uses the enzyme reverse transcriptase to copy its genome into the DNA of the host's cells chromosomes. This DNA segment is then permanently incorporated into the host cell's DNA within the nucleus. The integrated DNA segment can produce new RNA in the cytoplasm of the host cell. The new RNA in turn synthesizes viral proteins, which are eventually passed on to other host cells such as the immune system macrophages. Ultimately enough of the human immune cells are compromised such that immune function is lost. The HIV drugs such as didanosine (brand name Videx), zidovudine (brand name Retrovir), ritonavir (brand name Nor\>ir), and indinavir (brand name Crixivan) work by inhibiting certain steps in the HIV infection process within the target cells to halt the destruction of the immune system by the HIV retrovirus.

TM

PDF Editor

(PHARMACOLOGY

Anti-Infectives)

Which of the following is classified as an antifungal agent?

Bacitracin Amphotericin-B Polymyxin-B Neomycin

76 Copyright 2011-2012 - Dental Decks

^PHARMACOLOGY

Anti-Infectives)

The drug of choice for treating Candidiasis is:

Penicillin Erythromycin Nystatin Chloramphenicol

77 Copyright 2011-2012 - Denial Decks

TM

PDF Editor

Amphotericin-B This is an antifungal agent given intravenously or orally for the treatment of severe systemic fungal infections caused by fungi such as Candida species. Bacitracin, polymyxinB and neomycin are not antifungal agents. These are antibiotics effective against susceptible bacteria. Antibiotics in general do not have antifungal properties.
Summary of Some Antifungal Agents Topical Agents (dissolve and swallow) Clotrimazole (Mycelex Troche) Nystatin (Mycostatin) Topical Agents (cream/ointment) Amphotericin-B (Fungizone) Ketoconazole (Nizoral) Nystatin (Mycostatin) Systemic Agents Amphotericin-B (Fungizone) Ketoconazole (Nizoral) Fluconazole (Diflucan) Use Oropharyngeal Candidiasis Oral Cavity Candidiasis Use 1 Cutaneous and Mucocu J taneous candidiasis Use Systemic Candida Oral Esophageal Candidiasis Oralpharyngeal Candidiasis Mechanism on Fungal Cell Alters cell membrane Alters cell membrane Mechanism on Fungal Cell Alters cell membrane Alters cell membrane Alters cell membrane Mechanism on Fungal Cell Alters cell membrane Alters cell membrane Alters cell membrane Form Troche Oral Suspension Form Cream Cream Ointment Form IV injection Tablets Tablets

Note: Systemic amphotericin-B is associated with high incidence of kidney toxicity

Nystatin Candidiasis is an infection, usually of the oral cavity or vagina, with a Candida species, usually C. albicans, which causes an inflammatory, pruritic infection characterized by a thick, white discharge. It is common, especially in patients who have a deficiency in T-lymphocytes, or who are receiving chemotherapy, and in immunosupressed individuals (AIDSpatients). This yeast-like fungi is a normal inhabitant of the oral cavity and vaginal tract, however it is normally held in check b> the indigenous bacteria of these areas. Note: Angular cheilitis (bilateral ulcers at the corner of the mouth) has been linked to C. albicans. Remember: Nystatin and clotrimazole are the two antifungals that are used as "swish and swallow" to treat oral Candida infections. Nystatin (Mycostatin) is taken as an oral suspension to be swished around the mouth and swallowed. Clotrimazole (Mycelex) is taken as a troche (lozenge) which is slowly dissolved in the mouth and swallowed. They work by binding to sterols in the fungal cell membrane, increasing permeability and permitting the leakage of intracellular components. This leads to the death of the affected fungal cell. Note: The suffixes of the following generic drug names are indicative of the correspsonding drug classes: "azole" = azole-type antifungal drugs (e.g., clotrimazole) "coxib" = COX-2 inhibitors (e.g., celecoxib) "dipine" = dihydropyridine calcium channel blockers (e.g., nicardipine) "olol" = beta-adrenergic receptor blockers (e.g., nadolol) "ilol" or "alol" = beta-adrenergic receptor blocker that also blocks alpha,-adrenergic receptors (e.g., carvedilol or labetalol) "onium" or "urium" = quaternary ammonium compounds, usually competitive peripheral acting skeletal muscle relaxes (e.g., pancuronium) "osin" = alpha|-adrenergic receptor blockers (e.g., terazosin) "pril" = ACE inhibitors (e.g., enalapril) "sartan" = angiotensin II receptor blockers (e.g., olmesartan) "statin" = HMG-CoA reductase inhibitor (e.g., atorvastatin)

TM

PDF Editor

(PHARMACOLOGY

Anti-Infectives)

Which of the following is known as the original penicillin?

Penicillin V Penicillin G Ampicillin Amoxicillin

78 Copyright 2011-2012 - Dental Decks

(PHARMACOLOGY

Anti-Infectives)

Which penicillin is degraded in stomach acid and is usually given parenterally (by injection)?

Penicillin VK
1 Amoxicillin

Penicillin G Ampicillin

79 Copyright 2011-2012 - Dental Decks

TM

PDF Editor

Penicillin G

All penicillins are derivatives of 6-aminopenicillanic acid and contain a beta-Iactam ring structure joined to a thiazolidine ring. The beta-lactam ring is essential for its antibacterial activity. This basic structure is synthesized by the penicillium mold from two amino acids, L-cysteine and L-valine. Antibiotics containing this beta-lactam ring are referred to collectively as beta-lactam antibiotics and include the penicillins, cephalosporins, and the two newer groups of agents, the carbapenems and the monobactams. Penicillin G (benzylpenicillin) is the prototype for comparison. By side chain substitutions (specifically, this means substituting other groups at the R position of the penicillin molecule) of the basic 6-aminopenicillanic acid molecule, the semi-synthetic penicillins are produced which are more acid stable, have a broader spectrum, or are penicillinase resistant. Other naturally occurring penicillins include: Penicillin VK (Pen Vee K, V-cillin K) - preferred for treating oral infections because it is more acid stable (more reliable oral absorption) Penicillin G parental (Pfizerpen) - always given by IM route Penicillin G benzathine (Bicillin C-R) - always given by IM route. It is used for the treatment of syphilis and prevention of rheumatic fever Remember: 1. Penicillins are bactericidal; they inhibit cell wall synthesis. 2. Probenecid increases blood levels of natural penicillins and may be given concurrently for this purpose.

Penicillin G

Acid stable penicillins (may be used orally) include: Penicillin VK Amoxicillin Ampicillin Nafcillin Oxacillin Cloxacillin Dicloxacillin Extended spectrum penicillins include: The aminopenicillins Ampicillin and Amoxicillin Broad spectrum penicillins include: Piperacillin Ticarcillin *** These two penicillins have the widest spectrum of all the penicillins Penicillinase-resistant penicillins include: Nafcillin Oxacillin Cloxacillin Dicloxacillin

TM

PDF Editor

(PHARMACOLOGY

Anti-Infectives)

The antibiotic of choice for standard prophylactic regimen of antibiotic coverage for the prevention of bacterial endocarditis is:

Penicillin VK Tetracycline Erythromycin Amoxicillin

80 Copyright 2011-2012 - Dental Decks

(PHARMACOLOGY

Anti-Infectives)

Of the drugs listed which is the most preferable antibiotic for the treatment of non-penicillinase-producing gram-positive staphylococcal infections?

Tetracycline Clindamycin Ampicillin Cefaclor (Celcor) Penicillin VK

81 Copyright 2011-2012 - Denial Decks

TM

PDF Editor

Amoxicillin Premedication requirements for patients with valvular heart disease or congenital cardiac defects. If in doubt, have patient consult their physician as to need. Current American Heart Association Guidelines: Published lune 2007, JADA Volume 138, page 142. Standard Regimen Amoxicillin - Adults: 2.0 g orally 30-60 minutes prior to appointment - Children: 50 mg/kg (not to exceed adult dose) orally 30-60 minutes prior to appointment Allergy to Amoxicillin: Note: Only if allergy is not of anaphylactic type. Any one of the following can be used. Clindamycin -Adults: 600 mg orally 30-60 minutes prior to appointment -Children: 20 mg/kg orally 30-60 minutes prior to appointment Cephalexin -Adults: 2.0 g orally 30-60 minutes prior to appointment -Children: 50 mg/kg orally 30-60 minutes prior to appointment Azithromycin -Adults: 500 mg orally 30-60 minutes prior to appointment -Children: 15 mg/kg orally 30-60 minutes prior to appointment Clarithromycin -Adults: 500 mg orally 30-60 minutes prior to appointment -Children: 15 mg/kg 30-60 minutes prior to appointment

Penicillin VK It is prudent to use an antibiotic with narrow spectrum of action and one that is bactericidal in order to minimize the development of bacterial resistance. Penicillin VK has these properties. Ampicillin has a broader spectrum of action than penicillin VK and cefaclor is a broad spectrum cephalosporin. Tetracycline and clindamycin are bacteriostatic antibiotics and not bacteriocidal. Note: The major disadvantage of the penicillins is their rather high incidence of allergic reactions. Approximately 10% of the general population is allergic to penicillins. This incidence probably holds for any of the specific penicillins since there is cross allergy from one to the other. Skin rash (a delayed reaction) is the most prevalent allergic manifestation. Life threatening anaphylaxis can occur, but is very rare, particularly with oral dosing. In non-allergic individuals, penicillins at normal therapeutic doses have virtually no side effects what so ever. Penicillins are bactericidal, that is, they actually cause death of the invading bacteria.

TM

PDF Editor

(PHARMACOLOGY

Anti-Infectives)

Which penicillin has the widest spectrum of antibacterial activity?

Penicillin VK Dicloxacillin Amoxicillin Piperacillin

82 Copyright 2011-2012 - Dental Decks

(PHARMACOLOGY

Anti-Infectives)

Which of the following has a clinically significant drug interaction with Amoxicillin?

Triazolam (Halcion) Methotrexate Calcitrol Candesartan

83 Copyright 2011-2012 - Dental Decks

TM

PDF Editor

Piperacillin Members of the penicillin family having the broadest spectrums of action are piperacillin and ticarcillin. In addition to being active against gram-positive cocci (streptococci, staphylococci, pneumococci), and gram-positive rods (bacillus and others), these agents are active against Pseudomonas, Proteus, Klebsiella, and Bacteroides. These antibiotics are recommended for the treatment of urinary tract infections and other infections caused by susceptible gram-negative Pseudomonas species and Proteus species. Penicillin VK has a relative limited spectrum of action against aerobic gram-positive cocci and anaerobes. Dicloxacillin has a similar spectrum as penicillin VK but is active against penicillinaseproducing staphylococcus. Amoxicillin is an aminopenicillin which has an extended spectrum of action which includes not only aerobic gram-positive cocci and anaerobes, but some gram-negative bacilli such as Hemophilus, Proteus and Salmonella. Ampicillin also falls into the category of an aminopenicillin. Important: None of the penicillins are active against viruses, fungi, rickettsiae or other nonbacterial organisms.

Methotrexate *** Amoxicillin in large doses inhibits the renal tubular secretion of methotrexate, thereby causing higher, prolonged serum levels of methotrexate. Aminopenicillins (ampicillin and amoxicillin) are characterized by the amino substitution of penicillin G. They are able to penetrate gram-negative bacteria more readily than are the natural penicillins or the penicillinase-resistant penicillins. However, the aminopenicillins are not stable to beta-lactamases (penicillinase) of either gram-positive or gram-negative bacteria. Note: Gram-negative bacteria that are susceptible to aminopenicillins include: H.influenzae, some E.Coli and Proteus mirabilis. Ampicillin (Polycillin, Onipen) and Amoxicillin (Amoxil. Larotid) are used primarily to treat infections such as otitis media, bronchitis, sinusitis, and acute bacterial cystitis caused by susceptible organisms. Compared with ampicillin, amoxicillin has a higher oral absorption, higher serum levels, a longer half-life, and is less likely to cause adverse GI effects (diarrhea). Amoxicillin is given orally; ampicillin can be given orally or IV. Note: Ampicillin and amoxicillin are preferred agents in the treatment of urinary tract infections caused by susceptible enterococci (which are facultatively anaerobic, grampositive cocci that grow in short chains under extreme conditions mainly in the intestine).

TM

PDF Editor

(PHARMACOLOGY

Anti-Infectives)

What do the following penicillin antibiotics have in common?

Amoxicillin and clavulanate potassium (Augmentin) Ampicillin and sulbactam (Unasyn) Cloxacillin Dicloxacillin

84 Copyright 2011-2012 - Denial Decks

(PHARMACOLOGY

Anti-Infectives)

Which of the following bacterial enzymes belong to the family of beta-lactamases?

Cephalosporinase Penicillinase ATPase Protein kinase

85 Copyright 2011-2012 - Dental Decks

TM

PDF Editor

They are all penicillinase resistant penicillins A functional part of the chemical molecule of all the penicillins is the so-called betalactam ring, which is a four-membered imbedded ring structure consisting of three carbons and one nitrogen atom which is responsible for the antibacterial activity of penicillins. Any alteration to the beta-lactam ring will also alter the antibacterial activity. Penicillinase is an enzyme secreted by bacteria which splits open the beta-lactam ring. This renders the penicillin molecule ineffective against those penicillinase secretors. Cloxacillin and dicloxacillin resist the actions of penicillinase because they have a protected beta-lactam ring which prevents the actions of the enzyme. Augmentin and Unasyn contain the agents clavulanate potassium and sulbactam respectively which block the actions of penicillinase from reaching the beta-lactam ring. The majority of penicillins are directly excreted into the urine through renal tubular cell secretion. Probenecid (Benemid), an inhibitor of renal tubular cell secretion raises the blood levels of the penicillins by diminishing their tubular secretion. Probenecid is sometimes given simultaneously with penicillin to raise the blood levels for increased activity. Note: Probenecid is a drug used to treat gout.

Ccphalosporinase Penicillinase Beta-lactamases are enzymes produced and secreted by a wide range of gram-positive and gram-negative bacteria as a defense weapon against cephalosporin and penicillin antibiotics. These enzymes destroy the beta-lactam nucleus within these antibiotics by splitting open the beta-lactam ring structure. This action renders the antibiotic ineffective. Those beta-lactamases that work against cephalosporins are called cephalosporinases, and those that work against penicillins are called penicillinases. By combining clavulanic acid with a penicillin, the beta-lactamase enzyme is permanently inhibited by the acid, and the antibacterial activity of the penicillin is maintained. One popular commercial preparation is Augmentin, which contains amoxicillin and clavulanate potassium. Augmentin is used orally as pill or liquid form. Sulbactam is another beta-lactamase inhibitor. It is available for intravenous and intramuscular use combined with ampicillin under the brand name Unasyn.

TM

PDF Editor

(PHARMACOLOGY

Anti-Infectives)

Which of the following is often co-administered with antibiotics to delay the renal clearance of the antibiotic?

Imipenem Probenecid Hydrochlorothiazide Aztreonam

86 Copyright 2011-2012 - Dental Decks

(PHARMACOLOGY

Anti-Infectives)

Which antibiotic is used cautiously due to its side effects (pseudomembranous colitis, severe GI upset)?

Azithromycin (Zithromax; Z-Pak) Clindamycin Penicillin VK Cephalexin (Keflex)

87 Copyright 2011-2012 - Dental Decks

TM

PDF Editor

Probenecid *** This elevates and prolongs the serum concentrations of the antibiotic when high tissue concentrations are necessary. The majority of penicillins are handled by the kidneys as organic acids and excreted by tubular excretion. Probenecid interferes with tubular handling of organic acids within the nephron. Drugs affected by probenecid include most cephalosporins and natural penicillins, and other beta-lactam-related antibiotics such as aztreonam and imipenem. Note: In some cases, probenecid administration can more than double the serum concentration of the affected drug. The half-life is prolonged as well. Important: Nafcillin, oxacillin, cloxacillin and dicloxacillin are lipophilic and are excreted by biliary means. No combination with probenecid or dosage adjustment for renal dysfunction is necessary for these penicillins. 1. Imipenem is a beta-lactam antibiotic derived from thienamycin and is the first drug to be classified as a carbapenem antibiotic. It is currently the drug of choice for infections due to Enterobacter and Pseudomonas aeruginosa. It is usually combined with cilastatin and is used to treat severe or resistant infections, especially those that are nosocomial in origin. 2. Aztreonam is a parenteral synthetic beta-lactam antibiotic (classified as a monobactum). The spectrum is limited to aerobic gram-negative rods (i.e., Klebsiella, Pseudomonas, andSerratia). It has no gram-positive or anaerobic activity. It is synergistic with aminoglycosides.

Clindamycin
Summary of Important Antibiotics Drug Penicillins: Penicillin VK Amoxicillin Amoxicillin Clav. Acid (Augmentin) Ampicillin Cephalosporins: Cephalexin (Keflex) Cefaclor (Ceclor) Other: Clindamycin Macrolides: Azithromycin (Z-Pak) Clarithromycin (Biaxin) Erythromycin Tetracyclines: Tetracycline Doxyxycline (Vihramycin) Minocycline (Minocin; Arestin) Aminoglycosides: Streptomycin Gentamycin Miscellaneous: Ciprofloxacin (Cipro) Bacitracin Chloramphenicol Antibacterial Action Antibacterial Mechanism Bactericidal Same Same Same Important Adverse Effects

Inhibits cell wall synthesis Same Same Same

Hypersensitivity Same Same Same Hypersensitivity Same Pseudomembranous colitis None significant GI hyperactivity Same Superinfection None significant Same Ototoxicity, Nephrotoxicity Same Nausea, Headache Nephrotoxicity Blood disorders

Inhibits cell wall synthesis Bactericidal Same Same Inhibits 50S ribosomes Inhibits SOS ribosomes Same Same Inhibits 30S ribosomes Same Same Inhibits 30S ribosomes Same Bacteriostatic Bacteriostatic Same Same Bacteriostatic Same Same Bactericidal Same

Inhibits DNA Bactericidal Inhibits cell wall synthesis Same Inhibits 50S ribosomes Bacteriostatic

TM

PDF Editor

(PHARMACOLOGY

Anti-Infectives)

Which antibiotic/antimicrobial is associated with the highest incidence of drug allergy?

Ciprofloxacin (Cipro) Penicillin VK Clindamycin Metronidazole (Flagyl)

88 Copyright 2011 -2012 - Dental Decks

(PHARMACOLOGY

Anti-Infectives)

Stavudine (also known as d4T, or Zerit) is an antiretroviral drug used in the treatment of adults with HIV infection in combination with other antiretroviral agents. Which of the following categories does this agent belong to?

Nucleoside reverse transcriptase inhibitor Protease inhibitor Non-nucleoside reverse transcriptase inhibitor

89 Copyright 2011 -2012 - Dental Decks

TM

PDF Editor

Penicillin VK Three groups of allergic reactions to the penicillins: 1. Acute (anaphylactic shock): occurs within 30 minutes. Characterized by urticaria, angioedema, bronchoconstriction, GI disturbances, and shock. Death can result in a short time if treatment is not instituted immediately (parenteral administration of epinephrine). 2. Accelerated: occurs 30 to 48 hours after. Manifestations include urticaria, pruritis, wheezing, mild laryngeal edema, and local inflammatory reactions. Not life threatening. 3. Delayed: occurs after 2 to 3 days. Approximately 80-90% of all allergic reactions occurring with penicillin are of this type. Manifested by skin rashes. Hypersensitivity reactions occur in up to 10% of patients receiving penicillin. Manifestations range from a mild rash to anaphylaxis. The rash may be urticarial, vesicular, bullous, or maculopapular. Rarely, thrombopenic purpura develops. Note: A rash is the most common sign of an allergy to penicillin. Remember: The mechanism of action of the fluoroquinolones is inhibition of DNAgyrase, an enzyme that is essential in the transcription, replication, and repair of bacterial DNA. Drugs in this class include ciprofloxacin (Cipro), norfloxacin (Noroxin), levofloxacin (Levaquin), moxifloxacin (Avelox), and gemifloxacin (Factive). Fluoroquinolones are active against many gram-positive organisms and gram-negative aerobes, including Moraxella catarrhalis, Haemophilus influenzae, E. coli, chlamydia, and Mycoplasma pneumoniae. They are not active against Clostridium difficile. These drugs are well tolerated, however, the most common adverse effects are GI (e.g., nausea, vomiting, dianhea, abdominal pain), CNS (e.g., headache, dizziness, confusion) and dermatologic (e.g., rash, pruritis).

Nucleoside reverse transcriptase inhibitor These agents chemically are nucleosides and work by inhibiting the viral enzyme known as reverse transcriptase. This results in an inhibition of the HIV viral RNA from being made into a DNA segment; thus the genome of the HIV virus is not copied from RNA. Other agents in this class include didanosine (Videx), zalcitabine (Hivid; ddC) and zidovudine (Retrovir; AZT). Protease inhibitors suppress viral replication by inhibiting protease, the enzyme responsible for cleaving viral precursor peptides into infective virions. Some agents in this class include indinavir (Crixivan), nelfinavir (Viracept), ritonavir (Norvir), and saquinivir (Invirase). Nonnucleoside reverse transcriptase inhibitors inhibit the catalytic reaction of reverse transcriptase that is independent of nucleotide binding. Some agents in this class include delavirdine (Rescriptor), adefovir (Hepsera) and nevirapine (Viramune).

TM

PDF Editor

(PHARMACOLOGY

Anti-Infectives)

All of the following are common adverse effects caused by the use of tetracyclines EXCEPT one. Which one is the EXCEPTION?

Photosensitivity Nausea Bone marrow disturbances Diarrhea Discoloration of teeth and enamel hypoplasia in young children

90 Copyright 2011-2012 - Dental Decks

(PHARMACOLOGY

Anti-Infectives)

Which TWO penicillins are referred to as aminopenicillins?

Penicillin VK Ampicillin (Polycillin) Cloxacillin (Cloxapen) Amoxicillin (Amoxil)

91 Copyright 20! 1-2012 - Dental Decks

TM

PDF Editor

Bone marrow disturbances Another common adverse effect is the occurrence of opportunistic (superinfection) infections caused by Candida albicans. This is due to the wide spectrum of antibacterial action which alters normal flora. Examples include both vaginal and oral candidiasis. These conditions are also known as vaginal and oral yeast infections. Members of the tetracycline family of antibiotics include: Short acting: Intermediate acting: Long acting: Tetracycline (Generic) Demeclocycline (Declomycin) Minocycline (Minocin) Doxycycline (Vibramycin)

Remember: The usefulness of the tetracyclines in the treatment of odontogenic infections is limited since they can cause "yeast" infections very easily. They have been used as alternatives to penicillin in patients with ANUG (acute necrotizing ulcerative gingivitis) who require antibiotics. Note: In adults, Fanconi syndrome can be caused by various things that damage the kidneys, including certain medications (azathioprine, cidofovir, gentamicin, and tetracycline). Fanconi syndrome is a disorder of the kidney tubules in which certain substances normally absorbed into the bloodstream by the kidneys are released into the urine instead.

Ampicillin (Polycillin) Amoxicillin (Amoxil) These two antibiotics are characterized by the amino substitution of penicillin G. They work against many gram-positive organisms and some gram-negative bacteria such as Haemophilus influenzae, some Escherichia coli, and Proteus mirabilis. They are not penicillinase resistant. They are used for upper respiratory infections. The major difference in the drugs is the higher oral absorption, higher serum levels, and longer half-life for amoxicillin compared with ampicillin. Amoxicillin is given orally; ampicillin can be given orally and IV Remember: Oral amoxicillin is recommended as the drug of choice for standard general prophylaxis for bacterial endocarditis in patients undergoing invasive dental procedures. Note: Parenteral ampicillin is recommended as the drug of choice in patients unable to take oral medications and who are not allergic to penicillin for prophylaxis for bacterial

TM

PDF Editor

MPHARMACOLOGY

Anti-Infectives)

Cephalosporins have which of the following modes of action?

Affect bacterial cell wall Affect bacterial DNA Affect bacterial protein synthesis Interfere with bacterial metabolic pathways

92 Copyright 2011-2012 - Dental Decks

(PHARMACOLOGY

Anti-Infectives)

Which of the following is the mode of action of the Tetracyclines?

Bacterial cell wall destruction Prevent protein synthesis in the bacterial cell Interfere with nucleic acid synthesis Cause mutations within bacterial DNA

93 Copyright 2011-2012 - Dental Decks

TM

PDF Editor

Affect bacterial cell wall Basic mechanisms of actions of antibiotics: Affect bacterial cell wall: - Penicillin family - Cephalosporin family Agents affecting bacterial DNA: - Fluoroquinolone family (i.e., ciprofloxacin) - Metronidazole (Flagyl) Agents affecting bacterial protein synthesis: - Tetracycline family - Erythromycin family - Clindamycin - Chloramphenicol Agents interfering with bacterial metabolic pathways: - Sulfonamides (sulfa drugs)

Prevent protein synthesis in the bacterial cell The tetracyclines inhibit protein synthesis by binding to the 30 S subunit of the bacterial ribosome. The inhibition of this ribosomal function interferes with the attachment of the growing amino acid chain thus preventing complete formation of peptides from the ribosome. Since no peptides are formed, no proteins are formed. Since proteins are necessary for the bacterial cell to metabolically function, the lack thereof will cause a static state in which the bacterium becomes vulnerable to phagocytosis by the body's immune system. Absorption of the tetracyclines from the GI tract is inhibited by divalent and trivalent cations such as CA", Mg", Fe", and Al++\ These ions form chelation products with the tetracyclines and thus prevent their absorption. This is why tetracyclines should not be given with milk and dairy products (contain Ca**), iron-containing vitamins (contain Fe**), mineral supplements containing these irons, or antacids (contain Mg**).

TM

PDF Editor

(PHARMACOLOGY

Anti-Infectives)

Which antibiotic exerts its antibacterial activity through inhibition of protein synthesis in the bacterial cell?

Cephalosporins Ampicillin Tetracycline Penicillin G Penicillin VK

94
Copyright 2011 -2012 - Dental Decks

(PHARMACOLOGY

Anti-Infectives)

Erythromycin is a antibiotic which binds to the ribosomal subunit of susceptible bacteria. The result is the inhibition of protein synthesis.

Bactericidal; 30s Bactericidal; 50s Bacteriostatic; 30s Bacteriostatic; 50s

95 Copyright 2011 -2012 - Dental Decks

TM

PDF Editor

Tetracycline The tetracyclines are a group of broad-spectrum, bacteriostatic antibiotics that inhibit protein synthesis in the susceptible organism by binding to the 30S ribosome subunit, thereby impeding the binding of aminoacyl tRNA to the receptor site on the messenger RNA ribosome complex. The inhibition of this ribosomal function interferes with the attachment of the growing amino acid chain thus preventing complete formation of peptides from the ribosome. Since no peptides are formed, no proteins are formed. Since proteins are necessary for the bacterial cell to metabolically function, the lack thereof will cause a static state in which the bacterium becomes vulnerable to phagocytosis by the body's immune system. Tetracyclines are useful in treating the following infections: Medical infections caused by susceptible gram-positive and gram-negative bacteria Infections caused by Mycoplasma, Chlamydia , Protozoa or Rickettsia Exacerbations of chronic bronchitis Lyme disease (caused by Borrelia burgdorferi) Treatment of acne Treatment of gonorrhea and syphilis in patients allergic to penicillin Dental: treatment of periodontitis associated with the presence of Actinobacillius actinomycetemcomitans (AA) (i.e., localized aggressive periodontitis) Important: Tetracyclines are not the drug of choice for Streptococcus or Staphylococcus.

Bacteriostatic; 50s Erythromycin-type antibiotics are members of the Macrolide family of antibiotics. Members of the Macrolide family (generic and brand names) are: Azithromycin (Zithromax; Z-Pak) Clarithromycin (Biaxin) Erythromycin base (E-mycin; Eiyc) Erythromycin ethylsuccinate (E.E.S.) Erythromycin stearate (Erythrocin)

All of the erythromycins are very effective against gram-positive bacteria but not so effective against gram-negative bacteria. All of the erythromycins act through the same mechanism to bind to the 50s ribosomal subunit of susceptible bacteria. This causes the RNA to dissociate from the ribosome and prevents protein synthesis. Note: GI Tract upset is the most common side effect of the erythromycins (take with food).

TM

PDF Editor

MPHARMACOLOGY

Anti-Infectives)

Which antimicrobial below is associated with photosensitivity?

Tetracycline Penicillin VK Metronidazole (Flagyl) Clindamycin

96 Copyright 2011-2012 - Dental Decks

(PHARMACOLOGY

Anti-Infectives)

Which TWO antivirals are classified as neuraminidase inhibitors?

Acyclovir (Zovirax) Oseltamivir (Tamiflu) Amantadine (Symmetrel) Rimantadine (Flumadine) Zanamivir (Relenza)

97 Copyright 2011-2012 - Dental Decks

TM

PDF Editor

Tetracycline The photosensitivity reaction caused by the tetracycline family of antibiotics results in the appearance of red rashes or blotches over the skin in the presence of sunlight. Common adverse effects caused by the other agents: Penicillin VK: hypersensitivity resulting in skin rash and rare anaphylaxis Metronidazole (Flagyl): dizziness, headache, and nausea Note: Metronidazole is not a true antibiotic since it is not found in natural organisms; it is a synthetic substance produced in the chemical laboratory. Clindamycin: diarrhea, abdominal pain; known to cause pseudomembranous colitis

Oseltamivir (Tamiflu) Zanamivir (Relenza) Oseltamivir (Tamiflu) and zanamivir (Relenza) inhibit influenza virus neuraminidase enzymes, potentially altering virus particle aggregation and release. Tamiflu and Relenza are both used to treat acute illness due to influenza (A or B) infection. Acyclovir (Zovirax) is an antiviral that inhibits DNA synthesis rather than neuraminidase enzymes. Amantadine is a synthetic anti-viral drug that can inhibit the replication of viruses in cells. It was initially used to prevent influenza A during flu season, and, if given within 24 to 48 hours of the onset of flu symptoms, to decrease the severity of the flu. Later amantadine was found to cause improvement in the symptoms of Parkinson's disease. Note: The mechanism of amantadine's antiviral activity involves interference with a viral protein, M2, which is required for the viral particle to become "uncoated" once taken inside a cell by endocytosis. Rimantadine is a synthetic anti-viral drug that can prevent viruses in cells from multiplying. Like amantadine, rimantadine initially was used to prevent influenza A during flu season, and, if given within 24 to 48 hours after the onset of flu symptoms, to decrease the severity of the flu. Rimantadine is chemically related to amantadine, but rimantadine has fewer side effects on the nervous system than amantadine. Note: Rimantadine appears to exert its inhibitory effect early in the viral replicative cycle, possibly inhibiting the uncoating of the virus (similar to amantadine's mechanism of action).

TM

PDF Editor

(PHARMACOLOGY

Anti-Infectives)

Which drug can be used in the pregnant patient?

1 Tetracycline 1 Doxycycline (Vibramycin)

Minocycline (Minocin) Penicillin VK

98 Copyright E 2011-2012 - Dental Decks

(PHARMACOLOGY

Anti-Infectives)

A patient was given penicillin 15 minutes ago and develops the following signs and symptoms. What kind of reaction is this patient having?

Laryngeal edema Urticaria (welts that itch) Severe hypotension GI disturbances Bronchoconstriction (airway constriction) Shock

99 Copyright 2011-2012 - Dental Decks

TM

PDF Editor

Penicillin VK Penicillin VK is not harmful to the fetus when taken by pregnant mothers. It is not incorporated into bony tissue or in the teeth of children like the tetracyclines. Note: Tetracycline and all members of the tetracycline family are contraindicated in children up to 8 years old and in pregnant women. Tetracyclines have the ability to chelate calcium ions and become incorporated in the bony tissues. The teeth of children who have been given the drug may develop a greenish-brown discoloration. This effect is sometimes seen in the newly erupted teeth of infants whose mothers have received tetracycline during pregnancy. Remember: Penicillins, which are bactericidal against susceptible organisms, disrupt synthesis of the bacterial cell wall and compete for and bind to specific enzyme proteins that catalyze transpeptidation and cross-linking. The enzymes to which they bind are called penicillin-binding proteins (PBPs). They consist of transpeptidases, transglycosylases, and D-alanine caboxykinases and are implicated in the final phases of building and reshaping of the bacterial cell wall while it is growing and dividing.

Anaphylactic reaction (anaphylactic shock) *** This reaction most commonly occurs with parenteral administration of penicillin. This reaction can be fatal if countermeasures such as the injection of epinephrine are not taken promptly. Epinephrine prevents the release of substances from mast cells and antagonizes the actions of histamine and leukotrienes of smooth muscle. The most common adverse effect of penicillin therapy is an allergic reaction. These reactions occur in up to 10% of patients receiving penicillin. The most common manifestation is a mild rash. Allergic reactions to penicillin are classified into three groups: 1. Immediate - onset reactions (anaphylaxis) occur within 30 minutes; Ig-E mediated. 2. Accelerated - allergic reactions arise 30 minutes to 48 hours after administration. Urticaria, pruritis, wheezing, and local inflammatory reactions. In general, not lifethreatening. 3. Delayed - allergic reactions take longer than two days to develop. 80-90% of penicillin reactions are of this type. Basic skin rashes, which are generally mild.

TM

PDF Editor

(PHARMACOLOGY

Anti-PsychoticsJ

Antipsychotic drugs include all of the following EXCEPT one. Which one is the EXCEPTION?

1 The phenothiazines

The thioxanthenes
1 The butyrophenones 1 The benzodiazepines

100 Copyright 2011-2012 - Dental Decks

(PHARMACOLOGY

Anti-Psychotics J

All of the following are phenothiazines EXCEPT one. Which one is the EXCEPTION?

' Chlorpromazine (Thorazine) Haloperidol (Haldol) Prochlorperazine (Compazine) Thioridazine Perphenazine Trifluoperazine (Stelazine)

101 Copyright 2011 -2012 - Dental Decks

TM

PDF Editor

The benzodiazepines Antipsychotics, also known as major tranquilizers or neuroleptics, are used commonly in the treatment of a variety of psychotic disorders. By far the most widely used group of antipsychotic agents used in medicine are the phenothiazines. followed by the butyrophenones and the thioxanthenes. The first antipsychotic on the market, chlorpromazine, is the key drug among the phenothiazine antipsychotic agents. Currently, antipsychotics are divided into two generations. The first generation includes the older, "typical" drugs that treat the positive but not the negative symptoms associated with a psychotic state. Second-generation drugs have far fewer extrapyramidal symptoms (EPS) and tardive dyskinesia (TD). and they are used to treat both positive and negative symptoms of schizophrenia. Note: With the exception of risperidone, they are prolactin sparing. The exact mechanism of antipsychotic drug action is unknown. These drugs are thought to work by blocking postsynaptic dopamine receptors in the hypothalamus, basal ganglia, limbic system, brainstem, and medulla, and to some extent serotonin receptors. Typical antipsychotic drugs (first generation) are more potent antagonists of D 2 dopamine receptors than of D] receptors. First generation drugs include the phenothiazines (chlorpromazine, fluphenazine, perphenazine, prochlorperazine, trifluoperazine, mesoridazine, and thioridazine), the thioxanthines (thiothixene) and the butyrophenones (haloperidol). Newer (second generation) or atypical antipsychotic agents affect different receptor sites compared with first generation antipsychotics. They bind dopamine, including D|, D2, D4, and D5 receptors, with selectively for limbic dopamine receptors. They have increased affinity for serotonin (5-HT2) receptors compared with D2 receptors. They exhibit reduced ability or an inability to induce EPS. Second generation agents include clozapine, risperidone, olanzapine, quctiapine, ziprasidone, and aripiprazole. Important: 1. Aripiprazole (Ability) is the first of a new class of atypical antipsychotic agents called dopamine system stabilizers or dopamine partial agonists. It combines the actions of D2 and serotonin 5-HT2a receptor antagonism. 2. Clozapine is specific for limbic receptors and not for striated (muscle) receptors, which explains the low incidence of EPS and TD.

Haloperidol (Haldol) Antipsychotic agents include: First generation (typical) agents: 1. Phenothiazines: Chlorpromazine (Thorazine) Perphenazine Thioridazine Prochlorperazine (Compazine) Trifluoperazine (Stelazine) Mesoridazine Fluphenazine

2. Butyrophenones: Haloperidol is a highly effective antipsychotic drug used to treat schizophrenia. In addition it has been found to be effective in the treatment of Tourette's syndrome. 3. T h i o x a n t h e n e s : Thiothixene is a less potent antipsychotic. It is used for the treatment of schizophrenia. Second generation (atypical) agents: this group includes clozapine, risperidone, olanzapine, quetiapine, ziprasidone, and aripiprazole. These drugs are effective in treating schizophrenia and exhibit reduced ability or an inability to induce EPS. Adverse effects of antipsychotic drugs: Dystonia and akathisia (an unpleasant sensation of motor restlessness) Long QT-interval syndrome and ultimately increase the risk of fatal arrhythmias Neuroleptic malignant syndrome Parkinsonism Antimuscarinic effects Orthostatic hypotension Convulsions

TM

PDF Editor

(PHARMACOLOGY

Anti-Psychotics)

All of the following drugs are used to treat psychosis EXCEPT one. Which one is the EXCEPTION?

< Chlorpromazine
1 Thioridazine (Mellaril)

< Risperidone (Risperdol) < Azelastine (Astelin) ' Quetiapine (Seraquel) ' Clozapine (Clozaril) Olanzapine (Zyprexa) Haloperidol (Haldol)
102
Copyright 2011-2012 - Dental Decks

(PHARMACOLOGY

Anti-Psychotics)

Which statement describes the extrapyramidal syndrome (EPS) caused by the phenothiazine-type antipsychotics?

1 Orthostatic hypotension 1 Sedation

Headache Dry mouth


1

Muscle spasms of the oral-facial region

103 Copyright 6 2011-2012 - Dental Decks

TM

PDF Editor

Azelastine (Astelin) *** Azelastine (Astelin) is an antihistamine nasal spray. Antipsychotics are primarily indicated for treatment of psychosis associated with schizophrenia, paranoia, and manic symptoms of manic-depressive illness. Typical antipsychotic drugs (first generation) are more potent antagonists of D 2 dopamine receptors than of D| receptors. First generation drugs include the phenothiazines (chlorpromazine, fluphenazine, perphenazine, prochlorperazine, trifluoperazine, mesoridazine, and thioridazine), the thioxanthines (thiothixene) and the butyrophenones (haloperidol). Note: Haloperidol is a potent dopamine antagonist. Newer (secondgeneration) or atypical antipsychotic agents affect different receptor sites compared with first generation antipsychotics. They bind dopamine, including D|, D 2 , D 4 , and D 5 receptors, with selectively for limbic dopamine receptors. They have increased affinity for serotonin (5-HT2) receptors compared with D 2 receptors. They exhibit reduced ability or an inability to induce EPS. Second generation agents include clozapine, risperidone, olanzapine, quetiapine, ziprasidone, and aripiprazole.

Muscle spasms of the oral-facial region Extrapyramidal syndrome (EPS) refers to a variety of signs and symptoms that are a result of the blockade of dopamine receptors in specific brain regions. These symptoms include: Parkinson-like movements (shuffled gait, pill-rolling effect of fingers), muscle rigidity, spasms of neck and facial muscles, tremors, and loss of muscle movement. Tardive Dyskinesia (TD) is a serious, irreversible neurological disorder that can appear at any age. Tardive Dyskinesia is a side effect of taking antipsychotic/neuroleptics drugs. Symptoms involve uncontrollable movement of various body parts, including the body trunk, legs, arms, fingers, mouth, lips, or tongue. About 20 percent of people taking antipsychotic/neuroleptic drugs for more then one year will be affected. Remember: Newer (secondgeneration) or atypical antipsychotic agents affect different receptor sites compared with first generation antipsychotics. They bind dopamine, including D,, D 2 , D 4 , and D 5 receptors, with selectively for limbic dopamine receptors. They have increased affinity for serotonin (5-HTT) receptors compared with D2 receptors. They exhibit reduced ability or an inability to induce EPS. Second generation agents include clozapine, risperidone, olanzapine, quetiapine, ziprasidone, and aripiprazole. Important: Clozapine is specific for limbic receptors and not for striated (muscle) receptors, which explains the low incidence of EPS and TD.

TM

PDF Editor

(PHARMACOLOGY

Asp/Acet/NSAIDs)

All of the following are symptoms of acute aspirin toxicity EXCEPT one. Which one is the EXCEPTION?

Tinnitus (ringing in the ears) ' Vertigo (dizziness)


1 Nausea

Hallucinations
1 Respiratory alkalosis

Vomiting

104 Copyright 2011-20I2 - Dental Decks

(PHARMACOLOGY

Asp/Acet/NSAIDsJ

The main ingredient in Advil is also the main ingredient in which of the following prescription products?

Percodan Vicodin ES Motrin Naprosyn

105 Copyright 2011-2012 - Dental Decks

TM

PDF Editor

Hallucinations An overdosage of salicylates (acute aspirin toxicity) is life threatening and requires intensive supportive treatment in a hospital. Initial symptoms include respiratory alkalosis with hyperpnea and tachypnea, nausea, vomiting, hypokalemia, tinnitus, headache, dizziness, confusion, dehydration, hyperthermia, hyperactivity, and hematologic abnormalities, progressing to coma and respiratory collapse. Chronic aspirin toxicity presents with the following signs and symptoms: salicylism, CNS effects, bleeding and GI disutrbances. Aspirin inactivates the enzyme known as cyclooxygenase. Since cyclooxygenasc synthesizes the prostaglandins, the inhibition of this enzyme results in the inhibition of prostaglandin synthesis. As a result, aspirin is analgesic, antipyretic (fever reducing), and anti-inflammatory. Aspirin is an irreversible platelet inhibitor and can reduce blood clotting to prolong bleeding. Note: Aspirin inhibits both COX-1 andCOX-2. Low doses of aspirin taken regularly can have a cardioprotective effect. These doses reduce thromboxane production in platelets to result in the inhibition of platelet aggregation. In this way, aspirin has the ability to inhibit the formation of life-threatening thrombi (blood clots). The liver appears to be the principal site for salicylate metabolism, although other tissues may also be involved. The three chief metabolic products of salicylic acid are salicyluric acid, the ether or phenolic glucuronide and the ester or acyl glucuronide. Excretion of salicylates occurs principally via the kidney, through a combination of glomerular filtration and tubular excretion, in the form of free salicylic acid, salicyluric acid, as well as phenolic and acyl glucuronides. Contraindications to the use of aspirin: Bleeding disorders (aspirin will increase bleeding time) Do not use in children with viral infections (i.e., influenza or chickenpox) with or without fever due to a potential association with Reye's syndrome (this syndrome is a serious neurological defect) Pregnancy (especially during the third trimester) Peptic ulcers (aspirin may cause bleeding of the GI tract) Asthma; rhinitis; nasal polyps Concomitant use of anticoagulants

Advil contains 200 mg of ibuprofen and may be sold over-the-counter; whereas, Motrin contains 400 mg of ibuprofen or higher and can only be sold with a prescription. Nonsteroidal anti-inflammatory drugs (NSAIDs) have anti-inflammatory effects resulting from their ability to inactivate the enzyme prostaglandin endoperoxide synthase (cyclooxygenase). By doing this they inhibit the cyclooxygenase step of the arachidonic acid cascade and thus reduce local prostaglandin synthesis. NSAIDs also have analgesic, and antipyretic actions. Remember: The traditional NSAIDs such as ibuprofen, naproxen and aspirin inhibit COX-2 along with COX-1. Thus they are effective in reducing pain and inflammation, but are capable of inducing gastrointestinal ulcers. The COX-2 selective inhibitors will reduce pain and inflammation without any significant risk of causing gastrointestinal ulcers. For the traditional NSAIDs such as ibuprofen, naproxen, and aspirin, because they inhibit both COX-1 and COX-2 enzymes, they belong to the category of non-selective COX inhibitors. For celecoxib (Celebrex), because it inhibits COX-2 enzyme only, it belongs to the category of COX-2 selective inhibitors. Note: Rofecoxib (Vwxx) and Valdecoxib (Bextra) are also COX-2 selective inhibitors that were removed from the market because they were found to have added cardiovascular risks in some patients.

TM

PDF Editor

(PHARMACOLOGY

Asp/Acet/NSAIDs}

Which non-narcotic analgesics would you most likely use in a patient taking anti-coagulant medication?

1 Ibuprofen (Motrin, Advil) 1 Acetaminophen (Tylenol)

Aspirin Naproxen sodium (Aleve)

106 Copyright 2011-2012 - Dental Decks

MPHARMACOLOGY

Asp/Acet/NSAIDsJ

Which agent has anti-inflammatory properties?

Codeine Acetaminophen Hydrocodone Ibuprofen

107 Copyright 2011-2012 - Dental Decks

TM

PDF Editor

Acetaminophen (Tylenol) Acetaminophen has no effect on platelets nor the coagulation pathways and does not affect bleeding times or prothrombin times even with high doses. Acetaminophen has two major pharmacological actions: an analgesic effect and an antipyretic (fever reducer) effect. Acetaminophen is not effective enough to reduce severe pain, but it is effective in reducing mild to moderate pain. Acetaminophen is a weak inhibitor of prostaglandin formation. Large doses of acetaminophen can cause liver toxicity. Alcohol can seriously increase the hepatotoxic potential of acetaminophen. There are approximately 100 deaths annually due to liver toxicity produced by ingesting large continuous doses of acetaminophen. Acetaminophen very rarely causes drug sensitivities and can be given to patients having an allergy to aspirin. Differences Between Acetaminophen and Aspirin Acetaminophen Affect blood clotting Gastrointestinal ulcers No No Yes Yes Yes Yes Yes Aspirin

Associated with Reye's syndrome No Anti-inflammatory effects Analgesic effects No Yes

Ibuprofen Ibuprofen inhibits the production of prostaglandins in peripheral tissues at sites where pain and inflammation are present. Inhibition of prostaglandin production reduces the inflammatory response at sites of surgery, injury or infection. Reduction of inflammation results in reduction of perceived pain. Acetaminophen is a weak inhibitor of prostaglandin production in peripheral tissues. Thus, the inflammatory response is not affected to any great degree. Acetaminophen reduces pain through mechanisms other than inflammatory reduction. It is unclear exactly how acetaminophen works to reduce pain. Codeine and hydrocodone are narcotic analgesics that effectively reduce pain but do not reduce inflammation. Narcotics work within the brain to block ascending pain impulses traveling from the periphery into the brain.

TM

PDF Editor

(PHARMACOLOGY

AspMxet/NSAIDs)

Your patient has a history of drug abuse. Which agent could be given (if needed in the treatment plan) with no liability to cause an addiction?

' Phenobarbital
1

Ibuprofen

Hydrocodone Meperidine Codeine

108 Copyright 2011-2012 - Dental Decks

(PHARMACOLOGY

Asp/Acet/NSAIDsJ

NSAIDs reduce the production of associated with pain and inflammation.

Leukotrienes Cytokines Prostaglandins Interferons

109 Copyright C 2011-2012 - Dental Decks

TM

PDF Editor

Ibuprofen Ibuprofen is an NSAID and is classified as a non-narcotic analgesic. Non-narcotic analgesics have no liability for abuse or addiction. They are not a controlled substances. Phenobarbital is a barbiturate used as a sedative and to treat epilepsy. Barbiturates all have the potential to cause abuse and addiction, and are controlled substances requiring a DEA number from the prescriber. Hydrocodone (Vicodin), meperidine (Demerol) and codeine are narcotic analgesics with the potential to cause abuse and addiction. They are all controlled substances requiring a DEA number from the prescriber. Important: NSAIDs can inhibit the antihypertensive effect of ACE inhibitors, betablockers, and diuretics.

Prostaglandins Non-steroidal anti-inflammatory drugs (NSAIDs) inhibit the cyclooxygenase step of the arachidonic acid cascade and thus inhibit the activity of prostaglandin synthetase. Prostaglandins are a group of hormone-like substances that mediate a range of physiological functions, such as metabolism and nerve transmission. NSAIDs have analgesic, antipyretic, and anti-inflammatory properties (similar to aspirin). They are used for pain control, arthritis, and painful menstruation. Adverse reactions include GI upset (possible ulcers), and prolongation of bleeding time (reduction in platelet aggregation). Contraindications to the use of NSAIDs are impaired renal function, pregnancy, and GI disease (ulcers). Examples of NSAIDs: Proprionic acid derivatives: Naproxen sodium (Anaprox, Aleve) Oxaprozin (Daypro) Flurbiprofen (Ansaid) Ketoprofen (Orudis) Ibuprofen (Motrin, Advil) Fenoprofen (Nalfon) Naproxen (Naprosyn) Acetic acid derivatives: Indomethicin (Indocin) Diclofenac sodium (Votaren) Ketorolac (Toradol) Etodolac (Lodine) Sulindac (Clinoril) Alkanoic acid derivatives: Nabumetone (Relafen) Fenamic acid derivatives: Meclofenamate (Meclomen) Mefenamic acid (Ponstel) Oxicams Piroxicam (Feldene) Meloxicam (Mobic) Cox-2 inhibitors Celecoxib (Celebrex)

TM

PDF Editor

(PHARMACOLOGY

Asp/Acet/NSAIDs)

Which of the following agents has little value in treating acute inflammation? J

Ibuprofen
1 Acetaminophen

' Aspirin Naproxen Nabumetone

110 Copyright 2011-2012 - Dental Decks

MPHARMACOLOGY

Asp/Acet/NSAIDsJ

Your patient is taking the following medications. Which TWO of these are used to reduce the signs and symptoms of rheumatoid arthritis?

Amlodipine (Norvasc) Enalapril (Vasotec) Piroxicam (Feldene) Prednisone Fosamax (Alendronate sodium)

m
Copyright C 2011-2012 - Dental Decks

TM

PDF Editor

Acetaminophen (Tylenol) Acetaminophen inhibits central prostaglandin synthesis it is analgesic for low intensity pain and antipyretic. Because it is less effective than salicylates (aspirin) in blocking peripheral prostaglandin synthesis, it has no anti-inflammatory activity and does not affect platelet function and therefore will not affect clotting time. Note: In large doses (>7.5 g within 8 hours), acetaminophen can cause hepatic necrosis. Concurrent use with the following drugs may increase the risk of hepatotoxicity: barbiturates, hydantoins, carbamazepine, rifampin, sulfinpyrazone, and ethanol. Acute overdosage of acetaminophen can result in hepatotoxicity and is life threatening. Acetominophen is metabolized to a highly toxic intermediate product, which normally is detoxified by glutathione. When glutathione is depleted, the toxic intermediate attacks other cells, causing necrosis. Symptoms that appear in the first 24 hours are nausea, vomiting, drowsiness, lethargy, malaise, and confusion. Note: N-acetylcysteine (NAC) is the specific antidote for acetaminophen poisoning. Acetaminophen is preferred over aspirin when an analgesic or antipyretic drug is indicated and also the patient: Is allergic to aspirin Is asthmatic Is taking drugs such as probenecid or methotrexate Is at added risk for an ulcer Is experiencing bleeding Is taking anticoagulants Remember: Aspirin and Nonsteroidal Anti-inflammatory Agents (Ibuprofen, Naproxen, Nabumetone. and others) inactivate the enzyme known as cyclooxygenase. Since cyclooxygenase synthesizes the prostaglandins, the inhibition of this enzyme results in the inhibition of prostaglandin synthesis. Cyclooxygenase has an acronym of COX. Thus the Nonsteroidal Anti-inflammatory Agents (NSAIDs) are also known as COX inhibitors. As a result, they have analgesic, antipyretic, and anti-inflammatory actions. NSAIDs must be used cautiously in patients with peptic ulcer disease.

Piroxicam (Feldene) Prednisone Piroxicam (Feldene) is a non-steroidal anti-inflammatory drug (NSAID) that inhibits prostaglandin synthesis. It is used to manage inflammatory disorders and used for the symptomatic treatment of acute and chronic rheumatoid arthritis and osteoarthritis. Prednisone is a corticosteroid and has anti-inflammatory actions. It is used for the treatment of a wide variety of inflammatory diseases including rheumatoid arthritis and osteoarthritis. Remember: Common side effects of NSAIDs such as piroxicam (Feldene) include: - Gastric irritation - Heart burn - Nausea Short-term side effects of corticosteroids such as prednisone include: - Insomnia - Indigestion - Arthralgia Long-term side effects of corticosteroids include: - Edema (abdominal distension) - Psychological disturbances - Peptic ulcer - Osteoporosis - Muscle weakness

TM

PDF Editor

(PHARMACOLOGY

Asp/Acet/NSAIDs)

Celecoxib (Celebrex) is a member of which category of drugs?

Salicyclates Opiates COX-2 selective inhibitors Non-selective COX inhibitors Steroidal anti-inflammatories

112 Copyright 20! I-2012 - Dental Decks

^PHARMACOLOGY

Cholinergic)

If

The autonomic nervous system has cholinergic neurons that secrete:

Epinephrine Norepinephrine Acetylcholine Dopamine

113 Copyright 2011-2012 - Dental Decks

TM

PDF Editor

COX-2 selective inhibitors Cyclooxygenase, or COX, is the enzyme which produces prostaglandins. Two forms of COX exist: COX-1, and COX-2. The COX-1 enzyme produces prostaglandins in the GI Tract. The prostaglandins formed act as a protective substance against the formation of gastrointestinal ulcers. The traditional NSAIDs such as ibuprofen, naproxen, aspirin, and others inhibit the COX-1 enzyme thus diminishing the formation of the protective prostaglandins. Gastrointestinal ulcers are therefore a potential adverse effect with these drugs. The COX-2 enzyme produces prostaglandins at the sites of surgery, infection and inflammation. When this enzyme is inhibited, less prostaglandins are produced and there is less pain and inflammation. The traditional NSAIDs such as ibuprofen, naproxen and aspirin inhibit COX-2 along with COX-1. Thus they are effective in reducing pain and inflammation, but are capable of inducing gastrointestinal ulcers. The COX-2 selective inhibitors will reduce pain and inflammation without any significant risk of causing gastrointestinal ulcers. For the traditional NSAIDs such as ibuprofen, naproxen, and aspirin, because they inhibit both COX-1 and COX-2 enzymes, they belong to the category of non-selective COX inhibitors. For celecoxib (Celebrex), because it inhibits COX-2 enzyme only, it belongs to the category of COX-2 selective inhibitors. The COX-2 selective inhibitors: - Are not salicylates because they are not aspirin drugs - Are not opiates because they do not work like morphine - Are not steroidal anti-inflammatories because they are not corticosteroids such as hydrocortisone

Acetylcholine Cholinergic is a term for a nerve ending that releases acetylcholine (ACh) as the primary neurotransmitter; also, a synapse in which acetylcholine is the primary neurotransmitter. Acetylcholine receptors are called cholinergic receptors. They are subdivided as follows: Muscarinic receptors (sites): - At neuroeffector sites for all postganglionic cholinergic neurons (this is characteristic of all parasympathetic postganglionic nerves) - At neuroeffector sites of postganglionic sympathetic nerves to the sweat glands and a few blood vessels (these postganglionic nerves are also cholinergic) Nicotinic receptors (sites): - At the skeletal neuromuscular junction (involving somatic nerves) - At ganglionic sites (*** Tile same type of nicotinic receptor is present in sympathetic ganglia, parasympathetic ganglia, and the adrenal medulla) *** Drugs that resemble ACh in chemical structure and bind to these receptors imitate the effects of parasympathetic postganglionic activity. Important: 1. Preganglionic neurons of both divisions (parasympathetic and sympathetic) are cholinergic, as are postganglionic neurons of the parasympathetic division. 2. Postganglionic neurons of the sympathetic division are usually adrenergic. 1. The action of acetylcholine at postganglionic parasympathetic sites is often referred to as a muscarinic response. 2. The term nicotinic response is used to describe the stimulating action of ACh on the ganglia, as well as its action at the neuromuscular junction of skeletal muscle. 3. Muscarinic receptors are usually linked to Gq, phospholipase C (PLC), and Ca2*.

TM

PDF Editor

(PHARMACOLOGY

Cholinergic)

All of the following are cholinergic actions EXCEPT one. Which one is the EXCEPTION?

Slowing of the heart Dilation of the pupils The stimulation of the smooth muscles of the bronchi, GI tract, gallbladder, bile duct, bladder and ureters The stimulation of sweat, salivary, tear and bronchial glands

114 Copyright 2011-2012 - Dental Decks

(PHARMACOLOGY

Cholinergic)

AH of the following are antimuscarinic agents EXCEPT one. Which one is the EXCEPTION?

Atropine
1 Scopolamine 1 Glycopyrrolate (Robinul)

Mecamylamine (Inversine) Propantheline (Pro-Banthine)

115 Copyright 2011-2012 - Dental Decks

TM

PDF Editor

Dilation of the pupils *** This false; cholinergic action constricts the pupils. Cholinergic actions: The stimulation of sweat, salivary, tear and bronchial glands The stimulation of the smooth muscles of the bronchi, GI tract, gallbladder, bile duct, bladder and ureters (i.e., urination) Slowing of the heart (bradycardia) Constriction of the pupils (miosis) Important: Acetylcholine is the chemical mediator at all autonomic ganglia and parasympathetic postganglionic synapses. It is also the transmitter substance of the neuromuscular junction in skeletal muscle (local anesthetics prevent or reduce the liberation of ACh at the NMJ) and sweat glands. Acetylcholine causes an alteration in cell membrane permeability to produce the above actions. Anticholinergic actions: The secretions of all glands in the nose, mouth, pharynx and respiratory tract is inhibited (unpleasant "dry mouth") An inhibitory effect on motility throughout the GI tract (may cause constipation and urinary retention) The heart rate increases (tachycardia) A rise in body temperature Dilation of the pupils (mydriasis) Note: Termination of transmission by ACh takes place primarily by metabolism by acetylcholinesterase located on postsynaptic or postjunctional membranes.

Mecamylamine (Inversine) *** Mecamylamine is a nicotinic ganglion-blocking drug. The typical effects of anticholinergic drugs include mydriasis, antispasmodic actions and reduction in gastric and salivary secretions (dry mouth). Important: These drugs are contraindicated in patients with glaucoma. These drugs have no intrinsic activity of their own; they simply occupy the receptor site and prevent acetylcholine from occupying the same receptor. Accepted therapeutic indications include treating Parkinson's disease, motion sickness, postoperative bladder syndrome and traveler's diarrhea.
Synthetic Anticholinergic Drugs Drug Name GlycopvTrolate (Robinul) Major Use Inhibit salivation and excessive secretions preoperatively; control of upper airway secretions

Propantheline bromide (Pro-Banthine) Traveler's diarrhea and antisecretory Bcnztropine mesylate (Cogentin) Trihexyphenidyl HCL (Artane) Atropine Scopolamine Antiparkinsonism Antiparkinsonism To produce mydriasis and cycloplegia To prevent or reduce motion sickness

TM

PDF Editor

(PHARMACOLOGY

Cholinergic)

The currently available ganglionic blocker for clinical use is:

' Mecamylamine Hexamethonium Tetraethylammonium Trimethaphan

116 Copyright 20! I-2012 - Dental Decks

(PHARMACOLOGY

Cholinergic)

The prototype depolarizing neuromuscular blocking agent is:

Tubocurarine (Curare) Mi\ acurium (Mivacron) Succinylcholine (Anectine) Vecuronium (Norcuron) Doxacurium (Nuromax) Pancuronium (Pavulon) Atracurium (Tracrium) Cisatracurium (Nimbex) Rocuronium (Zemuron)
117

Copyright 20! I-20!2 - Dental Decks

TM

PDF Editor

Mecamylamine *** Hexamethonium, trimethaphan, and tetraethylammonium are no longer available in the U.S. for clinical use. Nicotinic receptor antagonists (nicotinic blocking agents) are divided into ganglionblocking drugs and neuromuscular blocking drugs. Remember: two major types of nicotinic receptors: 1. Those at the skeletal neuromuscular junction of the somatic system *** Neuromuscular blockers act here 2. Those at the autonomic ganglionic sites (both sympathetic and parasympathetic) *** Ganglionic blockers act here Although they are among the most potent agents available, ganglionic-blocking drugs are seldom used because of the annoying and sometimes disabling parasympathetic blockade. The side effects (caused by parasympathetic blockade) include a very pronounced xerostomia, constipation, blurred vision, and postural hypotension. These drugs have only very limited clinical uses. Mecamylamine (Inversine) is used clinically for: The treatment of severe or malignant hypertension An emergency hypertensive crisis *** It cause a rapid and reversible fall in blood pressure that enables it to immediately reverse an emergency hypertensive crisis. A "bloodless field" surgery

Succinvlcholine (Anectine) Neuromuscular blocking drugs are important for producing complete skeletal muscle relaxation and facilitate endotracheal intubation, as an adjunct to surgical anesthesia. These agents interact with nicotinic receptors at the skeletal neuromuscular junction. There are two classes of neuromuscular blockers: 1. Nondepolarizing: These agents competitively compete with acetylcholine at the nicotinic receptor. These agents bind to nicotinic cholinergic receptors and prevent acetylcholine from stimulating motor nerves, resulting in muscle paralysis. The prototype nondepolarizing agent is tubocurare (Curare). Other agents include mivacurium, vecuronium, doxacurium, pancuronium, atracurium, cisatracurium and rocuronium. Note: Neostigmine or pyridostigmine (which are cholinesterase inhibitors) can reverse the blockade of these agents. 2. Depolarizing (non-competitive): Succinylcholine (Anectine) is the only member of this group used in the United States. It acts like a nicotinic agonist and depolarizes (desensitizes) the neuromuscular end plate. It binds to the ACh receptor and stimulates depolarization causing initial excitation followed by block of neurotransmission and muscle paralysis. Important: Succinylcholine should be used with caution in patients with low levels of pseudocholinesterase, which breaks down succinylcholine. Respiratory failure may result. *** Succinylcholine may cause muscarinic responses such as bradycardia and increased glandular secretions. Very important: The major danger of all of these neuromuscular blocking drugs is too much paralysis (the patient cannot breathe). 1. Dantrolene is an skeletal muscle relaxing agent that acts within the skeletal muscle fiber rather than on the neuromuscular junction (does not block the nicotinic receptors). It inhibits the depolarization-induced release of Ca2* from the sarcoplasmic reticulum. The principal therapeutic applications of dantrolene are for the relief of spasticities associated with upper motor neuron disorders (i.e., stroke, cerebral palsy, and multiple sclerosis) and for the prophylaxis and treatment of malignant hyperthermia. 2. Botulinum toxin A (Botox) acts on the motor nerve terminal to prevent the release of ACh. It is used in ophthalmology to relax the extraocular muscles, and for muscle dystonias as well as to remove wrinkles.

TM

PDF Editor

(PHARMACOLOGY

Cholinergic)

All of the following are cholinesterase inhibitors EXCEPT one. Which one is the EXCEPTION?

Physostigmine
1 Edrophonium

Pyridostigmine Phentolamine Neostigmine

118 Copyright 2011-2012 - Dental Decks

(PHARMACOLOGY

Cholinergic)

All of the following are cholinergic drugs EXCEPT one. Which one is the EXCEPTION?

Acetylcholine Methacholine Bethanechol (Urecholine) Dobutamine Carbachol (Isopto-Carbachol) Pilocarpine hydrochloride

119 Copyright 2011-2012 - Dental Decks

TM

PDF Editor

Phentolamine *** Phentolamine (andphenoxybenzamine) are prototypes of non-selective alpha-adrenergic receptor blockers. Cholinesterase inhibitors act by blocking the enzyme (acetylcholinesterase) that degrades acetylcholine in the brain. This results in more acetylcholine in the synaptic cleft and enhances cholinergic transmission. These drugs act as indirect agonists at both muscarinic and nicotinic sites. Examples of cholinesterase inhibitors include: Edrophonium: reversible; very short duration of action; used to reverse curare-type drugs Neostigmine: reversible; extended duration of action; used to reverse curare-type drugs and to treat myasthenia gravis Physostigmine: reversible; short duration of action; used for glaucoma, and for an antidote for atropine Pyridostigmine: reversible; extended duration of action; used to treat myasthenia gravis Tacrine: reversible; extended duration of action; used to Alzheimer's disease Donepezil: reversible; extended duration of action; used to Alzheimer's disease Ma la th ion: irreversible; long duration of action; used as an insecticide Sarin: irreversible; long duration of action; used as a nerve gas Three cholinesterase inhibitors are commonly prescribed for Alzheimer's disease: Donepezil (Aricept), approved to treat all stages of Alzheimer's disease Rivastigmine (Exelon), approved to treat mild to moderate Alzheimer's Galantamine (Razadyne), approved to treat mild to moderate Alzheimer's Note: The stimulation of the skeletal muscle by excess acetylcholine eventually results in muscle paralysis.

Dohutamine

*** Dobutamine is an adrenergic agonist. Cholinergic drugs are compounds that mimic the actions of the endogenous neurotransmitter acetylcholine (ACh). Direct-acting, or cholinomimetic, agents combine with cholinergic receptors (muscarimic or nicotinic or both), to cause a response in an effector. These drugs include ACh, various choline-ester congeners of ACh. and some alkaloids. With few exceptions (i.e., nicotine and other ganglionic stimulants), these agents exert prominent muscarinic or parasympathomimetic effects. These drugs are longer-lasting than ACh because they are not subject to rapid metabolism like ACh. ACh is metabolized by acetylcholinesterase, located near receptors for ACh. In the plasma and other sites, ACh (and many other esters) are metabolized by pseudocholinesterase. The other cholinergic agonists used as drugs are metabolized slowly or not at all by these enzymes. Direct-acting agents: Choline esters: The most noticeable effects of these drugs are a fall in blood pressure attributable to generalized vasodilation, flushing of the skin, a slowing of the heart rate, and an increased tone and activity of both the GI and urinary tracts. Topical application of these drugs to the eye causes miosis and a decrease in intraocular pressure. These drugs include: Acetylcholine chloride: used in ophthalmology to produce miosis Bethanechol: used for postoperative abdominal distension and urinary retention Carbachol: used in ophthalmology to produce miosis Cholinergic alkaloids: These drugs include muscarine, pilocarpine, nicotine and lobelinc. Pilocarpine is the most useful alkaloid being employed as a miotic and to treat open angle glaucoma. Pilocarpine is also used to stimulate salivary flow in patients suffering from xerostomia due to radiation therapy in the treatment of head and neck cancer. Indirect-acting agents: Cholinesterase inhibitors: These drugs include physostigmine, edrophonium, pyridostigmine, neostigmine, malathion and sarin. They inhibit acetylcholinesterase at both mucarinic and nicotinic sites. They are indirect agonists at both muscarinic and nicotinic sites. Note: If any of the cholinergic agents are administered before acetylcholine, the action of acetycholine will be enhanced and prolonged.

TM

PDF Editor

(PHARMACOLOGY

Cholinergic)

(f

In dentistry, a cholinergic drug is used to:

Produce a dry field for taking impressions Calm an anxious patient Treat dry mouth by inducing salivation Reduce nausea

120 Copyright 20! 1-2012 - Dental Decks

(PHARMACOLOGY

Cholinergic)

All of the following statements concerning edrophonium are true EXCEPT one. Which one is the EXCEPTION?

> It is a direct-acting cholinergic agonist (cholinomimetic) It is a rapid-acting, short-duration, injectable cholinesterase inhibitor It is the drug of choice for diagnosing myasthenia gravis because of its rapid onset of action and reversibility > It is also useful in differentiating a myasthenic crisis from a cholinergic crisis

121 Copyright 2011-2012 - Dental Decks

TM

PDF Editor

Treat dry mouth by inducing salivation Cholinergic drugs used in dentistry are: Pilocarpine (Salagen), and Cevimeline (Evoxac). Pilocarpine (Salagen) - is indicated for treatment of xerostomia caused by salivary gland hypofunction resulting from radiotherapy for cancer of the head and neck. It's pharmacologic category is cholinergic agonist. Prevalent side effects are excess sweating and nausea/heartburn/diarrhea due to the cholinergic nature of the drug. Cevimeline (Evoxac) - is indicated for the treatment of symptoms of xerostomia in patients with Sjogren's syndrome. It's pharmacologic category is cholinergic agonist. Prevalent side effects are increased sweating and nausea/heartburn/diarrhea due to the cholinergic nature of the drug.

It is a direct-acting cholinergic agonist (cholinomimetic) *** This is false; edrophonium is an indirect-acting cholinergic agonist (cholinomimetic) as are pyridostigmine, physostigmine, and neostigmine (they are all cholinesterase inhibitors). Note: These drugs are indirect agonists at both muscarinic and nicotinic sites. Pyridostigmine is prescribed in the treatment of myasthenia gravis, however edrophonium is used to diagnose myasthenia gravis and not in the treatment of it due to its very short duration of action. Symptoms of a cholinergic crisis include: bradycardia (decreased heart rate), lacrimation, extreme salivation, vasodilation and muscle weakness. Because a cholinergic crisis can result in muscle weakness like that of a myasthenic crisis, distinguishing the two conditions may be difficult. Administration of a short-acting cholinomimetic such as edrophonium will improve a myasthenic crisis but worsen a cholinergic crisis. Remember: Typical cholinergic effects caused by stimulation of acetylcholine receptors (cholinergic receptors) include salivation, miosis, excessive sweating, flushing, increased GI motility and bradycardia.

TM

PDF Editor

(PHARMACOLOGY

Cholinergic)

Poisoning with an organophosphate cholinesterase inhibitor can be treated with:

Edrophonium Carbachol
1

Pralidoxime Nicotine

122 Copyright 2011-2012 - Dental Decks

(PHARMACOLOGY

Cholinergic)

The muscarinic effects of cholinergic agonists include all of the following EXCEPT one. Which one is the EXCEPTION?

Miosis Flushing Bronchoconstriction Increased GI motility Increased urination Tachycardia Salivation Increased urination Sweating
123 Copyright 2011-2012 - Dental Decks

TM

PDF Editor

Pralidoxime Pralidoxime (Protopam) is a cholinesterase reactivator which is used as an antidote to reverse muscle paralysis resulting from organophoshate anticholinesterase pesticide poisoning. It is also used to reverse the effects of an overdosage of anticholinesterase agents used in the treatment of myasthenia gravis (i.e., pyridostigmine and ambenomium). Symptoms of organophosphate poisoning include: excessive salivation, bronchocostriction, diarrhea and skeletal muscle fasciculations (twitching). Organophosphates are esters of phosphoric acid and an organic alcohol that inhibit the enzyme cholinesterase. Examples include: Isoflurophate: used in the treatment of glaucoma Malathion: a widely used insecticide Parathion: an insecticide Echothiophate: used in the treatment of glaucoma Tabun: one of the most potent and toxic nerve gases Soman: nerve gas Sarin: nerve gas

Tachycardia *** Bradycardia is a muscarinic effect of cholinergic agonists. Direct-acting cholinergic drugs include: methacholine, carbachol, bethanecol and pilocarpine. Indirect-acting cholinergic drugs (cholinesterase inhibitors) include: physostigmine, edrophonium, neostigmine, pyridostigmine, malathion, echothiophate, sarin and soman. Effects of anticholinesterases: Muscarinic: Nicotinic: Miosis Bronchoconstriction Muscle twitching Salivation Increased GI motility Weakness Sweating Urination Tachycardia Bradycardia Increase in BP \nti muscarinic drugs block the effect of acetylcholine and all drugs that stimulate muscarinic receptors. Atropine and scopolamine are prototypes. Other antimuscarinic drugs include: glycopyrrolate. propantheline, benztropinc, cyclopcntolate, tropicamide, trihexyphenidyl, homatropine, oxybutynin, and ipratropium. Antimuscarinic drugs produce the following effects: Salivary glands: reduced secretion (atropine) GI tract: reduced peristalsis, reduced secretion (propantheline, glycopyrrolate) Sweat glands: reduced secretion (atropine) Eyes: mydriasis (homatropine, cyclopcntolate, tropicamide) Bronchi: bronchodilation, reduced secretion (ipratropium) Bladder: urinary retention (oxybutynin) CNS: antimotion sickness (scopolamine) antitremor activityto treat parkinsonism (benztropine, trihexyphenidyl) 1. Contraindications for using antimuscarinic drugs include: narrow-angle glaucoma, prostatic hyperplasia, and tachycardia. 2. Pilocarpine and cevimeline (both cholinergic agonists) are used to stimulate salivary flow 3. SAL-TKO?lNE(atropine sulfate, USP tablets) is indicated to reduce salivation and is endorsed with the ADA Seal of Acceptance

TM

PDF Editor

(PHARMACOLOGY

Cholinergic)

All of the following drugs may be useful in controlling salivary secretions to help in obtaining a dry field EXCEPT one. Which one is the EXCEPTION?

Atropine sulfate Carbachol Glycopyrrolate (Robinul) Belladonna derivatives Propantheline bromide (Pro-banthine)

124 Copyright 20] 1-20(2 - Dental Decks

(PHARMACOLOGY

Corticosteroids)

The major natural glucocorticoid is:

Triamcinolone Cortisol Dexamethasone Prednisone Prednisolone

125 Copyright O 2011-2012 - Dental Decks

TM

PDF Editor

Carbachol *** Carbachol is a direct-acting cholinergic drug. The other drugs are all classified as anticholinergics. They block postganglionic cholinergic fibers. Contraindications to their use include: Glaucoma Cardiovascular problems Obstruction of the GI or GU tract Asthma These drugs also reduce spasms of smooth muscle in the bladder, bronchi, and intestine; relax the iris sphincter; decrease gastric, bronchial, and salivary secretions; decrease perspiration; and accelerate impulse conduction through the myocardium by blocking vagal impulses.

Cortisol
The corticosteroids arc steroid hormones produced by the adrenal cortex. They consist of two major groups: 1. Glucocorticoids: have important effects on metabolism, catabolism, immune responses and inflammation. The majority of the anti-inflammatory and immunosuppressive actions of the glucocorticoids are probably the result of their action on arachidonic acid metabolism. They induce the synthesis of a protein that inhibits phospholipase A2, thus decreasing the production of both prostaglandins and leukotrienes. The major natural glucocorticoid is Cortisol. The synthetic glucocorticoids include hydrocortisone (Cortef). cortisone, prednisone (Delatasone), prednisolone (Delta-Cortef), dexamethasone (Decadron), triamcinolone (Aristocort), methylyprednisolone (Medral), and betamethasone (Cestone). Glucocorticoids are most often used as anti-inflammatory and immunosuppressive agents. Adverse effects of the short-term administration of systemic glucocorticoids include secondary infections, hyperglycemia, and a range of mood and behavioral changes. Long-term therapy may cause osteoporosis, cataracts, hypertension, myopathy, and adrenal insufficiency. 2. Mineralocorticoids: regulate sodium and potassium reabsorption in the collecting tubules of the kidney. The major natural mineralocorticoid in humans is aldosterone. Other mineralocorticoids include deoxycorticosterone and fludrocortisone. Mineralcorticoids are used in replacement therapy in hypoadrenocorticism or Addison's disease. Note: Corticosteroids do not cure any disease. They represent replacement only in Addison's disease. Contraindications to corticosteroid use include: latent TB or fungal infection, AIDS, herpes infections and patients with peptic ulcer disease (specifically, gastric ulcers) these drugs themselves may cause peptic ulcers. Toxic effects of the corticosteroids include growth inhibition, hyperglycemia, osteoporosis, psychosis and salt retention.

TM

PDF Editor

(PHARMACOLOGY

Corticosteroids)

All of the following are pharmacologic effects of glucocorticoids EXCEPT one. Which one is the EXCEPTION? Stimulate protein breakdown, which results in increased plasma amino acid levels Stimulate gluconeogenesis in the liver and inhibit peripheral glucose use Impaired wound healing Reduce the immune response Decreased lipolysis Inhibit local edema, capillary dilation, migration and activation of white blood cells, and phagocytosis by macrophages Increase hemoglobin concentration and increase the numbers of circulating red blood cells and platelets
126 Copyright S 2011-2012 - Dental Decks

^PHARMACOLOGY

Corticosteroids)

The major natural mineralocorticoid in humans is:

Dexamethasone Aldosterone Cortisol Prednisone Triamcinolone

127 Copyright 2011-2012 - Dental Decks

TM

PDF Editor

Decreased lipolvsis *** This is false; glucocorticoids affect the mobilization of fats from areas of deposition. Increased lipolyisis occurs in areas of adipose accumulation, and serum fatty acid concentration increases. Other effects of glucocorticoids include an anti-inflammatory action, immunosuppression, and an a n I i-allergenic action. Note: These effects occur in target cells following the interaction of the steroid with a specific glucocorticoid receptor. Common Disorders Treated with Glucocorticoids Disorder Examples Contact dermatitis. Erythema multiforme. Psoriasis, Pemphigus, Stevens-Johnson syndrome, Seborrheic dermatitis Bronchial asthma. Sarcoidosis. COPD Adrenocortical insufficiency Systemic lupus erythematosus. Polymyositis, Temporal arteritis Rheumatoid arthritis. Psoriatic arthritis. Acute gouty arthritis Multiple sclerosis. Cerebral edema. Acute stroke and spinal cord injury Allergic conjunctivitis. Uveitis, Herpes zoster ophthalmicus

Allergic conditions Seasonal allergic rhinitis. Serum sickness. Drug hypersensitivity reaction Dermatologic Respiratory Endocrine Collagen, vascular Rheumatic Neurologic Ophthalmic

Note: Beclomethasone, budesonide and flunisolide are special glucocorticoids (usedas inhalers) that have been developed for use in chronic asthma and bronchial disease. These agents readily penetrate the airway mucosa but have very short half-lives after they enter the blood, so that systemic effects and toxicity are greatly reduced. Remember: The pharmacologic effects of mineralocorticoids include an increase in sodium retention and an increase in potassium depletion which can lead to edema and hypertension if excessive and may lead to dehydration and hypotension if insufficient.

Aldosterone *** Cortisol, prednisone, dexamethasone, and triamcinolone are all glucocorticoids. Aldosterone is secreted by cells located in the zona glomerulosa of the adrenal cortex. The secretion of aldosterone is regulated by ACTH and by the renin-angiotensin system and is very important in the regulation of blood volume and pressure. Aldosterone promotes reabsorption of sodium into the blood from the glomerular filtrate. Potassium is lost in the urine because of the electronegativity that is created by the reabsorption of sodium in the kidney tubules. Note: Increased blood aldosterone levels will result in high sodium and low potassium levels in the plasma. Remember: Decreased sodium concentration causes the juxtaglomerular cells of the kidneys to secrete renin, which converts angiotensinogen to angiotensin I. Angiotensin I is converted to angiotensin II, which, in turn, stimulates the adrenal cortex to release aldosterone. Noteol 1. Addison's disease is caused by the hyposecretion of aldosterone and cortisol. 2. ADH (Vasopressin) decreases the production of urine by increasing the reabsorption of water by the renal tubules (it increases the permeability of the collecting ducts and distal tubules). Without ADH, there would be extreme loss of water into the urine. 3. At high concentrations, ADH causes arterioles to constrict (increases blood pressure).

TM

PDF Editor

(PHARMACOLOGY

Corticosteroids)

Which corticosteroid is administered by inhalation to treat asthma!:

Hydrocortisone Prednisone Cortisone Fluticasone (Flonase) Methylprednisolone (Medrol)

128 Copyright 20! 1-2012 - Dental Decks

(PHARMACOLOGY

Corticosteroids)

All of the following drugs are classified as corticosteroids (steroids) EXCEPT one. Which one is the EXCEPTION?

Hydrocortisone Methylprednisolone Prednisone Compazine Triamcinolone Dexamethasone

129 Copyright 2011-2012 - Dental Decks

TM

PDF Editor

Fluticasone (Flonase) Corticosteroids in the inhaled form, decrease the inflammation in the airway in asthma. Reduction of inflammation enhances the bronchodilating effects of the beta2-adrenergic agonists. The following are some of the other popular inhaled corticosteroids used in the treatment of asthma: Triamcinolone (Azmacort) Beclomethasone (Beconase) Budesonide (Pulmicort) Flunisolide (AeroBid) Note: Inhaled steroids very often cause a fungal infection of the mouth and throat. Combination products used in the treatment of COPD and asthma include: albuterol/ipratropium (Combivent) fluticasone/salmeterol (Advair Diskus) budesonide/formoterol (Symbicort) Important: Leukotriene modifiers act on inflammatory mediators of asthma, the LTs (also known as slow-reacting substance of anaphylaxis fSRS-AJ), which contributes to airway obstruction. There are two subclasses: (1) Leukotriene receptor antagonists which include montelukast (Singulair) and zafirlukast (Accolate) (2) 5-lipoxygenase inhibitor (leukotriene synthesis inhibitor) which includes zileuton (Zyflo).

Compazine *** Prochlorperazine (Compazine) is an anti-psychotic medication in a group of drugs called phenothiazines. Two types of corticosteroids: 1. Glucocorticoids affect carbohydrate, lipid, and protein metabolism. They are used as to treat numerous disorders, primarily through their anti-inflammatory and immunosuppressive actions. 2. Mineralocorticoids regulate sodium and potassium metabolism. 1. These drugs are used to treat a variety of conditions which include asthma, arthritis, allergies, aphthous stomatitis, lupus erythematosus, and TMJ pain. 2. Contraindications to their use include latent infections (fungal, viral, or bacterial), AIDS, herpes infections, gastric ulcers, and congestive heart failure. 3. Adverse reactions include Cushing's syndrome (obesity and weakening of muscles), hyperglycemia, osteoporosis, peptic ulcers, and an increased risk of infection. 4. Corticosteroids do not cure any disease. They represent replacement only in Addison's disease. Inhaled corticosteroids used for asthma do not achieve significant blood levels to cause the adverse effects listed above for systemic agents. Popular aerosol corticosteroids are triamcinolone (Azmacort), beclomethasone (Beconase), fluticasone (Flovent) and budesonide (Pulmicort). Localized infections with Candida albicans occur frequently in the mouth and pharynx with repetitive use of inhalant corticosteroids. Nasal spray corticosteroids used for seasonal allergies also do not achieve significant blood levels, and are use for their localized effects. Popular nasal spray corticosteroid products are triamcinolone (Nasocort), fluticasone (Flonase) and budesonide (Rhinocort).

TM

PDF Editor

(PHARMACOLOGY

Cardiovascular)

All of the following drugs are direct vasodilators EXCEPT one. Which one is the EXCEPTION?

Hydralazine (Apresoline) Diazoxide (Proglycem) Captopril (Capoten) Sodium nitroprusside (Nipride) Nitroglycerin (Nitrostat)

130 Copyright 2011-2012 - Dental Decks

f PHARMACOLOGY

Cardiovascul:
)

AH of the following drugs are used to prevent or to provide relief of angina pectoris EXCEPT one. Which one is the EXCEPTION?

| I

1 Nitroglycerin (Nitrostat)

Isoflurophate Nifedipine (Procardia) Diltiazem (Cardizem) Propranolol (Inderal) Isosorbide (Isordil)

131 Copyright 02011-2012- Dental Decks

TM

PDF Editor

Captopril (Capoten) *** Captopril is an angiotensin-converting enzyme inhibitor. Direct vasodilators exert their antihypertensive effect by a direct vasodilator action on the smooth muscle of arterioles, resulting in a decrease in peripheral resistance and blood pressure. Compensatory responses may be marked and include salt retention and tachycardia. Adverse side effects include GI upset, headache, dizziness and tachycardia. Note: nitroprusside, diazoxide, hydralazine and nitroglycerin are parenteral vasodilators which are used in hypertensive emergencies.

Calcium channel blockers are also effective vasodilators (indirect) and have been applied to the management of hypertension. Verapamil (Calan SR), Nifedipine (Procardia), and Diltiazem (Cardizem) have been given orally for the treatment of mild to moderate hypertension. Remember: Hydralazine and minoxidil are peripheral vasodilators.

Isoflurophate *** Isoflurophate is an organophosphate cholinesterase inhibitor used in the treatment of glaucoma Angina pectoris is the pain in the heart and chest which occurs during the occlusion of coronary arteries. Triggers that can cause occlusion are physical exertion, increased blood pressure, and vasoconstriction. Antianginal drugs work by reducing cardiac rate and force, reducing peripheral vascular resistance, or dilating coronary blood vessels. Nitroglycerin is a coronary artery vasodilator. It relaxes blood vessels to provide increased blood flow and oxygenation to the heart muscle. It is sublingually effective within 2-4 minutes. The nitroglycerin skin patch releases the drug over a 12 hour period to provide sustained blood levels for prevention of angina. The two most common adverse effects caused by nitroglycerin are orthostatic hypotension and headache. Nifedipine (Procardia) and diltiazem (Cardizem) are calcium channel blockers used to prevent angina attacks. These drugs are used to dilate coronary blood vessels for improved blood flow to heart muscle. Note: Calcium channel blockers as a class have been associated with causing gingival hyperplasia. Propranolol (Inderal) is representative of the beta-blockers used to prevent angina attacks. Atenolol (Tenormin) is another popular beta-blocker used for this purpose. Betablockers are used to decrease the work load of the heart such that less oxygen is required.

TM

PDF Editor

PHARMACOLOGY

Cardiovascular
)

Amyl nitrite is used in the emergency treatment of cyanide poisoning because it:

Oxidizes hemoglobin Irreversibly binds cyanide Competes with cyanide for binding of cytochromes Inhibits tubular reabsorption of cyanide

132 Copyright C 20! I-2012 - Dental Decks

QPHARMACOLOGY
The positive inotropic effect of I)i<j Is dependent upon a normal cardiac rhythm Directly increases the force of myocardial contractions Is antagonized by beta-blockers All of the above

Cardiovascular

inoxin):

133 Copyright 2011 -2012 - Dental Decks

TM

PDF Editor

Oxidizes hemoglobin *** Amyl nitrite oxidizes hemoglobin to methemoglobin which binds cyanide tightly, keeping it in the peripheral circulation and preventing its access to tissues. Amyl nitrite is a vasodilator and a highly volatile substance administered by inhalation only. It is the most rapidly acting of the antianginal drugs, producing effects within 10 seconds. Its duration of action is only 3 to 5 minutes. Because of its extreme potency, there are uncomfortable side effects that invariably occur with its use (fainting and a pounding headache). Important: This drug is rarely prescribed and is not the first drug of choice in treating angina. It is abused to produce euphoria and as a sexual stimulant. Other anti-anginal drugs include: 1. Nitrate: Nitroglycerin *** This drug is the single most effective agent available for the management of acute angina episodes. Note: It dilates mostly veins. 2. Non-nitrate vasodilator: Dipyridamole (Persantine) 3. Beta adrenergic blocking drugs: reduce cardiac rate and force Propranolol (Inderal) Nadolol (Corgard) Atenolol (Tenormin) 4. Calcium channel blocking drugs: dialte peripheral and coronary blood vessels Verapamil (Isoptin, Calan) Diltiazem (Cardizem) Nifedipine (Procardia)

Directly increases the force of myocardial contractions *** This positive inotropic effect is independent of a normal sinus rhythm and adrenergic stimulation. The cardiac glycosides are often called "digitalis" because several come from the digitalis (foxglove) plant. Digoxin (Lanoxin) is the most versatile and widely used. Note: They are used to treat most supraventricular arrhythmias, cardiogenic shock and chronic heart failure. These drugs help the heart beat more strongly (positive inotropic effect), more slowly (bradycardia) and more efficiently. Cardiac glycosides inhibit the Na+-K+-ATPase membrane pump by inhibiting the adenosine triphosphate enzymes (transport ATPase or Na-K-ATPase). Na+-K+-ATPase splits adenosine triphosphate in the nerve and muscle cell and thus provides the energy necessary for transporting sodium across the cell membrane. Key point: This inhibition of the Na'-K*-ATPase enzyme leads to an increased calcium ion influx which augments the positive inotropic effect of cardiac glycosides. Adverse side effects include nausea and vomiting, appetite loss, diarrhea, ventricular arrhythmias, heart block, and visual and mental disUirbances. Contraindications to their use include ventricular fibrillation and ventricular tachycardia. Drug interactions: Many drugs affect digoxin levels. However, digoxin does not affect the levels of other drugs. In addition, when beta-blockers are added to digoxin in patients with AV conduction abnormalities, complete heart block can result. Erythromycin, clarithromycin and tetracycline may increase digitalis absorption and toxicity. Thyroid replacement therapy increases dose requirements of digoxin. Drugs that lower plasma potassium levels (e.g., thiazide and loop diuretics) increase digitalis toxicity. Drug treatment of mild to moderate heart failure proceeds in the following order:(l) Diuretics in patients with fluid retention (2) ACEIs or ARBs in all patients unless contraindicated (3) Beta-blockers in all stable (minimal fluid retention) patients unless contraindicated and (4) Digoxin. Note: If these medications are not sufficient to control heart failure, the following additional drugs may be given: (1) Spironolactone (an aldosterone antagonist) (2) Nitrates and hydralazine and (3) Calcium channel blockers (i.e., amlodipine andfelodipine only) Remember: Most drugs useful in treating cardiac arrhythmias act primarily by increasing the refractory period of cardiac muscle.

TM

PDF Editor

(PHARMACOLOGY

Cardiovascular)

All of the following are angiotensin-converting enzyme (ACE) inhibitors EXCEPT one. Which one is the EXCEPTION?

Captopril (Capoten)

Hydralazine (Apresoline) Enalapril (Vasotec) Lisinopril (Zestril) Fosinopril (Monopril)

134 Copyright 20! 1-2012 - Dental Decks

(PHARMACOLOGY

Cardiovascular)

AH of the following drugs are Class II antiarrhythmic agents EXCEPT one. Which one is the EXCEPTION?

Propranolol (Inderal) Acetbutolol (Sectral) Metoprolol (Lopressor) Verapamil (Calan)

135 Copyright 2011-2012 - Dental Decks

TM

PDF Editor

Hydralazine (Apresoline) *** Hydralazine is a direct peripheral vasodilator. ACE inhibitors interfere with the conversion of angiotensin I (a weak vasoconstrictor) to angiotensin II (a highly effective constrictor). They do this by being inhibitors of angiotensin-converting enzyme (ACE). These drugs are used to treat hypertension and congestive heart failure. Adverse effects include cough, hypotension, neutropenia, anorexia and polyuria. Note: They can alter the sense of taste and in a few rare cases, cause angioneurotic edema. Angiotensin II is a potent vasoconstrictor and is a stimulus for aldosterone release from the adrenal glands. Reduction in aldosterone secretion results in less water absorption and sodium/potassium exchange in the distal renal tubule, causing a slight increase in serum potassium. ACE inhibitors inhibit the breakdown of bradykinin, a potent and naturally occurring vasodilator, by blocking the enzyme kininase II. This though, is thought to be the cause of the cough commonly experienced by patients who take this class of drugs. Other ACE inhibitors (ACEIs) include: Benazepril (Lotensin) Ramipril (Altace) Moexipril (Univasc) Quinapril (Accupril) Captopril (Capoten) Trandolapril (Mavik) Lisinopril (Zestril) Elanapril (Vasotec)

Angiotensin II receptor blockers (ARBs) include: Losartan (Cozaar) Valsartan (Diovan) Candesartan (Atacand) Eprosartan (Teveten) Olmesartan (Benicar) Telmisartan (Micardis) Irbesartan (Avapro)

*** ARBs block the effects of angiotensin II by blocking the binding of angiotensin II to its receptors. They do not effect bradykinin. Adverse effects include dizziness, diarrhea and myalgia. The antihypertensive effects of ARBs have been proven comparable with those of the ACEIs. ACEIs and ARBs are of particular values for the treatment of hypertensive patients who have concomitant illnesses such as diabetes, renal insufficiency, left ventricular dysfunction, and CHF. Note: ACE inhibitors and Angiotensin II receptor blockers indirectly inhibit fluid volume increases when interfering with angiotensin II because angiotensin II stimulates the release of aldosterone, which promotes sodium and water retention.

Verapamil

(Calan)

The Vaughan-Williams classification system traditionally has been used to classify antiarrhythmic drugs. This scheme places the available agents into one of four classes, usually denoted by Roman numerals I IV. This system is loosely based on the channel or receptor involved. Class I: Sodium channel blockers Class I drugs are further classified on the basis of their effects on action potential duration: IA agents include: Quinidine (Quinidex), procainamide (Procan SR) and disopyramidc (Norpace). These agents prolong the action potential. IB agents include: Lidocaine(Xylocaine), mexiletine (Mexitil) and tocainide (Tonocard. These agents shorten the action potential. IC agents include: Flecainide (Tambocor) and propafenone (Rythmol). These agents have no effect on action potential duration. Class II: Beta-adrenergic blockers - Propranolol (Inderal) is the prototype antiarrhythmic beta-blocker. Other drugs in this class include: esmolol (Brevibloc) acetbutolol (Sectral) and metoprolol (Lopressor). These agents increase refractory period, decrease conduction velocity and reduce automaticity. Class III: Potassium channel blockers - Amiodarone (Cordarone) is the prototype drug in this class. Sotalol (Betapace), ibutilide (Corvert) and dofetilide (Tikosyn) are also in this class. These drugs increase refractory period and reduce automaticity. Class IV: Calcium channel blockers - Verapamil (Calan) is the prototype. Diltiazem is also included in this group. These drugs increase refractory period, decrease conduction velocity and reduce automaticity. Note: Miscellaneous antiarrhythmic drugs include: adenosine (Adenocard) which increases refractory period and reduces automaticity and digoxin (Lanoxin) which increases the force of contraction of the heart muscle and decrease conduction velocity.

TM

PDF Editor

(PHARMACOLOGY

Cardiovascular)

Amiodarone (Cordarone) is the most potent and "broad-spectrum" antiarrhythmic compound currently available. Procainamide is an antihypertensive drug used to treat high blood pressure.

The first statement is true; the second statement is false The first statement is false; the second statement is true Both statements are true Both statements are false

136 Copyright 2011 -20! 2 - Dental Decks

^PHARMACOLOGY

Cardiovascular)

Verapamil is useful for the treatment of:

Angina
1 Hypertension

Supraventricular tachyarrhythmias All of the above

137 Copyright 20! 1-20I2 - Dental Decks

TM

PDF Editor

The first statement is true; the second statement is false Procainamide is a Class 1A antiarrhythmic agent that is used in the treatment of several cardiac arrhythmias including atrial fibrillation, atrial flutter, paroxysmal atrial tachycardia, and ventricular tachycardia. It is not used as an antihypertensive. It is a derivative of the ester local anesthetic procaine. Procainamide has properties similar to those of other Class IA agents, quinidine and disopyramide. These drugs decrease myocardial conduction velocity, excitability, and contractility by inhibiting the influx of sodium through "fast" channels of the myocardial cell membrane, thereby increasing the recovery period after repolarization. Note: Quinidine is primarily used to treat supraventricular tachyarrhythmias. Remember: Generally speaking, the use of beta-blockers (Class II agents) as antiarrhythmics is reserved for patients who require only control of ventricular rate during atrial tachyarrhythmias or who have mildly symptomatic ventricular arrhythmias. Side effects include bradycardia and hypotension. Important: Amiodarone (Cordarone) is generally considered a Class III agent even though it also blocks sodium channels, a Class I action. It is unique in that it is the most potent and "broad-spectrum" antiarrhythmic compound currently available. It blocks sodium, calcium, and potassium channels as well as beta receptors. It has impressive efficacy in suppressing both supraventricular and ventricular arrhythmias. Note: Adverse effects include pulmonary fibrosis, thyroid abnormalities, skin discoloration, and peripheral neuropathy. I-^ 1. Common adverse effects of some antiarrhythmics: Procainamide: mental changes, torsade de pointes (TDP - is an uncommon variant of ventricular tachycardia) Quinidine: hypotension, cinchonism (headache, ringing in the ears, deafness), and torsade de pointes Lidocaine: convulsions Propranolol: bronchoonstriction, heart block Calcium channel blockers: flushing, headache, hypotension, gingival hyperplasia and reduced contractility of the heart

All of the above Verapamil is the prototypical Class IV (calcium channel blockers) antiarrhythmic agent. It inhibits the intracellular entry of calcium through the slow channels of the calcium dependent tissues of the myocardium, which are concentrated in the SAand AV nodes. It is the drug of choice for the suppression of paroxysmal supraventricular tachycardias stemming from the AV node (which is characterized by a rapid cardiac rate, usually 160-190 per minute). Of the calcium channel antagonists available, only verapamil and diltiazem possess significant antiarrhythmic activity. Note: Calcium channel blockers (e.g., verapamil, diltazem and nifedipine) are useful as antianginal agents and antihypertensive agents as well. They block calcium entry through the membranous calcium ion channels of cardiac and vascular smooth muscle. This has three effects: 1. Peripheral arterioles dilate (vasodilators) and total peripheral resistance decreases, reducing after-load and reducing myocardial oxygen requirements. 2. Increases oxygen delivery to the myocardium. 3. Reduces blood pressure if hypertension is present. Important: Nitrites (amyl nitrite) and nitrates (nitroglycerin) are fast-acting antianginal agents that are used to relieve acute anginal attacks. Note: For angina, nitroglycerin and nifedipine are usually used before verapamil.
Drugs used in acute hypertension Hypertensive emergencies (parenteral) Hypertensive urgencies (oralpreferred) Sodium nitroprusside Nitroglycerin Trimethaphan Diazoxide Hydralazine Clonidine Nifedipine Captopril Labetalol

TM

PDF Editor

(PHARMACOLOGY

Cardiovascular)

Ail of the following drugs are classified as low molecular weight heparin type anticoagulants EXCEPT one. Which one is the EXCEPTION?

Enoxaparin (Lovenox) Dalteparin (Fragmin) Tinzaparin (Innohep) Clopidogrel (Plavix)

138 Copyright C 2011-2012 - Dental Decks

MPHARMACOLOGY

Cardiovascular)

All of the following drugs are classified as glycoprotein Ilb/IIIa inhibitor type of anti-platelet agents EXCEPT one. Which one is the EXCEPTION?

Abci.ximab (Reopro) Anagrelide (Agiylin) Eptifibatide (Integrilin) Tirofiban (Aggrastat)

139 Copyright 2011 -2012 - Dental Decks

TM

PDF Editor

Clopidogrel (Plavix)
** Clopidogrel (Plavix) inhibits blood clotting by inhibiting platelet aggregation in an irreversible manner. Votes?/ 1. These type of anticoagulants agents (low molecular weight heparins) arc used to treat acute symptomatic deep vein thrombosis. They arc used to prevent deep vein thrombosis following knee or hip surgery. 2. They arc administered subcutaneously since they are unable to be absorbed from the GI tract. 3. Standard heparin consists of components with molecular weights ranging from 4,000 to 30.000 daltons with a mean of 16,000 daltons. Low molecular-weight heparins range in molecular weights from 2,000 to 8,000 daltons. 4. Heparin acts at multiple sites in the coagulation system and binds with antithrombin HI at two specific sites, resulting in its anticoagulant effect. At the first site factor Xa is neutralized and at the second site factor Ha (thrombin) is neutralized. 5. Low molecular weight heparins have a small effect on partial thromboplastin lime but strongly inhibit factor Xa (not /la). 6. The antidote for heparin is protamine; the antidote for warfarin is vitamin K.
Some Anticoagulants Class Heparin group Heparin Low molecular weight heparin Subclass Generic/trade Name heparin sodium enoxaparin/Lovenox dalteparin/Fragmin Onzaparin/Innohep Heparinoids danaparoid'Orgaran fondaparimrx/AriXtra Direct thrombin inhibitors bivalirudin/Angiomax argatroban/Argatroban lepirudin/Refludan Oral anticoagulants warfarin/Coumadin anisindione/Miradon

Anagrelide (Agrylin)
Glycoprotein Hb/IIla inhibitors are reversible anti-platelet agents used to prevent acute cardiac ischemic complications and used in patients with acute coronary syndrome. They are administered intravenously. These agents block the platelet glycoprotein I lb/11 la receptor, the binding site for fibrinogen, von Willebrand factor, and other ligands. Inhibition of binding at this final common receptor reversibly blocks platelet aggregation and prevents thrombosis. Platelet aggregation inhibition is reversible following cessation of the IV administration of the drug minutes after an intravenous infusion. Note: The glycoprotein Ilb/IIIa inhibitors have a rapid onset of action. Their maximal antiplatelet effect occur within minutes after an intravenous infusion. Important: The most serious adverse effects of GPIIb/IIIa antagonists include major bleeding, intracerebral hemorrhage and thrombocytopenia.
Some Anticoagulants Class Platelet aggregation inhibitors Traditional Subclass Generic/trade Name acetylsalicylic acid/aspirin dipyridamole/Persantine Adensosine diphosphate-induced clopidogrel/Plavix platelet-fibrinogen binding inhibitors ticIopidine/Ticlid Platelet glycogen Ilb/IIIa inhibitors abciximab/ReoPro epti fibatide/lnlegrelin t i rofi ban/A ggrastat Platelet-reducing agent Phosphodiesterase III inhibitors anagrelide/Agrylin cilostazol/Pletal Thrombolytic agents alteplase/Acti vase reteplase/Retavase streptokinase/Streptase tenecteplase/TNKase urokinase/Abbokinase

TM

PDF Editor

(PHARMACOLOGY

Cardiovascular)

AH of the following drugs are classified as what type?

Lepirudin (Reflidan) Argatroban Danaparoid (Orgaran) Bivalirudin (Angiomax)

140 Copyright 2011-2012 - Dental Decks

(PHARMACOLOGY

Cardiovascular)

For a patient who is taking anticoagulants, what is the most valuable test used in evaluating the patient as a surgical risk?

PTT (Partial Thromboplastin Time) PT (Prothrombin Time) Platelet count

141 Copyright 2011-2012 - Dental Decks

TM

PDF Editor

Thrombin-inhibitor type anticoagulants These agents are administered intravenously for prevention of post-operative deep vein thrombosis following elective hip replacement surgery; for prophylaxis or treatment of thrombosis in adults with heparin-induced thrombocytopenia. Their mechanism of action is through the direct inhibition of thrombin within the coagulation pathway, thus inhibiting fibrin formation.

PT (Prothrombin Time) This test is a one-stage test for detecting certain plasma coagulation defects owing to a deficiency of factors V, VII, or X. Thromboplastin and calcium are added to a sample of the patient's plasma and simultaneously, to a sample from a normal control. The length of time required for clot formation in both samples is observed. Thrombin is formed from prothrombin in the presence of adequate calcium, thromboplastin, and the essential tissue coagulation factors. A prolonged PT therefore indicates deficiency in one of the factors, as in liver disease, vitamin K deficiency, or anticoagulation therapy with the drug Coumadin. International Normalized Ratio (INR). Once prothrombin times are determined, they are expressed as an INR value. INR stands for International Normalized Ratio and essentially is the ratio of the prothrombin time measured in the patient divided by a standard prothrombin time value, and multiplied by a constant. An INR value of 1 means normal prothrombin times of approximately 12 seconds; normal blood clotting would be present. INR values greater than 1 indicate that there is an anticoagulant effect. The higher the INR value, the greater the anticoagulant effect. Many patients taking anticoagulants have INR values of 2,3 and even up through 6.

TM

PDF Editor

MPHARMACOLOGY

Cardiovascular)

All of the following conditions are managed by using anticoagulants and anti-platelet agents EXCEPT one. Which one is the EXCEPTION?

Coronary Artery Disease (CAD) Angina Pectoris (Unstable Angina) Myocardial Infarction (Heart Attack) Stroke Hypertension

142 Copyright O 2011-2012 - Dental Decks

(PHARMACOLOGY

Cardiovascular)

Which of the following class of drugs lower blood cholesterol levels by inhibiting a key enzyme in the cholesterol synthesis pathway in the liver?

HPG-CoB Reductase Inhibitors HCG-CoG Reductase Inhibitors HMG-CoA Reductase Inhibitors HCG-CoB Reductase Inhibitors

143 Copyright 2011-2012 - Dental Decks

TM

PDF Editor

Hypertension Anticoagulants such as warfarin (Coumadin) and anti-platelet agents such as aspirin and clopidogrel (Plavix) are used in the conditions listed for the following reasons: Coronary artery disease (CAD): will help prevent threat of myocardial infarction in CAD patients. Angina pectoris (unstable angina): will help prevent thrombus from forming within the coronary arteries. Myocardial infarction (MI): drugs that prevent blood clotting have been shown to prevent the threat of future infarcts. Stroke: will help prevent thrombus from forming thus preventing threat of a cerebral embolism. Note: Unless there are other accompanying cardiovascular problems such as those listed above, anticoagulant drugs are not necessary in the treatment and management of hypertension. These drugs do nothing to lower blood pressure.

IIMG-CoA Reductase Inhibitors


Coronary artery disease (CAD) is a condition of narrowing of the blood vessels of the heart restricting oxygen flow to heart muscle. It has been correlated with the levels of blood cholesterol and triglycerides. If not treated, CAD can lead to myocardial infarction (heart attack). Drugs such as the statins, which lower blood cholesterol arc effective in minimizing the threat of CAD. Currently, six classes of antihy perlipidemic drugs arc available, and each class has its own mechanism for lowering lipid levels. Note: The two major lipoproteins that arc targeted arc VLDLs and LDLs. The primary lipids of VLDLs are the triglycerides, whereas the primary lipids of LDLs arc cholcstcryl esters. Higher levels of LDL increase the risk of cardiovascular disease, whereas higher levels of VLDL increase the risk of pancreatitis. HMG-CoA Reductase inhibitors: the enzyme 3-hydroxy-3-mcthylglutaryI coenzyme A (HMG-CoA) reductase is necessary in the key step to synthesize cholesterol. When the "statin" drugs inhibit this enzyme, cholesterol is not produced in the liver and blood levels decrease. The family of "statin drugs" include atorvastatin (Lipitor), simvastatin (Zocor), fluvastatin (Lescol). lovastatin (Mevacor) pravastatin (Pravachol) and rosuvastatin (Creslor). Fibric acid derivatives: their primary lipoprotein effect is to decrease triglyceride and raise HDL concentrations. They do this by increasing lipoprotein lipase activity, which results in increased catabolism of VLDL. Examples include gemfibrozil (Lopid) and fenofibrate (TriCor). Bile acid sequestrants: these drugs bind to bile acids, bile acid scquestrants increase the diversion of cholesterol to bile acid synthesis, lower intracellular stores of cholesterol, and resull in creased catabolism of LDL by (he liver. Examples include cholestyramine (Questran), and colesevelam (WelChol). Other agents: nicotinic acid (niacin) is believed to act on a hormonc-sensitive lipase; this leads to inhibition of release of free fatty acids from adipose tissue (lipolysis). The inhibition of lipolysis leads to reduced free fatty acid transport to the liver and therefore decreased synthesis of VLDL. Combination products: examples include lovastatin/niacin (Advicor) and ezetimibe/simvastatin (Vytorin). Selective cholesterol absorption inhibitors: ezetimibe (Zetia) is the first agent in a new class of drugs that appear to act on the brush border of intestinal epithelial cells, where it selectively inhibits the absorption of cholesterol from dietary and biliary sources. Important: The "statin" drugs have the capability to increase the breakdown of skeletal muscle thereby releasing muscle protein. If the protein overloads the kidneys, renal failure could result. The erythromycin drugs enhance the capabilities of the "statins" to cause this effect. It is advisable for patients medicated with a "statin" drug not to be given erythromycin products.

TM

PDF Editor

(PHARMACOLOGY

Cardiovascular^

Antihypertensive agents lower blood pressure by reducing total peripheral resistance and by reducing cardiac output through a variety of mechanisms. ACE inhibitors inhibit the conversion of inactive angiotensin I to the angiotensin II, a vasoconstrictor.

The first statement is true; the second statement is false The first statement is false; the second statement is true Both statements are true Both statements are false

144 Copyright 2011-2012 - Dental Decks

(PHARMACOLOGY

CardiovascularJ

Which of the drugs below can prolong the QT interval of cardiac electrical conduction and thus can increase the risk of cardiac arrhythmias?

1 Erythromycin 1 Ibuprofen

Hydrocodone Penicillin VK Azithromycin (Z-Pak)

145 Copyright C 2011-2012 - Dental Decks

TM

PDF Editor

Both statements are true Antihypertensive agents are classified by mechanisms of action: Diuretics: include thiazide and thiazide-like ([NaClJ inhibitors) such as hydrochlorothiazide (HCTZ) and loop diuretics such as furosemide (Lasix) and bumetanide (Bumex). These drugs inhibit sodium reabsorption in renal tubular cells within the kidney to cause excess sodium and urinary excretion resulting in reduced blood volume. Beta-adrenergic receptor blockers (beta-blockers): reduce die volume of cardiac output into the circulation resulting in reduce peripheral pressure. Two types: 1. Cardioselective beta-blockers (beta j-receptor block in heart muscle): examples include atenolol (Tenormin), metoprolol (Lopressor; Toprol XL), acebutolol (Secral), esmolol (Brevihloc) 2. Non-cardioselective (beta/ and beta^) beta-blockers: examples include nadolol (Corgard). propranolol (Inderal), timolol (Betimol), penbutolol (Levatol), and sotalol (Betapace) *** Note: labetalol (Trandate) and carvedilol (Coreg) are non-selective beta-blockers that also block alpha | receptors. Alpha-adrenergic receptor blockers: cause dilation of arterioles and veins and reduce peripheral vacular resistance.Two types: 1. Cardioselective alpha-blockers (alpha j-receptor blockers): examples include doxazosin (cardura), prazosin (Minipress) and terazosin (Hytrin). 2. Non-cardioselective (alpha/ and alpha f) alpha-blockers: examples include phentolamine (Regitine) and phenoxybenzamine (Dibenzyline). Angiotensin-converting enzyme inhibitors (ACE Inhibitors): examples include lisinopril (Prinivil; Zestril). ramipril (Altace) and enalapril (Vasotec): inhibit the conversion of inactive angiotensin I to the angiotensin II, a vasoconstrictor. This results in peripheral vasodilation and secondarily, an increase in urinary volume excretion. Both actions result in reduced blood pressure. Angiotensin II receptor blockers (ARBs): examples include losartan (Cozaar), valsartan (Diovan). and candesartan (Atacand). ARBs block the effects of angiotensin II by blocking the binding of angiotensin II to its receptors. They do not effect bradykinin. Calcium channel blockers examples include Verapamil (Calan), amlodipine (Norvasc), diltiazem (Cardizem), and nifedipine (Procardia): inhibit calcium entry into vascular smooth muscle causing vasodilation of coronary and peripheral blood vessels thus lowering blood pressure.

Erythromycin Erythromycin is one of the drugs confirmed to prolong the QT interval and is accepted as having a risk of causing arrhythmias. The QT interval is measured as the time and distance between the Q point of the QRS complex and the end of the T wave in the ECG tracing. A long QT syndrome was first described in the 1950's as a congenital syndrome involving QT interval prolongation and syncope and sudden death. These congenital long QT syndromes were characterized by a peculiar electrocardiographic appearance of the QRS complex involving a premature atria beat followed by a pause, then a subsequent sinus beat showing marked QT prolongation and deformity. This type of cardiac arrhythmia was originally termed "torsade de pointes" (from thefrench "twisting of the points "). Erythromycin, an antibiotic used to treat bacterial infections, is considered as having a risk of causing torsade de pointes, an unusual adverse reaction for an antibiotic. Clarithromycin, an antibiotic within the erythromycin family also causes prolongation of the QT interval. Azithromycin, another member of the erythromycin family of antibiotics is not associated with prolonging the QT interval.

TM

PDF Editor

(PHARMACOLOGY

Cancer/ChemoJ

Exemestane (Aromasin) and letrozole (Femara) are anticancer drugs classified under what category listed below?

Antibiotics Aromatase inhibitors Antimetabolites Alkylating agents Antimicrotubular

146 Copyright 2011 -2012 - Dental Decks

(PHARMACOLOGY

Cancer/Chemo)

Alkylating agents are most effective in treating all of the following EXCEPT one. Which one is the EXCEPTION?

Chronic leukemias Lymphomas AIDS Myelomas Carcinomas of the breast and ovary

147 Copyright 2011-2012 - Dental Decks

TM

PDF Editor

Aromatase inhibitors Exemestane is an irreversible, steroidal aromatase inactivator. It prevents conversion of androgens to estrogens by tying up the enzyme aromatase. In breast cancers where growth is estrogen dependent, this drug will lower circulating estrogens. Exemestane is used in the treatment of advanced cancer in post menopausal women whose disease has progressed following tamoxifen therapy. Letrozole (Femara) works by a similar mechanism as above. It is used as first line treatment of hormone receptor positive or metastatic breast cancer in postmenopausal women. It is also indicated as an extended adjuvant treatment of early breast cancer in postmenopausal women who have received 5 years of adjuvant tamoxifen therapy.

The alkylating agents contain a diverse group of compounds which all form alkyl bonds to nucleic acids. All of these agents share a similar mechanism of action and mechanism of resistance. The alkylating agents form covalent bonds with nucleic acids, and proteins. The N-7 position of guanine is a common binding site.
Major Alkylating Agents Drug Nitrogen Mustards: Mechlorethamine (Mustargen) Cyclophosphamide (Cytoxan) Chlorambucil (Leukeran) Melphalan (Alkeran) Nitrosoureas: Carmustine {BCNU.BiCNU) Lomustine (CCNU, CeeNU) Semustine (Methyl-CCNU) Busulfan (Myleran) Use Hodgkin's disease and other lymphomas Lymphomas, leukemias, multiple myeloma, neuroblastoma, retinoblastoma, and cancers of the breast and ovary Chronic lymphocytic leukemia Hodgkin's disease and other lymphomas Several types of cancer, including brain cancer Several types of cancer, including brain cancer and Hodgkin's disease Several types of cancer, including brain cancer Chronic myelogenous leukemia

TM

PDF Editor

(PHARMACOLOGY

Cancer/ChemoJ

The following drugs belong to what pharmaceutical class of agents: Darbepoetin alpha Pegfilgrastim Sargramostin

Immune modulators Monoclonal antibodies Colony stimulating factors Interferons Immunosuppressants

148 Copyright 2011 -2012 - Dental Decks

(PHARMACOLOGY

Cancer/Chemo)

Name the eight classes of drugs used in cancer chemotherapy.

149 Copyright 2011-2012-Dental Decks

TM

PDF Editor

Colony stimulating factors These drugs stimulate the production of neutrophils and erythroid progenitor cells in the hematopoietic process. Darbepoetin alpha: induces erythropoiesis by stimulating the division and differentiation of erythroid progenitor cells. It is used to treat anemia associated with chronic renal failure. Pegfilgrastim (Neulasta): stimulates the production, maturation and activation of neutrophils. It is used to decrease the incidence of infection by stimulation of granulocyte production in patients with nonmyeloid malignancies. Sargramostin (Leukine): is used for myeloid reconstitution after autologous bone marrow transplantation.

Listed below are the eight classes of drugs used in chemotherapy: 1. Alkylating agents - these agents alkylate DNA such that it cannot replicate. Popular agents include Cisplatin and Cyclophosphamide. 2. Anthracyclines - these agents destroy DNA such that the cell cannot replicate. Popular agents include Daunorubicin and Doxorubicin. 3. Antibiotics - these antibiotics are not used for antibacterial therapy but were specifically designed for cancer chemotherapy. The agent in this class is Dactinomycin. 4. Antimetabolites - these agents interfere which selected biochemical reactions necessary for cell growth. Popular agents are 5-Fluorouracil (5-FU), 6-Mercaptopurine and Methotrexate. 5. Antimicrotubular - affects the microtubular assembly with cells to inhibit cell mitosis. The popular agent is Paclitaxel (Taxol). 6. Antiestrogen - these agents block the tumors on which estrogen has a stimulatory effect. The popular agent is Tamoxifen (Nolvadex). 7. Vinca Alkaloids - these are mitotic spindle poisons. Examples include Vinblastine and Vincristine. 8. Gonadotropin hormone-releasing antigen - these inhibit gonadotropin secretion, an action which is effective in reducing certain carcinomas. The popular agent is Leuprolide. Note: The anticancer drugs Asparaginase and Interferons do not fall within any category but are used in the treatment of certain cancers. Asparaginase deprives tumor cells of certain amino acids such that protein production is blocked. Interferons boost the immune system. Agents used in treating breast cancer: Antiestrogens: tamoxifen (Nolvadex), fulvestrant (Faslodex), and toremifene (Fareston). Aromatase inhibitors: letrozole (Femara) and exemestane (Aromasin)

TM

PDF Editor

(PHARMACOLOGY

Cancer/ChemoJ

Your patient with a history of breast cancer is undergoing chemotherapy with a drug called cisplatin (Platinol). All of the following are expected adverse effects of cisplatin EXCEPT one. Which one is the EXCEPTION?

Nausea and vomiting Hair loss (alopecia) Blood test abnormalities (low magnesium, low calcium, low potassium) Peptic ulcers Changes within the tissues of the oral cavity such as mucosititis Low white blood cells Low red blood cells (anemia) Xerostomia
150

Copyright O 2011-2012 - Dental Decks

MPHARMACOLOGY

Cancer/ChemoJ

Which is the most likely complication seen with chemotherapy treatment?

Renal failure

1 Alopecia

Peripheral neuropathy Glaucoma

151 Copyright 2011-2012 - Dental Decks

TM

PDF Editor

Peptic ulcers Cisplatin is an anti-cancer ("antineoplastic") chemotherapy drug. It is classified as an "alkylating agent." Remember: Alkylating agents - these agents alkylate DNA such that it cannot replicate. Popular agents include Cisplatin and Cyclophosphamide. Notes about cisplatin side effects (and most chemotherapy drugs): Most people do not experience all of the side effects The side effects are often predictable in terms of their onset and duration The side effects are almost always reversible and will go away after treatment is complete. The side effects and their severity depend on how much of the drug is given. In other words, high doses may produce more severe side effects. The most common side effects of chemotherapy are: Anemia Fatigue
Hair loss

Low WBCs Xerostomia Mucosititis

Increased chance of bruising, bleeding, and infection Nausea and vomiting Other less common side effects include: Appetite and weight changes Kidney and bladder irritation Nerve and muscle problems Sexual and fertility issues because of effects on reproductive Drv and/or discolored skin organs Note: Mucosititis is a common reaction to cancer chemotherapy. It is an inflammation of the mucous membranes. During chemotherapy and radiation therapy, mucosal tissues begin to desquamate and develop into ulcerations. The mucosal integrity is broken and is secondarily infected by oral flora. Palliative treatment is indicated for mucositis. The antineoplastics such as 5-fluorouracil (5FU), methotrexate and doxorubicin are commonly associated with the development of oral mucositis.

Alopecia Alopecia (hair loss) occurs with administration of most chemotherapeutic agents one to two weeks after treatment. Other common side effects include GI upset, increased incidence of infection (especially Candidiasis), and degeneration of lymphatic tissue. Remember: Most chemotherapy drugs have been shown to be teratogenic in humans and should be avoided by pregnant women. Note: Methotrexate may cause ulceration of the oral tissues.

TM

PDF Editor

(PHARMACOLOGY

Cancer/ChemoJ

Antimetabolites are cell cycle-specific drugs acting primarily in the:

G0 phase or resting phase of the cell cycle S phase of the cell cycle G] phase of the cell cycle G2 phase of the cell cycle

152 Copyright 2011 -2012 - Dental Decks

(PHARMACOLOGY

DiureticsJ

The prototype loop diuretic is:

Hydrochlorothiazide (HCTZ) Furosemide (Lasix) Spironolactone (Aldactone) Triamterene (Dyrenium)

153 Copyright S5 20] I-2012 - Dental Decks

TM

PDF Editor

S phase of the cell cvcle *** The S phase of the cell cycle = DNA synthesis Antimetabolites are one of the oldest and most important classes of antineoplastic agents. They attack the cells in the S phase of the reproduction cycle by interfering with the biosynthesis of the purine and pyrimidine bases.
Antimetabolites used as Antineoplastic Agents Drug Folic acid analog: Methotrexate (Amethopterin) Pyrimidine analogs: Fluorouracil (5-FU) Floxuridine (FUDR) Cytosine arabinoside (Cytosar-U) Purine analogs: Mercaptopurine (Purinethol) Thioguanine (6-TG, Tabloid) Use Choriocarcinoma, leukemia in the spinal fluid, osteogenic sarcoma, breast cancer, and head and neck cancer Several types of cancers including colon and head and neck cancers Some cancers involving the liver Treating acute leukemias Treating several types of leukemias Treating acute nonlymphocytic leukemia

Important: Alopecia (liair loss) occurs with the administration of most chemotherapeutic agents one to two weeks after treatment. Other common side effects include GI upset, increased incidence of infection (especially Candidiasis), and degeneration of lymphatic tissue. Remember: Most chemotherapy drugs have been shown to be teratogenic in humans and should be avoided by pregnant women.

Furosemide (Lasix)
Diuretics are used to treat congestive heart failure by relieving edema and symptoms of dyspnea arising from pulmonary congestion. They are also used to treat hypertension, and in the management of edema associated with hepatic or renal disease. Categories of widely used diuretics: Thiazides: inhibit sodium reabsorption in the distal portion of the renal tubule within the kidney causing increased excretion of sodium and water. Prototype agent is hydrochlorothiazide (HCTZ). Other examples include chlorothiazide (Diuril), indapamide (Lozol), and metolazon (Zaroxolyn). Note: These drugs can cause hypokalemia (abnormally low level of potassium in the blood), hyponatremia (abnormally low level of sodium in the blood) and may increase plasma uric acid. Loop diuretics: inhibit reabsorption of sodium and chloride in the ascending Loop of Henle thus causing increased secretion of water, sodium and chloride. Prototype agent is furosemide (Lasix). Other examples include bumetanide (Bumex), ethacrynic acid (Edecrin), and torsemide (Demadex). Note: These drugs can cause hyperuricemia, tinnitus, hearing loss, hyponatremia and excessive fluid loss. Potassium-sparing diuretics: Sodium channel blockers: these agents inhibit sodium reabsorption through sodium channels in renal epithelial cells. This inhibition creates a negative potential in the luminal membranes of principal cells, located in the distal convoluted tubule and collecting duct. Negative potential reduces secretion of potassium and hydrogen ions. Conserve potassium while causing diuresis. Thus, no potassium is lost from the body as is the case with other diuretics such as the thiazides and loops. Triamterene (Dyrenium) and amiloride (Midamor) are examples of this drug group. Aldosterone antagonists: these drugs competitively inhibit the aldosterone receptor. This causes increased amounts of sodium and water to be excreted, while potassium is retained. Spironolactone (Aldactone) and eplerenone (Inspra) are examples. *** Fixed-dose combination therapy: Dyazide is the brand name for the combination of triamterene and hydrochlorothiazide (HCTZ). This product combines the potassium-sparing diuretic with HCTZ for greater efficacy than either one individually. Note: Positive inotropic drugs (drugs that make muscle contract more forcefully) can be used in the acute treatment of heart failure, these include dobutamine, dopamine, inamrinone, and milrinone. These drugs may be given intravenously to stimulate heart contractions and help keep blood circulating. They are only used temporarily because long-term use shortens life.

TM

PDF Editor

MPHARMACOLOGY

Diuretics)

Which of the following is a pharmacologic antagonist of aldosterone in the collecting tubule?

Mannitol (Osmitrol) Glycerin (Glyrol) Spironolactone (Aldactone) Urea (Ureaphil)

154 Copyright 0 2011-2012 - Dental Decks

MPHARMACOLOGY

Hypoglycemics)

All the following drugs are second-generation sulfonylureas EXCEPT one. Which one is the EXCEPTION?

Glyburide (DiaBeta) Metformin (Glucophage) Glipizide (Glucotrol) Glimepiride (Amaryl)

155 Copyright 2011 -2012 - Dental Decks

TM

PDF Editor

Spironolactone (Aldactone) Potassium-sparing diuretics result in increased sodium and decreased potassium concentrations at the end of the distal convoluted tubules. There are two categories of potassium-sparing diuretics: 1. Antagonist of aldosterone in the collecting tubules: Spironolactone (Aldactone) Note: Spironolactone has been found to be effective in the treatment of primary aldosteronism. It may be useful in the treatment of heart failure because hyperaldosteronism is commonly seen in this condition. 2. Block the sodium channels in the collecting tubules: Triamterene (Dyrenium) Amiloride (Midamor) *** The most important toxic effect of potassium-sparing diuretics is hyperkalemia. Osmotic diuretics are highly filtered by the glomerulus and exert a solute-induced diuresis in the proximal tubule. They are used to reduce excess edema associated with neurosurgery or trauma to the CNS. Examples include: mannitol, glycerin, and urea. Remember: Carbonic anhydrase inhibitors (i.e., Acetazolamide) are relatively weak diuretics because of the ability of more distal sites in the loop of Henle to increase their reabsorption of sodium. Note: Acetazolamide is used to prevent and reduce the symptoms of altitude sickness.

Metformin (Glucophage) *** Metformin is classified as a biquanide Antidiabetic agents or oral hypoglycemic agents are drugs used as adjuncts to diet to treat non-insulin dependent diabetes mellitus (type 2 diabetes) that cannot be controlled by diet alone. Sulfonylureas, the first drag group introduced into the U.S. in 1955, close potassium channels in cell membranes, stimulate the beta cells to produce more insulin., and increase the sensitivity of target organs to insulin. The original "first generation" sulfonylureas include tolbutamide (Orinase). tolazamide (Tolinase). and chlorpropamide (Diabinese). These drugs work well in lowering the blood sugar, but they have a major drawback. Because they bind to proteins in the blood, they can be dislodged by other medications that bind to these same proteins. Once dislodged, their activity can increase rapidly and lead to low blood sugars. Second generation sulfonylureas include glipizide (Glucotrol), glyburide (Micronase) and glimepiride (Amatyl). These drugs have an advantage for those who use other medications since they do not bind to carrier proteins in the blood. Because of this, drug interactions that may cause low blood sugars are less likely. Biquanides: metformin (Glucophage) primarily decreases hepatic glucose production. It also has minor effects on insulin sensitivity in both the liver and peripheral tissues. It has no direct effect on the pancreas and therefore does not enhance insulin secretion. Thiazolidinediones: rosigiitazone (Avandia) and pioglitazone (Actos) increase senstivity in the muscle and liver by improving control of glycemic utilization. This in turn reduces circulating insulin levels. Note: Functioning beta-cells are required for these medications to work. Meglitinides: repaglinide (Prandin) and nateglinide (Starlix) lower blood sugar by stimulating the release of insulin from the pancreas in short bursts. Note: Functioning beta-cells are required for these medications to work. a-Glucosidase inhibitors: acarbose (Precose) and miglitol (Glyset) act through inhibition of pancreatic a-amylase and membrane-bound intestinal a-glucoside hydrolase enzymes. This enzyme inhibition delays glucose absorption. These enzymes do not enhance insulin secretion.

TM

PDF Editor

(PHARMACOLOGY

Hypoglycemics)

Insulin regular, also known as Humulin R, is considered a/an:

Short-acting insulin with a duration of 6-8 hours Long-acting insulin with a duration of 20-24 hours Intermediate-acting insulin with a duration of 10-16 hours Rapid-acting insulin with a duration of 3-4 hours

156 Copyright 0 2011-2012 - Dental Decks

(PHARMACOLOGY

Hypoglycemics}

All of the following are effects of insulin EXCEPT one. Which one is the EXCEPTION?

Decreased gluconeogenesis Increased triglyceride storage Decreased protein synthesis Increased glycogen synthesis

157 Copyright 2011 -2012 - Dental Decks

TM

PDF Editor

Short-acting insulin with a duration of 6-8 hours Insulin preparations mimic the activity of endogenous insulin, which is required for the proper utilization of glucose in normal metabolism. They are used in type 1 diabetes and in type 2 diabetes which cannot be controlled completely by the oral antidiabetic drugs or by diet alone. Rapid-acting Insulin aspart (NovoLog) Insulin glulisine (Apidra) Insulin lispro (Humalog) Short-acting Insulin regular (Humulin R) 30-60 min Intermediate-acting NPH (Humulin N) Long-acting Insulin detemir (Levemir) Insulin glargine (Lantus) 1 hour 2 hours No peak No peak 6-24 hours 20-24 hours 2-4 hours 6-10 hours 10-16 hours 2-4 hours 6-8 hours Onset 5-15 min 5-15 min 5-15 Peak 1-3 hours 30-90 min 30-90 min Duration 3-5 hours 3-4 hours 3-4 hours

Important: Hypoglycemia (low blood sugar) is the most serious complication of insulin therapy. Symptoms include: sweating, weakness, confusion, slurred speech and blurred vision. Administration of a concentrated glucose source will relieve mild hypoglycemia.

Decreased protein synthesis *** This is false; insulin increases protein synthesis. Insulin is a pancreatic hormone secreted by the pancreatic beta-cells of the islets of Langerhans and is essential for the metabolism of glucose and for the homeostasis of blood glucose (it reduces blood glucose by increasing the conversion to glycogen and fat). Insulin injection is by subcutaneous administration. Remember: Insulin is required in treating type 1 diabetes mellitus because the beta cells of the pancreas are devoid of insulin. In treating type 2 diabetes mellitus, oral hypoglycemics can often be used because the beta cells are able to secrete insulin, although in a more sluggish manner.

TM

PDF Editor

MPHARMACOLOGY

Hypoglycemics J

Humulin 70/30 is the brand name for a mixture of insulins containing: 70% insulin NPH and 30% regular insulin injection. The advantage of this insulin mixture is which of the following?

Fast onset, short duration


1

Slow onset, short duration Fast onset, long duration Slow onset, long duration

158 Copyright 2011 -2012 - Dental Decks

(PHARMACOLOGY

MiscJ

Which of the following is a high molecular weight heteropolysaccharide that inactivates thrombin and other coagulation factors and thus prevents blood clotting?

Prothrombin Fibrin Heparin Plasmin

159 Copyright 2011-2012 -Dental Decks

TM

PDF Editor

Fast onset, long duration Humulin 70/30 mixture is a popular form of insulin that many diabetic patients take. Its advantage is that after a single injection, the regular insulin component (30%) provides a fast onset of blood sugar control beginning one-half hour after injection and the insulin NPH component (70%) starts acting within a couple of hours to provide a long duration of blood sugar control. Note: Using either agent alone would not provide both the fast onset of insulin action along with the long duration.

Mixtures Humulin 70/30 Novolin 70/30 Novolog 70/30 Humulin 50/50 Humalog 75/25

Onset 30 min 30 min 10-20 min 30 min 15 min

Peak 2-4 hours 2-12 hours 1-4 hours 2-5 hours

Duration 14-24 hours Up to 24 hours Up to 24 hours 18-24 hours

30 min - 2hours 16-20 hours

Premixed insulins are a combination of specific proportions of intermediate-acting and rapid-acting or short-acting insulin in one bottle or insulin pen (the numbers following the brand name indicate the percentage of each type of insulin).

Heparin Heparin is contained within mast cells and basophils. These cells occur in connective tissue and in extracellular spaces near blood vessels. Remember: Heparin not only neutralizes tissue thromboplastin, but also blocks thromboplastin generation. 1. The administration of heparin will result in an increase in bleeding time due to a potentiation of antithrombin III thereby inactivating thrombin. This prevents the conversion of fibrinogen to fibrin. 2. It is used for prophylaxis and treatment of thromboembolic disorders.

TM

PDF Editor

(PHARMACOLOGY

Misc!)

(T

The following drugs are classified as what type of drugs?

Etanercept (Enbrel) ' Infliximab (Remicade) Adalimumab (Humira)

160 Copyright 20! I-2012 - Dental Decks

(PHARMACOLOGY

Misc.)

Triazolam (Halcion), a pre-operative sedative in dentistry, is metabolized in the liver by the P-450 isoform CYP3A4 enzyme. Drugs which inhibit the actions of CYP 3A4 would affect triazolam in which way?

Cause an increase in serum levels of triazolam Cause a decrease in serum levels of triazolam Cause no change in serum levels of triazolam

161 Copyright 2011-2012 - Dental Decks

TM

PDF Editor

Anti-rheumatic aeents There are currently three tumor necrosis factor alpha (TNF) inhibitors FDA approved for the treatment of RA (listed in order of their approval for RA): Etanercept (Enbrel) is used to reduce the signs and symptoms of active rheumatoid arthritis in patients who have had inadequate response to one or more disease-modifying anti-rheumatic drugs (DMARDs). It is a recombinant DNA-derived protein which binds to tumor necrosis factor alpha (TNF). Infliximab (Remicade) is used to treat Crohn's disease and rheumatoid arthritis. Infliximab is a chimeric monoclonal antibody that binds to tumor necrosis factor alpha (TNF) thereby reducing the inflammatory actions of this endogenous compound. Adalimumab (Humira) is a fully human anti-TNF monoclonal antibody with high specificity for TNF. Adalimumab binds specifically to TNF and blocks its interaction with the p55 and p75 cell surface TNF receptors, thereby interfering with endogenous TNF activity. Adalimumab binds to both soluble as well as cell bound TNF. It is administered by subcutaneous injection every two weeks but can be increased to weekly, if needed. Adalimumab is effective in RA, Psoriatic arthritis, and ankylosing spondylitis, and Crohn's disease. Note: Tumor necrosis factor alpha (TNF) is a pro-inflammatory cytokine produced by macrophages and lymphocytes. It is found in large quantities in the rheumatoid joint and is produced locally in the joint by synovial macrophages and lymphocytes infiltrating the joint synovium. TNF is one of the critical cytokines that mediate joint damage and destruction due to its activities on many cells in the joint as well as effects on other organs and body systems.

Cause an increase in serum levels of triazolam Triazolam is known to interact with drugs that inhibit its metabolism via the CYP 3A4 enzyme. Drugs that inhibit the metabolic pathway may have a profound effect on the clearance of triazolam. The resultant effects would be an increase in serum concentrations with an associated unexpected increase in the actions of triazolam. Consequently, triazolam should be avoided in patients receiving very potent inhibitors of CYP 3A4. Antifungal agents (itraconazole, ketoconazole, fluconazole, miconazole, voriconazole) can significantly elevate the serum levels of triazolam resulting in toxicity with therapeutic doses. These antifungal agents inhibit the CYP 3A4 isoform responsible for hepatic metabolism of triazolam. Thus the normal metabolism of triazolam is inhibited. Do not administer triazolam to patients taking any of these antifungal agents.

TM

PDF Editor

(PHARMACOLOGY

Misci)

When writing a prescription, q.i.d. is the abbreviation for:

Twice a day

Every 4 hours Three times a day Four times a day

162 Copyright 2011-2012 - Dental Decks

(PHARMACOLOGY

Misc.)

All of the following drugs are useful in treating what medical condition?

Pilocarpine (Isopto-Carpine) Latanoprost (Xalatan) Betaxolol (Betoptic) Bimatoprost (Lumigan)

163 Copyright 2011-2012 - Dental Decks

TM

PDF Editor

Four times a day

Other popular and common abbreviations used when writing prescriptions: b.i.d. = twice a day t.i.d. = three times a day q.l2.h. = every 12 hours q.4.h. = every 4 hours stat. = immediately h.s. = at bedtime p.r.n. = as needed a.c. = before meals p.c. = after meals Sig. = Label Note: Always document prescriptions that are given to a patient in the patient's chart, along with the date they were written and any specific instructions for patient use. Remember: 1 grain = 65 mg 1 ounce = 30 g or 30 mL

Glaucoma

Glaucoma is characterized by an increase in intraocular pressure. It is caused by poor drainage of the aqueous humor (fluid in the eye) and can cause blindness. Pilocarpine (Isopto-Carpine): eye drops in the eyes causes papillary constriction thus allowing for drainage of the aqueous humor to reduce pressure Latanoprost (Xalatan): a prostaglandin analog; eye drops in the eye reduces intraocular pressure by increasing the outflow of the aqueous humor Betaxolol (Betoptic): a beta-blocker; eye drops in the eye reduces intraocular pressure by reducing the production of aqueous humor Bimatoprost (Lumigan): same action as latanoprost (Xalatan)

TM

PDF Editor

QPHARMACOLOGY
f ' Amitriptyline (Elavil)
1 Diphenhydramine (Benadryl)

MiscJ

The following drugs are noted for causing what prominent oral side effect?

Atropine Diazepam (Valium)

164 Copyright C 2011-2012 - Dental Decks

QPHARMACOLOGY
Levodopa is used in the treatment of Parkinson's disease to replenish the brain's supply of serotonin, the neurotransmitter that is deficient in this disorder. Administering carbidopa in combination with levodopa reduces the required dose of levodopa by about 75%.

Misc!)

The first statement is true; the second statement is false The first statement is false; the second statement is true Both statements are true Both statements are false

165 Copyright 2011-2012 - Dental Decks

TM

PDF Editor

Xerostomia (dry mouth) Xerostomia can be caused by certain drug classes that inhibit the production and secretion of saliva. Amitriptyline (Elavil) is representative of the tricyclic antidepressants, a class of drugs that causes significant xerostomia. They probably work through an anticholinergic action. Diphenhydramine (Benadryl) is representative of the sedating-type antihistamines, a class of drugs that causes significant xerostomia. They probably work through an anticholinergic action. Atropine is a powerful anticholinergic which blocks the production of saliva in the salivary glands. Other anticholinergics will have a similar action. Diazepam (Valium) is representative of the benzodiazepine tranquilizers. These drugs have moderate anticholinergic actions to reduce the outflow of saliva. Note: The xerostomia actions produced by these classes of drugs are reversible with normal salivary flow regained after discontinuance of the drug.

The first statement is false; the second statement is true


*** Levodopa is used in the treatment of Parkinson's disease to replenish the brain's supply of dopamine, the neurotransmitter that is deficient in this disorder. Parkinson's disease (PD) results from a relative excess of cholinergic activity and a deficiency of dopaminergic activity in the basal ganglia. It is a chronic, debilitating disease with no known cure. Drug treatment for PD has centered on increasing the availability of dopamine in the CNS, inhibiting the effects of acetylcholine, and attempting to prevent further cell membrane damage through neuroprotective trials. Note: The D2-receptor subtype is the primary modulator of both clinical improvement and adverse reactions such as dystonia and hallucinations. Levodopa has been the single most important drug in the treatment of PD. Administering carbidopa in combination with levodopa (Sinemet) reduces the required dose of levodopa by about 75%. When levodopa is given alone, much of the dose is metabolized before the drug reaches the brain. Therefore, large doses are required, and these are apt to cause unwanted side-effects. Carbidopa inhibits the peripheral decarboxylation of levodopa. This action simultaneously reduces the likelihood of peripheral side effects and allows more levodopa to reach the brain. Since carbidopa does not cross the blood-brain barrier, the levodopa in the brain is converted there to dopamine. Thus, co-administration of carbidopa plus levodopa in the form of Sinemet allows a significant reduction of levodopa dosage without reducing the desired effects. Other antiparkinson agents: Monoamine oxidase B (MAO-B) inhibitors: selegiline (Eldepyrl) and rasagiline (Azilect). These drugs irreversibly inhibit the enzyme MAO-B, which is responsible for the oxidative deamination of dopamine in the brain. This causes dopamine to accumulate in surviving nerve cells and reduces the symptoms of PD. Glutamate antagonist (antiviral): amantadine (Symmetrel) appears to potentiate dopaminergic responses. Dopamine agonists: bromocriptine, pcrgolide, apomorphine (Apokyn), pramipexole (Mirapex), and ropinirole (Requip). These drugs are direct dopamine receptor agonists. Anticholinergic agents (antimuscarinic drugs): benztropine (Cogentin) and trihexyphenidyl (Tasmar). These drugs suppress central cholinergic activity and may inhibit reuptake and storage of dopamine in the CNS, thus prolonging the action of dopamine. Catechol-O-methyl transferase (COMT) inhibitors: tolcapone (Tasmar) and entacapone (Comtan). These drugs are inhibitors of COMT, another enzyme that metabolizes dopamine. Certain antidepressants and antihistamines such as diphenhydramine that have antimuscarinic actions may be given in the early stages of disease.

TM

PDF Editor

(PHARMACOLOGY

Misc!)

Which antiarrhythmic agent is effective only on the ventricle and is often administered IV to treat life-threatening ventricular arrhythmias?

Quinidine (Quinidex) Lidocaine (Xylocaine) Flecainide (Tambocor) Propafenone (Rythmol) None of the above

166 Copyright 0 2011-20I2 - Dental Decks

(PHARMACOLOGY

Misc.)

Write up a sample prescription, illustrating all of the pertinent information that needs to be included when writing a prescription for a patient.

167 Copyright 2011-2012 - Dental Decks

TM

PDF Editor

Lidocaine When lidocaine is used IV to treat ventricular arrhythmias, it acts on the fibrillating ventricles to decrease the cardiac excitability and spares the atria. It can effectively reverse a life-threatening situation. Quinidine is considered as the prototype antiarrhythmic agent and is used primarily to treat atrial fibrillation. It is not effective in treating life-threatening ventricular fibrillation. Flecainide (Tambocor) is potent antiarrhythmic agent, effective in a wide range of ventricular and atrial arrhythmias and tachycardias. Propafenone (Rythmol) is used to treat both ventricular arrhythmias and supraventricular tachycardias.

Dr. John Doe, DDS 11 Any Street Any City, Any State 00000 (000)555-1212 Patient's Name: Patient's Address:

License # Federal Drug Registry #

Age: Date:

Rx: Amoxicillin 500 mg. tablets Disp. Four (4) tabs Sig: Take 4 tabs (2000 mg) 30-60 minutes prior to dental appointment

Signature: Substitution permissible Substitution not permissible Number of refills Notes: Superscription - Patient's name, address, age, date Inscription - Name of drug and the strength of the drug (i.e., 500 mg tablets) Subscription - Directions to the pharmacist (dosage form and amount to be given Disp.) Transcription or signa - Directions to the patient (Sig.) Signature - Signature of person prescribing medication must appear

TM

PDF Editor

PHARMACOLOGY

MiscJ

All of the following drugs are useful for treating what common medical condition?

1 Prednisone

' Gold injections Methotrexate Nabumetone (Relafen) Piroxicam (Feldene)

168 Copyright 2011-2012 - Dental Decks

PHARMACOLOGY

isc.J Misc

Which TWO gastrointestinal drugs listed below reduce the formation of stomach acid by inhibiting the proton pump of the stomach parietal cells?

Ranitidine (Zantac) Omeprazole (Prilosec) Cimetidine (Tagamet) Famotidine (Pepcid) Lansoprazole (Prevacid)

169 Copyright 2011-2012-Dental Decks

TM

PDF Editor

Rheumatoid arthritis Rheumatoid arthritis (RA) is a chronic inflammatory disease of joints that results in joint pain, swelling, and destruction. RA is characterized by chronic inflammation of the synovium, which lines the joint. With disease progression, there is the accumulation of prostaglandins, leukotrienes and other mediators in the inflammatory changes and tissue destruction in the synovial lining. How these drugs work in treating rheumatoid arthritis: Prednisone: decreases the inflammatory response Gold injections: may decrease prostaglandin production Methotrexate: unknown, but may affect immune function Nabumetone (Relafen): an NSAID that inhibits prostaglandin synthesis Piroxicam (Feldene): an NSAID that inhibits prostaglandin synthesis Note: All of the above drugs except gold injections are also useful in the treatment of osteoarthritis (OA). OA is characterized by progressive loss of articular cartilage. This may be the result of excessive loads on the joint or other factors. Agents useful in treating O A provide an analgesic and anti-inflammatory action to reduce pain within the joint.

Lansoprazole (Prevacid) Omeprazole (Prilosec) Hydrochloric acid (HCl) is produced by the parietal cells of the stomach through a pump within each cell which pumps protons (H*) into the stomach contents. The pump is called the HfK' ATPase pump. HCl is used for food digestion but an abundancy can cause heart burn and acid indigestion. Omeprazole and lansoprazole inhibit the pump such that no protons are pumped into the stomach contents and thus no HCl is produced. These two drugs are classified as proton-pump inhibitors. Stomach acid can also be reduced by inhibiting the effects of histamine in the stomach at the histamine type-2 receptors (H2 receptors). Ordinarily, histamine stimulates the gastric parietal cells to produce hydrochloric acid. Ranitidine, cimetidine and famotidine block the effects of histamine by blocking at the H2-receptors. These three drugs are classified as H2-receptor blockers. Both the proton-pump inhibitors and H2-blockers are used to treat heartburn, indigestion, sour stomach, active duodenal ulcer disease and gastroesophageal reflux disease (GERD). Remember: Antacids neutralize excess stomach acid by a chemical reaction. Antacids include aluminum hydroxide (Amphogel), bismuth subsalicylate (Pepto-Bismol), calcium carbonate (Turns) magnesium hydroxide (Maalox) and sodium bicarbonate.

TM

PDF Editor

(PHARMACOLOGY

MiscJ

Which TWO drugs listed below inhibit blood clotting by affecting the coagulation pathway to prevent fibrin formation?

1 Heparin 1 Vitamin K

Aspirin Clopidogrel (Plavix) Warfarin (Coumadin)

170 Copyright 2011-2012 - Dental Decks

(PHARMACOLOGY

Misc.)

What serious dental effect is associated with the following drugs: zolendronic acid (Zometa), pamidronate (Aredia), risedronate (Actonel), ibandronate (Boniva) and alendronate (Fosamax)?

Mucositis Osteonecrosis of the jaw bone Angular cheilitis Oral yeast infection Xerostomia

171 Copyright 2011-2012 - Dental Decks

TM

PDF Editor

Heparin Warfarin (Coumadin) Heparin inactivates thrombin and prevents the conversion of fibrinogen to fibrin (blood clot); warfarin (Coumadin) interferes with the hepatic synthesis of vitamin-K dependent coagulation factors II, VII, IX and X resulting in the inability of the coagulation pathway to form fibrin (blood clot). Vitamin K is a group of fat soluble vitamins that are essential for the synthesis of coagulation factors II, VII, IX and X, and prothrombin in the liver. Vitamin K will enhance blood clotting rather than inhibit blood clotting. Aspirin inhibits blood clotting by inhibiting platelet aggregation in an irreversible manner. Inhibition of platelet aggregation prevents activation of the coagulation pathway; thus no fibrin (clot) is formed. Aspirin does not affect the coagulation pathway. Discontinuation of aspirin for 5 to 7 days allows for normal clotting time to reappear due to the synthesis of new platelets. Clopidogrel (Plavix) inhibits blood clotting by inhibiting platelet aggregation in an irreversible manner. Thus the effects on blood clotting are the same as aspirin. Clopidogrel (Plavix) does not cause gastric ulcers like aspirin does and is the antiplatelet agent of choice in patients with history of ulcers. Note: Clopidogrel, aspirin, abciximab (ReoPro), and anticogulants (i.e., heparin) are used to lessen the chance of heart attack in people who need percutaneous coronary intervention (PCI), a procedure to open blocked arteries of the heart.

Osteonecrosis of the jaw bone Zoledronic acid (Zometa), pamidronate (Aredia), risedronate (Actonel), ibandronate (Boniva), and alendronate (Fosamax) are members of the bisphosphonate class of drugs used to treat and manage Pagefs disease, osteoporosis and to prevent hypercalcemia of malignancy. Bisphoshonate therapy has been associated with osteonecrosis, primarily of the jaw; this has been observed mostly in cancer patients, but also in patients with postmenopausal osteoporosis and other diagnoses. Risk factors include a diagnosis of cancer, with concomitant chemotherapy, radiotherapy or corticosteroids; anemia, coagulopathy, infection or pre-existing dental disease. Symptoms included nonhealing extraction socket or an exposed jawbone. There is no data addressing whether discontinuation of therapy reduces the risk of developing osteonecrosis. However, as a precautionary measure, dental exams and preventative dentistry should be performed prior to placing patients with risk factors on chronic bisphosphonate therapy. Important: Bisphosphonates have an affinity for hydroxyapatite crystals in bone and act as antiresorptive agents. Their primary mechanism of action involves inhibition of osteoclastic bone resorption. Adverse effects (besides osteonecrosis of the jaw bone) include GI symptoms and esophageal erosions. Other agents that have an effect on bone: SERMs (Selective estrogen receptor modulators): Raloxifene (Evista) reduces resorption of bone and decreases overall bone turnover. It is used for treating osteoporosis. Hormones: Calcitonin: is a naturally occurring hormone that is produced by parafollicular C-cells in the thyroid gland. It is known to block bone resorption through its potent inhibitory effects on osteoclasts. Parathyroid hormone: PTH is the primary regulator of calcium and phosphate metabolism in bone and kidney. Physiologic actions include regulation of bone metabolism, renal tubular reabsorption of calcium and phosphate, and intestinal calcium absorption. Once-daily administration of PTH (teriparatide fForteoj) stimulates new bone formation via preferential stimulation of osteoblastic activity over osteoclastic activity. Note: PTH also acts on the kidneys by reducing renal clearance of calcium which increases plasma calcium. It also stimulates the production of the active form of vitamin D in the kidney. Vitamin D: helps to ensure that the body absorbs and retains calcium and phosphorus, which are critical for building bone. Note: 1,25-dihydroxycholecalciferol is the biologically active form of vitamin D (Cholecalciferoi)

TM

PDF Editor

(PHARMACOLOGY

MiscJ

Which substance is an enzyme formed in the kidney and released into the bloodstream where it has an important role in the formation of angiotensin?

Plasmin Lysozyme Renin Heparin

172 Copyright 2011-2012 - Dental Decks

(PHARMACOLOGY

Misc.)

Disulfiram (Antabuse) is used in the management of:

Nicotine abuse Ethanol abuse Opioid abuse NSAID abuse

173 Copyright 2011-2012 - Dental Decks

TM

PDF Editor

Renin is proteolytic enzyme produced by and stored in the juxtaglomerular apparatus that surrounds each arteriole as it enters a glomerulus. Renin acts on the precursor substance angiotensinogen, which is manufactured by the liver and is present in the blood. Renin converts angiotensinogen to angiotensin I. In turn, angiotensin I is converted to angiotensin II by a converting enzyme associated with the walls of capillaries, particularly in the lungs. Important: "The converting enzyme that converts angiotensin I to angiotensin II is known as angiotensin converting enzyme or ACE." Angiotensin II is a potent vasopressor. It not only increases total peripheral resistance but. by stimulating aldosterone release, leads to an increase in plasma volume, venous return, stroke volume, and ultimately an increase in cardiac output. Note: Aliskiren (Tekturna) is a renin inhibitor used to treat hypertension. This is the first of a new class of drugs and little information is available. It is used alone or in combination with other agents for the treatment of hypertension.

Ethanolabuse Disulfiram is not a cure for alcoholism but is a deterrent to ethanol consumption. Disulfiram is an antioxidant that interferes with the hepatic oxidation of the acetaldehyde metabolized from alcohol. Specifically it inhibits aldehyde dehydrogenase, a mitochondrial enzyme found in the liver. Even the ingestion of small amounts of ethanol results in high concentrations of acetaldehyde in the body. The unpleasant reaction that occurs (called the Disulfiram-Ethanol Reaction or DER) consists of a throbbing headache, dyspnea, throbbing in the neck, nausea, copious vomiting, thirst, tachycardia and hypotension. Note: Metronidazole, also inhibits aldehyde dehydrogenase. Ethanol is a sedative-hypnotic drug and is the most important alcohol of pharmacologic interest. Its abuse is responsible for many socioeconomic problems. Drugs that are synergistic with ethanol include diazepam, meperidine, pentobarbital and chlorpromazine. When combined with alcohol these drugs could cause fatal oversedation. Remember: Synergism refers to the combined action of two or more drugs that is greater than that achieved with a single drug.

TM

PDF Editor

(PHARMACOLOGY

Misc!)

Antacids do not prevent the overproduction of acid. Antacids do neutralize the acid once it is in the stomach.

The first statement is true; the second statement is false The first statement is false; the second statement is true Both statements are true Both statements are false

174 Copyright 20! 1-2012 - Dental Decks

(PHARMACOLOGY

Misc.)

Inhaled ammonia is the drug of choice for acting against:

Anaphylaxis Heart attack Syncope Urticaria

175 Copyright 2011-2012 - Dental Decks

TM

PDF Editor

Both statements are true Gastric antacids are drugs that directly neutralize the gastric acid (HCL) secreted in the stomach. Antacid therapy is directed at decreasing the concentration and total load of gastric acid. Some common over-the-counter antacid products: Sodium bicarbonate products: - Alka-Seltzer Calcium carbonate products: - Amitone - Turns Aluminum hydroxide products: - Alterna GEL - Amphojel Magnesium hydroxide products: - Milk of magnesia Bismuth salt products: - Pepto-Bismol Magnesium and aluminum products: - Maalox - Mylanta *** Aluminum hydroxide is the most potent of these but has less neutralizing capacity than calcium carbonate or sodium bicarbonate. Note: Dyspepsia means an impairment of the power or function of digestion.

Svncope Inhaled ammonia irritates trigeminal nerve sensory endings, with a resulting reflex stimulation of medullary respiratory and vasomotor centers. An aromatic ammonia vaporole is crushed between the fingers and held near the patient's nose. Note: The administration of oxygen will aid in combating tissue anoxia. The symptoms of syncope include beads of sweat on the upper lip, a weak thready pulse, cold clammy skin, pallor and a dizzy feeling. The loss of normal vasomotor tonus produces pooling of blood peripherally so that the normal blood volume becomes insufficient. Placing the patient in a supine position and elevating the feet gives the patient a transfusion of whole blood by utilizing the forces of gravity. Note: The head should not be more than about 10 degrees lower than the rest of the body. Types of syncope: Vasovagal Neurogenic nic > Treat with high-flowing 100% oxygen Orthostati tic ) Hyperventilation syndrome - Oxygen is not indicated

Note: 100% oxygen is contraindicated for a person who suffers from chronic obstructive pulmonary disease (COPD).

TM

PDF Editor

(PHARMACOLOGY

MiscJ

Ethyl alcohol (ethanol) causes a well-marked diuresis by inhibiting the production of which hormone?

> Growth Hormone


1 Insulin 1 Antidiuretic Hormone (ADH)

Epinephrine

176 Copyright 2011-2012 - Dental Decks

(PHARMACOLOGY

Misc.)

Growth hormone is also called:

Dopamine Aldosterone Vasopressin Somatotropin

177 Copyright 2011 -2012 - Dental Decks

TM

PDF Editor

Antidiuretic Hormone (ADH) also called vasopressin ADH is a hormone that decreases the production of urine by increasing the reabsorption of water by the renal tubules. Without ADH, there would be extreme loss of water into the urine. Ethanol (ethyl alcohol) inhibits the production of ADH. Remember: Ethyl alcohol dilates blood vessels of the skin, depresses the CNS, and in blood levels in excess of 400 mg % usually results in coma and death. Alcohol acts primarily on the nerve cells within the brain. Alcohol interferes with communication between nerve cells and all other cells, suppressing the activities of excitatory nerve pathways and increasing the activities of inhibitory nerve pathways. Note: Recent evidence has shown that frequent ingestion of moderate amounts of alcohol in any form (beer, wine, distilled spirits) will reduce the risk of heart disease, particularly in men. The order in which alcohol affects the various brain centers is as follows: Cerebral cortex Limbic system Cerebellum Hypothalamus and pituitary gland Medulla (brain stem)

Somatotropin Growth hormone (GH) is produced by, and secreted from the anterior pituitary gland. Basic Metabolic effects of growth hormone: Increased rate of protein synthesis in all cells of the body Decreased rate of carbohydrate utilization throughout the body Increased mobilization of fats and use of fat for energy Human growth hormone is prepared commercially and used as replacement therapy in patients with growth hormone deficiency. The commercial preparation is prepared as the purified polypeptide hormone of recombinant DNA origin with the same amino acid sequence as that produced by the pituitary gland. Human growth hormone is indicated in children for the treatment of growth failure due to lack of adequate endogenous growth hormone secretion. It has been used in adults who have a growth hormone deficiency as a result of pituitary disease. Note: Human growth hormone is administered as subcutaneous injection or intramuscular injection, usually three times per week.

TM

PDF Editor

(PHARMACOLOGY

Misc!)

All of the following are useful in the treatment of what medical condition?

1 Colchicine

Indomethacin Sulfinpyrazone Probenecid Allopurinol

178 Copyright 2011-20I2 - Dental Decks

MPHARMACOLOGY

Misc.)

All of the following are catecholamines EXCEPT one. Which one is the EXCEPTION?

Epinephrine Norepinephrine Chlorpromazine Isoproterenol Dopamine Dobutamine

179 Copyright 2011-2012 - Dental Decks

TM

PDF Editor

The therapeutic management of gout involves three different aspects of the disease: 1. Reducing the inflammation during acute attacks: The drug of choice is colchicine. Colchicine impairs leukocytic migration to inflamed areas and disrupts urate deposition and the subsequent inflammatory response. It is most effective when initiated 12 to 36 hours after symptoms begin. Notes: 1. Colchicine should never be given IM or subcutaneously (it causes tissue irritation). 2. Colchicine can severely damage the liver and kidney; long-term therapy may cause bone marrow depression. Remember: NSAIDs are also important for the treatment of acute gouty arthritis. Indomethacin (may cause renal damage or bone marrow depression) is most commonly used. 2. Decreasing uric acid production: Allopurinol (Zyloprim) is the drug of choice in the management of chronic gout. It inhibits xanthine oxidase, an enzyme that converts hypoxanthine to xanthine, and xanthine to uric acid. May cause GI disturbances. 3. Enhancing uric acid clearance: Uricosuric agents include probenecid (Benemid) and sulfinpyrazone (Anturane). These agents act primarily in the kidney (specifically, the proximal convoluted tubules) and inhibit the secretion of other weak acids (i.e., penicillin), in addition to inhibiting the reabsorption of uric acid. Remember: Normally penicillins and cephalosporins have to be given in high and frequent doses due to their high rate of elimination by the kidneys. Their excretion is slowed by giving probenecid.

Chlopromazine *** Chlorpromazine is a dopamine antagonist and antipsychotic agent. Catecholamines are any one of a group of sympathomimetic compounds composed of a catechol molecule and the aliphatic portion of an amine. Some catecholamines are produced naturally by the body (called endogenous) and function as key neurological chemicals (i.e., epinephrine, norepinephrine and dopamine). Note: Epinephrine, Norepinephrine and Isoproterenol are considered to be direct-acting catecholamines.
Sympathomimetics Drug (Catecholamine) Epinephrine Norepinephrine Isoproterenol Dopamine Dobutamine Drug (Not Catecholamine) Amphetamine, phenmerrazine Ephedrine Phenylephrine Albuterol, melaproterenol, terbutaline Oxymetazoline, xylometazoline Narcolepsy, obesity, attention deficit disorder Urinary incontinence, to cause vasoconstriction, decongestion To cause mydriasis, vasoconstriction, decongestion Asthma, premature labor To cause nasal decongestion (long acting) Clinical Applications Anaphylaxis, glaucoma, asthma, to cause vasoconstriction To cause vasoconstriction in hypotension Asthma Shock, heart failure Shock, heart failure Immediate precursor of NE Comments Stimulates the myocardium Stimulates the myocardium

TM

PDF Editor

(PHARMACOLOGY

MiscJ

All of the following are skeletal muscle spasmolytic drugs EXCEPT one. Which one is the EXCEPTION?

Methocarbamol (Robaxin) Cyclobenzaprine (Flexeril) Baclofen (Lioresal) Succinylcholine (Anectine) Carisoprodol (Soma)

180 Copyright 2011-2012 - Dental Decks

(PHARMACOLOGY

Misc.)

All of the following drugs are used in the treatment of Parkinson's disease EXCEPT one. Which one is the EXCEPTION?

Levodopa (Dopar) Bromocriptine (Parlodel) Pergolide (Permax) Haloperidol (Haldol) Selegiline (Eldepryl) Amantadine (Symmetrel)

181 Copyright 2011-2012 - Dental Decks

TM

PDF Editor

Succinylcholine (Anectine) Succinylcholine is the prototype depolarizing neuromuscular blocking drug. Spasmolytic drugs (skeletal muscle relaxants) are agents that relieve muscle spasms without paralysis. They act in the CNS or in the skeletal muscle cell rather than at the neuromuscular end plate. These drugs are used in certain chronic diseases of the CNS (i.e., multiple sclerosis, cerebral palsy, cerebrovascular accidents) that are associated with painful muscle spasms. By reducing the spasms there is a reduction in pain and improved mobility for the patient. Drugs used for chronic muscle spasm: Baclofen (Lioresal): is a derivative of GABA; its site of action in reducing muscle spasms is the spinal cord. It stimulates GABA B receptors that are linked to the G protein, G], resulting in an increase in K' conductance and a decrease in Ca:* conductance. Used is the treatment of multiple sclerosis and other spinal cord diseases. All of the other skeletal muscle relaxants do not bind to the GABABreceptors: Diazepam (Valium) and tizanidine (Zanafiex) also act in the spinal cord and are effective muscle relaxants. Carisoprodol (Soma): is used in the treatment of muscle spasms and pain associated with acute temporomandibular joint pain. Its precise mechanism of action is not clear but many effects have been ascribed to its central depressive action. Drugs used for acute muscle spasm: Cyclobenzaprine (Flexeril): relieves muscle spasm through a central action, possibly at the brain stem level. It is used to relieve acute, painful musculoskeletal conditions. It is not effective for muscle spasm secondary to cerebral or spinal cord disease. Methocarbamol (Robaxin): is a centrally acting muscle relaxant that is used to relieve acute, painful musculoskeletal conditions and in the management of tetanus. Note: Quinine is widely used for the effective relief of nocturnal leg cramps.

Haloperidol (Haldol) *** Haloperidol is used to treat psychotic syndromes. Parkinson's disease is a slowly progressing, degenerative disorder of the nervous system. It has several distinguishing characteristics: tremor (shaking) when at rest, sluggish initiation of movements and muscle rigidity. In Parkinson's disease, nerve cells in the basal ganglia degenerate, resulting in lower production of dopamine. It may be treated (but not cured) with a wide variety of drugs: Levodopa (in combination with carbidopa) is the precursor of dopamine. It is the main treatment for Parkinson's disease. It is given with carbidopa to increase effectiveness and reduce side effects. Bromocriptine or pergolide are dopamine agonists which are often given in addition to levodopa early in the treatment to enhance levodopa's action, or may be given later when levodopa's side effects become more of a problem. Selegiline is a selective inhibitor of MAO Type B, the enzyme that is responsible for the oxidative deamination of dopamine in the brain. It is used as an adjunct to levodopa. Amantadine appears to potentiate dopaminergic responses. Antiparkinsonian actions are unrelated to the antiviral effects. It is used in the early stages for mild disease. Anticholinergic drugs (benztropine and trihexyphenidyl, certain antidepressants and antihistamines such as diphenhydramine) may be given without levodopa in the early stages of disease, with levodopa in later stages. 1. Dental implications of Anti-Parkinson's agents: Anti-Parkinson's agents (mainly levodopa, the direct dopamine agonists and the COMT inhibitors) have been associated with orthostatic hypotension. Xerostomia (caused by anticholinergics and MAO-B inhibitors). Schedule appointment times based on when the patient is feeling the best. Dyskinesia (abnormal muscle movements) caused by some of the drugs (mainly levodopa, the direct dopamine agonists, and the COMT inhibitors) can present a challenge when trying to perform dental treatment.

TM

PDF Editor

(PHARMACOLOGY

MiscJ

All of the following are CNS stimulants EXCEPT one. Which one is the EXCEPTION?

Caffeine Doxapram (Dopram) Phenobarbital Methylphenidate Phendimetrazine Strychnine

182 Copyright 2011-20I2 - Dental Decks

(PHARMACOLOGY

Misc!)

The symptoms of chronic caffeine consumption can include all of the following EXCEPT one. Which one is the EXCEPTION?

Feelings of anxiety and nervousness Sleep disruption Coughing Irritability Diuresis Stomach complaints Palpitations and arrhythmias
183 Copyright 2011-2012 - Dental Decks

TM

PDF Editor

Phenobarbital *** Phenobarbital is a barbiturate (sedative-hypnotic). CNS stimulants are a heterogenous group of compounds that produce various degrees of stimulation. Analeptic is a term that refers to a CNS stimulant which has the ability to overcome drug-induced respiratory depression and hypnosis. In the past the CNS stimulants were widely used therapeutically, but today they have only limited clinical application. Note: Their use for respiratory depression caused by an overdose of CNS depressants is generally not safe or recommended. Analeptics and respiratory stimulants: doxapram and strychnine. These agents have limited use, but are occasionally indicated to stimulate respiration when a patient has pulmonary disease or to hasten recovery from a general anesthetic. Xanthines include caffeine, theophylline and theobromine. These stimulants improve mental alertness, reduce the urge to sleep and elevate the mood. Caffeine is the only approved OTC stimulant. Theophylline and theobromine are weaker CNS stimulants than caffeine. Theophylline is the only xanthine important in the treatment of asthma. It stimulates the respiratory centers of the medulla and is able to cause bronchial dilation in patients with asthma. Note: It has a low therapeutic index and its metabolism is affected by several other drugs. Sympathomimetic amines include the amphetamines and other related agents (i.e., methylphenidate, phendimetrazine, etc.). They are potent CNS stimulants. They are used to treat narcolepsy, obesity and attention deficit disorder.

Coughing

Caffeinism is a tenn used for people who are dependent upon caffeine (i.e., suffer side effects from having too much caffeine, take larger amounts and need to keep drinking caffeine to function properly). It is thought to occur if you have an intake of above 600 to 750 mg of caffeine per day (more than 10 cups of coffee). Drinking more than 1000 mg per day is well into the toxic range. Important: Caffeine stimulates the CNS unequally, with the cortex being the most and the spinal cord being the least excited. Caffeine Content of Commonly Ingested Products Product Beverages (/ cup or glass) Caffeine Content (mg) Brewed coffee Instant coffee Decaffeinated coffee Tea Cola drinks OTC Drugs Anacin Excedrin NoDoz Vivarin 32 65 100 200 100-150 50-100 2-35 40-110 35-60

TM

PDF Editor

PHARMACOLOGY

isc.J Misc

All of the following drugs on the right are used to treat what condition? Dexmethylphenidate (Focalin) Extended-release methylphenidate (Concerto) Mixed amphetamine salts (Adderal) Atemoxetine (Strattera) Controlled-delivery methylphenidate (Metadate CR) Lisdexamfetamine (Vyvanse)

' Insomnia in adults


1

Tourette's syndrome in children Attention deficit hyperactivity disorder (ADHD)

Insomnia in children
184 Copyright 2011-2012 - Dental Decks

PHARMACOLOGY

Misc.)

Which of the following drugs is a member of the opioid family and reduces GI motility ?

Loperamide (Imodium) Lorazepam (Ativan) Diphenoxylate and atropine (Lomotil) Propranolol (Inderal)

185 Copyright 2011-2012 - Dental Decks

TM

PDF Editor

Attention deficit hyperactivity disorder (ADHD) Drugs used to treat ADHD: Methylphenidate (Ritalin): a mild central nervous system stimulant. In children with ADHD, this drug results in an increase in attention span, reduction in hyperativity, and an improvement in behavior. Dexmethylphenidate (Focalin): a form of methylphenidate called dexmethylphenidate. Adderal (dextroamphetamine): the brand name for mixed amphetamine salts which act the same as methylphenidate in treating ADHD. Atemoxetine (Strattera): the brand name for atemoxetine, the first non-stimulant approved for treating ADHD. It is approved for use in children and adults. Metadate CR: the brand name for a controlled-delivery methylphenidate, another long acting form of the drug. Lisdexamfetamine (Vyvanse): Lisdexamfetamine is a prodrug; after administration it is converted to dextroamphetamine (Adderall) in the intestines and/or liver. The dextroamphetamine, an amphetamine and stimulant for the brain, is responsible for the effect of lisdexamfetamine in ADHD. Extended-release methylphenidate (Concerto): a long-acting form of methylphenidate.

Loperamide (Imodium)
Antidiarrheals:

Opiate and opioid derivatives - Loperamide (Immodium): 1. Is an anti-diarrheal which acts on intestinal muscles to inhibit peristalsis. 2. Is a member of the opioid family. It does not penetrate the central nervous system like the opioids such as codeine; thus it can be sold over the counter. 3. Has no evidence of drug abuse or dependence (unlike other opioids such as codeine, morphine and meperidine). - Diphenoxylate is an anti-diarrheal and inhibits excessive GI tract motility and GI propulsion. Commercial preparations contain a sub-therapeutic amount of atropine to discourage abuse. Diphenoxylate and atropine (Lomotil), unlike loperamide (Imodium), requires a prescription. Antisecretory:bismuth subsalicylate (Pepto-Bismol) Adsorbents: attapulgite (Kaopectate) Remember: Laxatives act in the reverse manner of the anti-diarrheals and increase the motility of the GI tract. They are used to treat constipation. Examples include: magnesium hydroxide (Milk of Magnesia), castor oil. Metamucil and methylcellulose. Antiemetics: act on the "vomiting center" in the medulla. This center has four different sources of stimuli. The chemoreceptor trigger zone (CTZ) is located outside the blood-brain barrier near the vomiting center in the medulla. It communicates with the vomiting center after input is received from drugs and hormones. The classes of antiemetics are: Antidopaminergic: Phenothiazines: prochlorperazine (Compazine) and promethazine (Phenergan) Benzamides: metoclopramide (Region) and trimcthobenzamide HCL (Tigan) Anticholinergics: Antihistamines: meclizine (Antivert). dimenhydrinate (Dramamine), and scopolamine (TransdermScop) Serotonin 5-HT3 receptor antagonists: ondansetron HCL (Zqfran), dolasetron (Anzemet), granisetron HCL (Kytril) and palonosetron (Aloxi)

TM

PDF Editor

(PHARMACOLOGY

Misc!)

All of the following are prominent toxic effects of mercury EXCEPT one. Which one is the EXCEPTION?

Irritability Xerostomia Loosened teeth Gum disorders Slurred speech Tremors

186 Copyright 2011-2012 - Dental Decks

(PHARMACOLOGY

Misc.J

Of the amino acid neurotransmitters listed below which one is an excitatory neurotransmitter?

Glycine Glutamate
1 GABA (y -aminobutyric acid)

None of the above

187 Copyright 2011-2012 - Dental Decks

TM

PDF Editor

Xerostomia *** Excessive saliva is a prominent toxic effect of mercury not xerostomia These symptoms are all of the chronic form, which results from the inhalation of the vapors of dust of mercurial compounds or from repeated ingestion of very small amounts. The presence of mercury in the body is determined by a urine test. Treatment may include gastric lavage with milk and egg white or sodium bicarbonate, chelation with British antilewisite (BAL), and fluid therapy. Note: British Anti-Lewisite (BAL) or Dimercaprol and penicillamine are two drugs currently marketed for promoting the excretion of mercury, lead, and several other agents. A few additional agents are available for the treatment of poisoning by metals other than mercury (e.g., edetate calcium disodium for lead and deferoxamine for iron). Mercury that is absorbed into the circulatory system may be deposited in any tissue. Higher-than-average accumulations occur in the brain, liver, and kidney. Mercury does not collect irreversibly in human tissues. There is an average half-life of 55 days for transport through the body to the point of excretion. Thus, mercury that came into the body years ago is no longer present in the body. 1. Children and adults who are to be treated for lead poisoning should only be given the Edetate Calcium Disodium (Calcium Disodium Versenate) form of "EDTA." 2. Penicillamine is also a highly effective chelator of copper and is of primary importance in the management of Wilson's disease (hepatolenticular degeneration). 3. Deferoxamine is a drug that chelates to absorbed iron very well and is eliminated in urine. 4. For carbon monoxide poisoning the treatment is 100% oxygen therapy (involves breathing oxygen through a tight-fitting mask). 5. Cyanide poisoning can be treated with rapid oxygen administration and the antidotes sodium nitrite and sodium thiosulfate.

Glutamate
Chemicals that transmit the signal from one neuron to the next are called neurotransmitters. They are synthesized in the cell body or nerve terminal of the presynaptic neuron. Neurotransmitters are released from the synapse and cross the synaptic cleft. The dendrite on the nerve cell body receives the signal. Various receptors on the postsynaptic membrane of the dendrite accept only certain neurotransmitters. In the brain. 30 different neurotransmitters have been classified as amino acids, amines, and neuropeptides.
Amino acid neurotransmitters:

- Glutamate, GABA, and glycine. Glutamate is an excitatory neurotransmitter. GABA and glycine are inhibitory neurotransmitters. GABA is the major inhibitory neurotransmitter within the CNS. Amines: - Include the catecholamines dopamine, norepinephrine, and epinephrine a s well as serotonin, histamine, and acetylcholine. Neuropeptidesmost are also hormones; these include vasopressin, oxytocin, insulin, somatostatin, gastrin, substance P, endorphin, and enkephalin. Remember: Acetylcholine: effects in CNS generated by interaction with a mixture of nicotinic and muscarinic receptors Dopamine: a catecholamine which acts through at least two subtypes Dl (activates adenyl cyclase), D2 (inhibits adenyl cyclase) Serotonin: is 5-hydroxytryptamine which works through at least 14 subreceptor "tryptominergic" type neurons f ~ ~ 1. Biosynthetic pathway of ACH: Step (1) Choline (taken up into nerve via action of permease) .Notes Step (2) Choline acetylcholinesterase catalyzes the synthesis of Ach from acetyl CoA and choline 2.Biosynthesis of NE and E: Step (1) Tyrosine to DOPA (enzyme is tyrosine hydroxylase Step (2) DOPA to Dopamine (enzyme is aromatic L-amino acid decarboxylase) Step (3) Dopamine to NE (enzyme is dopamine beta hydroxylase) Step (4) (mostly in the adrenal medulla): NE to E (enzyme is phenylethanolamine N-methyltransferase)

TM

PDF Editor

PHARMACOLOGY

isc.J Misc

Oral contraceptives block ovulation by inhibiting which TWO anterior pituitary hormones below?

Follicle stimulating hormone (FSH) Growth hormone (GH) Thyroid stimulating hormone (TSH) Luteinizing hormone (LH) Adrenocorticotrophic hormone (ACTH)

188 Copyright 0 2011-2012 - Dental Decks

PHARMACOLOGY

Misc!)

Drugs travel through the blood stream by binding to albumin plasma protein. However, only a small number of drugs travel through the blood stream by this mechanism.

The first statement is true; the second statement is false The first statement is false; the second statement is true Both statements are true Both statements are false

189 Copyright 2011-2012 - Dental Decks

TM

PDF Editor

Follicle stimulating hormone (FSH) Luteinizing hormone (LH) In addition to the above effects, oral contraceptives produce alterations in the genital tract, including changes in cervical mucus, rendering it unfavorable for sperm penetration even if ovulation occurs. Changes in the endometrium may also occur rendering it unfavorable for nidation (implantation of the fertilized ovum). Estrogens and progesterone-like compounds (progestins) are used for oral contraception. Estrogens: are a group of chemically similar steroid hormones. In humans, estrogens are made primarily in the female ovaries and in small amounts in the male testes and the adrenal glands, brain, and fat of both sexes. Estradiol is the most abundant and potent natural estrogen in humans. Other estrogens inlcude ethinyl estradiol and mestranol. Progestins: are a group of chemically similar steroid hormones as well. In humans, progestins are made primarily in the female ovaries and male testes. Progesterone is the most abundant and potent progestin in humans. Other progestins include levonorgestrel, norethindrone, medroxyprogesterone, norgestimate and norgestrel.
Types of oral contraceptives:

Combination: oral contraceptive agents usually contain both an estrogen agent and a progestin agent. Combination drugs include: ethinyl estradiol and norethindrone (OvconSO. Brevicon 21, and Modicon 28) ethinyl estradiol and levonorgestrel (Portia 0.15/30. Alesse 28 andAviane 21) thinyl estradiol and norgestrel (Cryselle, Ovral. and Orgestrel) Progestin-only: norethindrone (Micronor) Emergency contraception: levonorgestrol (Plan B) Warnings/Precautions with Oral Contraceptives: The risk of cardiovascular side effects increases in women who smoke cigarettes, especially those who are >35 years of age. May increase the risk of thromboembolism. Women with hypertension should be encouraged to use a nonhormonal form of contraception. Note: Antibiotics have the potential to diminish the effectiveness of oral contraceptives. Advise patients to use additional method of birth control when taking antibiotics and oral contraceptives concurrently.

The first s t a t e m e n t is true; the second s t a t e m e n t is false *** Most drugs travel through the blood stream by binding to albumin protein, which is abundant in plasma. In this way, drugs can be carried to all the tissues and organs. A drug which is bound to plasma albumin always has some fraction which is not bound. The unbound portion is free to leave the blood compartment to be taken up by tissues where the drug will elicit its phannacological effect. The remaining bound fraction of drug then continuously releases more free drug to be taken up by tissues. Eventually all drug in the blood compartment will be taken up by this process. Important: Interactions between two or more drugs can occur if they compete for binding on the plasma albumin. If drug A is bound to albumin prior to the patient taking drug B, and drug B has a greater binding affinity to albumin than drug A, then when drug B is taken, it will displace drug A from albumin to result in large amounts of unbound drug A which could lead to adverse reactions due to the sudden large amounts gaining access to the tissues. One of the most fundamental aspects of drug action is the relationship between dose administered and the effect obtained (drugs are dose dependent). Dose and response are related and can be represented by a dose-response curve. There are two basic types of dose-response curves: The graded dose-response (DR) curve plots the degree of a given response against the concentration of the drug. These curves are useful for determining characteristics of agonists and antagonists. The quantal dose-effect curve: In this case, a given quantal effect is chosen (e.g., a certain degree of cough suppression), and the concentration of the drug is plotted against the percentage of a specific population in which the drug produces the effect. The median effective dose (ED50 or the dose at which 50% of the individuals exhibit the specified quantal effect) and the median lethal dose (LD50 or the dose at which death is produced in 50% of the experimental animals in preclinical studies) can be estimated from quantal dose-effect curves. With this type of curve, the relative effectiveness of various drugs for producing a desired or undesired effect, as well as the relative safety between various drugs, can be determined.

TM

PDF Editor

a
1

PHARMACOLOGY

isc.l Misc

What are four criteria to consider when selecting an analgesic agent for a patient?

Type of pain Location of pain

1 Age of patient

Sex of patient Patient's weight Concurrent medication Pregnancy


190 Copyright 2011-2012 - Dental Decks

PHARMACOLOGY

Misc!)

Which of the following competitively blocks vitamin K-binding sites and inhibits the synthesis of vitamin K-dependent coagulation factors VII, IX, X, and II (prothrombin) and anticoagulant proteins C and S?

Heparin Warfarin (Coumadin) Aspirin


1 Anagrelide

191 Copyright 2011-2012 - Dental Decks

TM

PDF Editor

Type of pain Age of patient Concurrent medications Pregnancy Criteria considered when selecting an analgesic agent: 1. Type of pain: Severe Mild Moderate 2. Age of the patient: Infant " When calculating dosage, the height, weight, body surface area and renal hepatic Child / function must be taken into account Adult Elderly: drug response is affected by age-related changes in physiology and pharmokinetics 3. Concurrent medications: Consider unwanted interactions, especially with the elderly. 4. Pregnancy: Because virtually any drug a pregnant woman takes can cross the placenta and enter the fetal circulation, drug use in pregnant patients is a source of special concern (check with patient's OB/GYN). FDA Rating System for the Teratogenic Effects of Drugs: The FDA, the government agency that oversees the safety of drugs, provides the most widely used system to grade the teratogenic effects of medications. The FDA assigns a safety category for medications by using a 5-letter system: A, B, C, D, and X, with A being considered the most safe. X category medications: Studies in animals or humans have demonstrated fetal abnormalities or there is positive evidence of fetal risk based on adverse reaction reports from investigational or marketing experience, or both, and the risk of the use of the drug in a pregnant woman clearly outweighs any possible benefit (for example, safer drugs or other forms of therapy are available).

Warfarin (Coumadin) At therapeutic levels, warfarin decreases liver synthesis of vitamin K-dependent clotting factors by 30% to 50%. These clotting factors have different half-lives. Factor VII has the shortest half-life (6-9 hr) vs. factor II and X (up to 72 hr). Oral anticoagulants do not reverse ischemic damage or lyse an established thrombus but rather prevent extension of the existing thrombus and the formation of new thrombi by blocking synthesis of clotting factors. Oral anticogulants are used after a myocardial infarction to prevent coronary occlusion, in the treatment of pulmonary embolism, and in the treatment of venous thrombosis. Note: Enhanced anticoagulant effects are seen when these drugs are combined with aspirin. For this reason, use acetaminophen (Tylenol) for pain control. Patients on anticoagulant therapy may have excess bleeding after dental treatment. Always check medical history. If patient is on anticoagulants, his/her physician should have documentation of INR values to assess anticoagulant effects. INR stands for International Normalized Ratio and essentially is the ratio of the prothrombin time measured in the patient divided by a standard prothrombin time value, and multiplied by a constant. An INR value of 1 means normal prothrombin times of approximately 12 seconds; normal blood clotting would be present. INR values greater than 1 indicate that there is an anticoagulant effect. The higher the INR value, the greater the anticoagulant effect. Many patients taking anticoagulants have INR values of 2, 3 and even up through 6.

TM

PDF Editor

^PHARMACOLOGY

MiscJ

These drugs act to induce gene transcription, inhibit cellular growth and alter the state of cell differentiation.

Immune globulins Immunosuppressants Keratinocyte growth factor Interferons

192 Copyright C 2011 -2012 - Dental Decks

^PHARMACOLOGY

Misc.)

Granisetron (Kytril) and ondansetron (Zofran) are selective 5-HT3 receptor antagonists used to treat what condition?

Breast cancer Osteoporosis Emesis caused by cancer chemotherapy Mucositis caused by radiation therapy

193 Copyright C 2011-2012 - Dental Decks

TM

PDF Editor

Interferons Interferons are used for a variety of conditions including: hairy cell leukemia (interferon alpha-2a) chronic hepatitis B (interferon alpha-2b), recurring genital warts (interferon alpha-n3) and treatment of multiple sclerosis (interferon beta-la). Immune globulins provide passive immunity by increasing antibody titers. Immunosuppressants are drugs such as cyclosporin which prevent organ transplant rejection. Growth factors such as keratinocyte growth factor promote cell proliferation and angiogenesis.

Emesis caused by cancer chemotherapy The 5-HT3 receptor is a serotonin receptor which when activated during chemotherapy for cancer, causes emesis (nausea and vomiting). Both granisetron and ondansetron are indicated for prophylaxis of chemotherapy-related emesis, prophylaxis of nausea and vomiting associated with radiation therapy, and prophylaxis and treatment of postoperative nausea and vomiting (PONV). Other serotonin 5-HT3 receptor antagonists include: dolasetron (Anzemet) and palonosetron (Aloxi) Note: 5-HT3 stands for 5-hydroxytryptamine type 3 receptor.

TM

PDF Editor

PHARMACOLOGY

isc.J Misc

The following drugs belong to what pharmaceutical class of agents: Pimecrolimus Sirolimus Tacrolimus

Immune modulators Monoclonal antibodies Immunosuppressants Interferons Colony stimulating factors

194 Copyright 20! I-2012 - Dental Decks

PHARMACOLOGY

isc.J Misc

The following drugs belong to what pharmaceutical class of agents: Adalimumab Alefacept Infliximab Trastuzumab

Immune modulators
1

Monoclonal antibodies

1 Immunosuppressants 1 Interferons

Colony stimulating factors


195 Copyright 2011-2012 - Dental Decks

TM

PDF Editor

Immunosuppressants Pimecrolimus (Elidel) is an immunosuppressant agent used for treatment of mild to moderate atopic dermatitis. Sirolimus (Rapamune) is an immunosuppressant agent used for prophylaxis of organ rejection in patients receiving renal transplants. Tacrolimus (Protopic) is an immunosuppressant agent used to treat moderate to severe atopic dermatitis in patients not responsive to conventional therapy.

Monoclonal antibodies Adalimumab (Humira) is a recombinant monoclonal antibody that binds to human tumor necrosis factor alpha (TNF-alpha) receptor sites. It is used to treat active rheumatoid arthritis. Alefacept (Amevive) is a monoclonal antibody used to treat moderate to severe plaque psoriasis. Infliximab (Remicade) is a monoclonal antibody used to treat ankylosing spondylitis, Crohn's disease, and rheumatoid arthritis. Like adalimumab, infliximab works by binding to TNF-alpha receptor sites. Trastuzumab (Herceptin) is a monoclonal antibody which binds to the extracellular domain of the human epidermal growth factor receptor 2 protein (HER-2). It is used for the treatment of patients with metastatic breast cancer whose tumors overexpress the HER-2 protein and who have not received chemotherapy for their metastatic disease.

TM

PDF Editor

^PHARMACOLOGY

MiscJ

Your patient's medication history includes modafinil (Provigil). Your patient is probably using this drug in order to:

Improve wakefulness during daytime sleepiness Improve salivary flow in dry mouth disorders Manage psychotic disorder Treat mental depression

196 Copyright C 2011-2012 - Dental Decks

^PHARMACOLOGY

Misc!)

Which herbal supplement below is known to be somewhat effective in treating mild forms of mental depression?

Ginkgo biloba St Johns Wort Garlic Echinacea Ginseng

197 Copyright 2011-2012 - Dental Decks

TM

PDF Editor

Improve wakefulness during daytime sleepiness Modafinil (Provigil) represents a class of central nervous system stimulants used to improve wakefulness in patients with excessive daytime sleepiness associated with narcolepsy and shift work sleep disorder. It also has an unlabeled use to treat attention deficit/hyperactivity disorder (ADHD). The exact mechanism of modafinil is unclear; however part of its action may be do to decreased GABA-mediated neurotransmission.

St Johns Wort
Some studies have shown the St Johns Wort can inhibit the re-uptake of serotonin at neuronal synapses resulting in the elevation of serotonin within the CNS. This effect is similar to those antidepressants within the fluoxetine (Prozac) family. Of all the herbal supplements available to the consumer, St Johns Wort is associated with use as a mild antidepressant. Ginkgo biloba is a herbal supplement used as a peripheral artery vasodilator. Garlic is a herbal supplement used to lower cholesterol and to inhibit platelet aggregation resulting in a decrease in blood clotting. Echinacea is a herbal supplement and immune stimulator used to minimize the severity of the common cold and seasonal flu. Ginseng is a herbal supplement used to stimulate the immune system. Chamomile has a long history of use in Europe for digestive ailments. The active constiuents of chamomile have anti-inflammatory properties, and ease spasm and discomfort in the digestive tract. Note: Chamomile contains coumarin, a naturally-occurring compound with anticoagulant or bloodthinning effects. It should not be combined with warfarin or other anticogulants. Saw palmetto has long been used in Europe to treat an enlarged prostate or benign prostatic hyperplasia (BPH).

TM

PDF Editor

(PHARMACOLOGY

MiscJ

St Johns Wort is a herbal supplement which can interact with some conventional medications to:

Increase the effectiveness of medications Decrease the effectiveness of medications Increase the toxicity of medications

198 Copyright 2011-2012 - Dental Decks

(PHARMACOLOGY

Misc.)

All of the following drugs are serotonin 5-HT1D receptor agonists used to treat migraines EXCEPT one. Which one is the EXCEPTION?

Sumatriptan (Imitrex) Frovatriptan (Frova) Naratriptan (Amerge) Donepezil (Aricept) Rizatriptan (Maxalt) Almotriptan (Axert) Zolmitriptan (Zomig)
199 Copyright 2011-2012 - Dental Decks

TM

PDF Editor

Decrease the effectiveness of medications St Johns Wort may induce the cytochrome P450 enzyme system to result in a more rapid metabolism of many drugs in this pathway. More rapid metabolism of a drug equates to less active drug available and less (decreased) effectiveness. Specifically, data indicates probable interactions between St Johns Wort and its effect in decreasing the actions of the following drugs: The HIV-1 protease inhibitor indinavir The antirejection medication cyclosporin The sedative midazolam The antihypertensive nifedipine Drug-drug interactions can involve prescription or nonprescription (over-the-counter) drugs. Drugs interact by acting at the same receptor or signal transduction pathway, or more commonly, a drug may affect the pharmacokinetics of another drug. The most common form of drug-drug interaction is one drug affecting the metabolism of another drug, involving either induction or inhibition of metabolizing enzymes. Induction of metabolism: is a reaction to certain drugs in which the number of liver enzymes increases, resulting in a reduction in the effect of the other drug. Inhibition of metabolism: is a process by which one drug either competes for metabolism of another or directly inhibits drug-metabolizing enzymes. r _ ^ 1. Herbal medicines such as garlic, ginseng, and ginkgo (ginkgo biloba) are thought to interact Notes with anticoagulant or antiplatelet therapy. Remember: Chamomile contains coumarin and should not be used in patients taking warfarin for fear of an additive affect. 2. St. John's wort is the herbal product most often reported to be involved in drug-herb interactions. 3. Echinacea has potential interactions with immunosuppressants. 4. Many herbal weight loss and "herbal speed" products rely on the pharmacodynamic interaction between ephedra and caffeine. Two primary alkaloids contained in ephedra ephedrine and pseudoephedrine have additive cardiovascular effects when taken with caffeine. At higher doses, the ephedra-caffcine interaction has been cited as a cause of death. Important:The rate of drug metabolism can vary greatly, depending on the cytochrome p450 isozyme profile of a patient. Over the next few years, genetic screening of the cytochrome p450 system will be able to identify which patients are likely to develop toxicity or drug-drug interactions, and with which drugs. Ultimately, metabolic screening based on studies using highly-specific probes could help decide whether or not a drug is prescribed.

Donepezil (Aricept) *** Donepezil (Aricept) is a cholinesterase inhibitor used in the treatment of Alzheimer's disease. Migraine medications: The most commonly used migraine medications are the serotonin j (5-HTj) receptor agonists, commonly known as the "triptans." All 5-HTj receptor agonists (triptans) have a similar chemical structure and a comparable mechanism of action. 5-HT1B and 5-HT]D receptors located on the extracerebral, intracranial blood vessels that become dilated during a migraine attack and on nerve terminals in the trigeminal system. Therapeutic activity is caused by activation of these receptors, which results in cranial vessel constriction, inhibition of neuropeptide release, and reduced transmission in trigeminal pain pathways. Ergotamine derivatives demonstrate partial agonist and/or antagonist activity against dopamine, tryptaminergic, and alpha-adrenergic receptors, depending on their site. Ergotamine derivatives have three primary actions. They (1) depress central vasomotor center, (2) constrict peripheral and cranial blood vessels, and (3) reduce extracranial blood flow and decrease hyperfusion of the basilar artery area. Examples include: erotamine tartrate (Cafeigot) and dihydroergotamine (DHE-45) Combination drugs: Midrin is a combination capsule that contains 65 mg of isometheptene mucate (an ergotamine derivative), 325 mg of acetaminophen, which exerts an analgesic effect, and 100 mg of dichloralphenazone, a mild sedative that acts centrally to allay anxiety. Note: Caffeine, which is added to many migraine combinations, helps to promote constrictive properties, and enhances absorption. Note: Methylsergide (Sansett) blocks 5-HT2 receptors (serotonin antagonist) and is used for prophylaxis against migraines.

TM

PDF Editor

(PHARMACOLOGY

Misc!)

AH of the following are drugs used to treat hypothyroidism EXCEPT one. Which one is the EXCEPTION? Levothyroxine sodium (Synthroid) Liothyronine (Cytomel) Liotrix (Thyrolar) Methimazole (Tapazole)

200 Copyright 2011-2012 - Dental Decks

MPHARMACOLOGY

Misc!)

Nicotine is rapidly absorbed across the pulmonary capillary membrane and is delivered to the brain in high concentration within seconds of inhalation. The nasal spray has the fastest delivery of the NRTs and more closely resembles the onset of the nicotine effects of smoking.

The first statement is true; the second statement is false The first statement is false; the second statement is true Both statements are true Both statements are false

201 Copyright 5 2011-2012 - Dental Decks

TM

PDF Editor

Methimazole (Tapazole) Thyroid medications: Thyroid supplements: used to treat hypothyroidism Levothyroxine sodium (synthetic Tj) Liothyronine (synthetic Tf) \Ao\x\x(T4:Ti= 4:1) Note: These drugs are usually taken orally in a single daily dose, preferably before breakfast. The treatment of choice for hypothyroidism is T4. It has a relatively slow onset of action, and its effects are cumulative over several weeks. T3 has a more rapid onset of action and dissipation of action. Adverse side effects include nervousness, nausea, diarrhea, tachycardia, tremor, weight loss and heat intolerance. Thyroid suppressants: used to treat hyperthyroidism Methimazole (Tapazole) Propylthiouracil (PTU) Note: These drugs inhibit thyroid peroxidase, which is an enzyme expressed mainly in the thyroid that liberates iodine for addition onto tyrosine residues on thyroglobulin for the production of the thyroid hormones thyroxine (T4 \ or triiodothyronine (Ta). Through reduced absorption of iodine, thyroid hormone synthesis is diminished. Adverse effects include rash, nausea, and agranulocytosis. Note: Thyrogen is a recombinant DNA source of human TSH useful in the management and treatment of thyroid cancer patients.

Both statements are true


Many pharmacologic approaches have been used to help people stop smoking. In current use are the nicotine replacement therapy (NRT) products, bupropion (Zyban. Wellbutrin) and varcnicline (Chantix). Nicotine replacement products are meant to help patients stop smoking. They are not meant to help get a patient through a long flight or an all-day meeting in a nonsmoking building.

Class

Generic Name

Trade Name Nicorette Nicoderm CQ Habitrol Nicotrol Pro Step Nicotrol NS Nicotrol inhaler Commit Zyban, Wellbutrin Chantix

Formulation Gum Transdermal Transdermal Transdermal Transdermal Nasal spray Oral inhaler Lozenge Tablet Tablet

Nicotine replacement therapy Nicotine polacrilex Nicotine

Antidepressant Nicotine receptor agonist

Bupropion Varenicline

1. The patch (transdermal) is the most commonly used type of NRT. Patches arc available in 15 and 21 mg strengths for treatment initiation. 2. The gum delivers nicotine faster than the patch and can be used for incidence of craving. The gum is available in two strengths: 2 mg and 4 mg. 3. The lozenge also comes in 2 mg and 4 mg doses and delivers about 25% more nicotine lhan gum. 4. The nasal spray has the fastest delivery of nicotine of the NRTs. 5. Inhaled nicotine (using the nicotine inhaler) is absorbed in the mouth not in the lungs. Each cartridge contains 10 mg nicotine and delivers 4 mg. It also contains I mg menthol. 6. Bupropion as a systemic medication seems to reduce the craving for cigarettes or the urge to smoke. Its mechanism of action is unclear. 7. Varenicline is the newest drug in the smoking cessation arsenal. Note: The FDA has issued an alert to providers to monitor patients taking this medication for depression, severe mood swings, abnormal dream states, and thoughts of suicide.

TM

PDF Editor

(PHARMACOLOGY

Narcotic Analgesics)

Opioid drugs (narcotics) are used as all of the following EXCEPT one. Which one is the EXCEPTION?

' Analgesics > Anti-inflammatories


1 Antitussives 1 Antidiarrheals

Preanesthetic medications

202 Copyright C 2011-2012 - Dental Decks

(PHARMACOLOGY

Narcotic Analgesics)

Which drug is the standard to which all opiates (narcotics) are compared?

Codeine Oxycodone Hydrocodone Meperidine Morphine Fentanyl

203 Copyright 2011-2012 - Dental Decks

TM

PDF Editor

Anti-inflammatories Opiates are very effective analgesics. They suppress the cough reflex (antitussive). They cause constipation and thus are effective antidiarrheal agents. When used as preanesthetic medications, opiates permit a reduction in the amount of general anesthetic required for surgical anesthesia. Some common side effects of opiates (narcotics) include: sedation/drowsiness, dizziness and nausea. Other less common adverse effects are vomiting, hypotension, irregular/labored breathing (dyspnea), lightheadedness, nightmares and insomnia. 1. Respiratory depression is dose related and is the cause of death in narcotic drug overdose. It can happen with any of the narcotics. 2. Abuse can and does occur with all the narcotics. They are all controlled drugs falling under DEA Schedule II or DEA Schedule III of the schedules of controlled substances. 3. A DEA number is required in order to prescribe narcotics.

Morphine
Opioid analgesics are thought to inhibit painful stimuli in the substantia gelatinosa of the spinal cord, brainstem, reticular activating system, thalamus, and limbic system. Opiate receptors in each of these areas interact with neurotransmitters of the autonomic nervous system, producing alterations in reaction to painful stimuli. The opioid action of the drug manifests as: Decreased peristaltic motility Analgesia Nausea and vomiting Drowsiness Respiratory depression Euphoria Depression of the cough reflex Mental clouding Orthostatic hypotension Miosis (pupillary constriction) Morphine is a naturally occurring opiate that is metabolized chiefly through glucuronidation by uridine diphosphate glucuronosyl transferase (UGT) enzymes in the liver. These enzymes produce an active analgesic metabolite (morphine-6-glucuronide). Signs and symptoms of acute opioid intoxicarion:the intoxicated person is stuporous or asleep and has constricted pupils (pin-point pupils) and depressed respiration. As the severity of intoxication increases, coma ensues. Note: Death from acute intoxication by an opioid analgesic is the result of profound, direct respiratory depression. Remember: 1. Codeine is weaker (less potent) than morphine and less addictive. It is converted to morphine by cytochrome p450 2D6. Note: It differs from morphine in that a methoxy (OXII 3 ) substitution replaces the hydroxyl (OH) group on the aromatic ring of the molecule. This relatively minor structural change provides codeine with significant oral effectiveness. 2. Oxycodone has a similar potency as morphine. In combination with acetaminophen it is known as Percocet or Tylox. 3. Hydrocodone has a similar potency as morphine. In combination with acetaminophen it is known as \ icodin and Lorcet. 4. Meperidine (Demerol) is more potent than codeine but less potent than morphine but just as addictive. It has a shorter duration of action. It is the only narcotic agent that does not cause miosis (pupillary constriction). 5. Methadone (Dolophine) is an opioid agonist used in maintenance for treating opioid addiction. 6. Buprenorphine (Subutex) is an opioid partial agonist used to treat opioid dependence.

TM

PDF Editor

(PHARMACOLOGY

Narcotic Analgesics)

Which adverse effect is associated with opioid analgesics and not non-narcotic pain relievers?

Allergic response Nausea Vomiting Respiratory depression

204 Copyright 6 2011-2012 - Dental Decks

MPHARMACOLOGY

Narcotic Analgesics)

All of the following are opium alkaloids, EXCEPT one. Which one is the EXCEPTION?

Meperidine Morphine Codeine

205 Copyright 2011-2012 - Dental Decks

TM

PDF Editor

Respiratory depression The most significant and well-known adverse reaction to opioids is respiratory depression. Death secondary to opioid overdose is nearly always due to respiratory depression. When opioids are appropriately used, the risk of severe respiratory depression is generally small as tolerance rapidly develops to this effect.
Opioid Ana Igesics Drug Morphine Length of Effectiveness IV or IM: 2-3 hours By mouth: 3-4 hours Sustained release: 8-12 hours By mouth: 3^1 hours Taken with aspirin or acetaminophen IV or IM: about 3 hours By mouth: not very effective Other Information Starts to work quickly. Oral form can be very effective for cancer pain. Less potent than morphine Can cause seizures, tremors, and muscle spasms

Codeine Meperidine Methadone

By mouth: 4-6 hours, sometimes longer Also used for treating heroin withdrawal Generally taken with aspirin or acetaminophen to treat mild pain Oral form is strong. Can be used instead of morphine Usually combined with acetaminophen Usually combined with aspirin or acetaminophen Can block painkilling action of other opioids. About as strong a codeine. Can cause confusion and anxiety, especially in the elderly.

Propoxyphene By mouth: 3-4 hours Levorphanol Hydrocodone Oxycodone Pentazocine IV or IM: 4 hours By mouth: 4 hours By mouth: 4-6 hours By mouth: 3-4 hours By mouth: up to 4 hours

Meperidine
Chemical classification of opioid analgesics: Opium alkaloids: Morphine Codeine Synthetic derivatives: Morphine group: Hydromorphone (Dilaudid) Oxymorphone (Numorphan) Nalbuphine (Nubain) Codeine group: Hydrocodone (in Vicodin) Oxycodone (in Percodan, Percocet and Tylox) Synthetic narcotics: Meperidine group: Meperidine (Demerol) Alphaprodine (Nisentil) Alfentanil (Alfenta) Fentanyl (Sublimaze) Sufentanil (Sufenta) Diphenoxylate (in Lomotil) Loperamide (in Imodium) Methadone group: Methadone (Dolophine) Propoxyphene (Darvon)

TM

PDF Editor

(PHARMACOLOGY

Narcotic Analgesics)

ff

Which drug is the prototype opioid antagonist?

Morphine Naloxone Propranolol Ibuprofen

206 Copyright 2011-2012 - Dental Decks

(PHARMACOLOGY

Narcotic Analgesics)

The central nervous system contains three types of endogenous opioids. Which one of the following is NOT one of them?

Beta-endorphins Morphine Enkephalins Dynorphins

207 Copyright 2011-2012 - Dental Decks

TM

PDF Editor

Naloxone Naloxone (Narcan) is a narcotic antagonist and is used in medical emergencies to reverse narcotic overdose. Overdose of narcotics results in respiratory depression and death due to respiratory shut down. Naloxone will reverse the respiratory depressant effects of the narcotics thus counteracting the lethal effects of these agents. Naloxone is given intravenously, intramuscularly or subcutaneously. Note: Naloxone does not have agonist activity at any opioid receptors. Nalmefene (Revex) and naltrexone (ReVia) are the other two narcotic reversal agents used to reverse the respiratory depressive effects of the narcotic analgesics. Typical narcotic analgesic drugs which are reversed by naloxone (Narcan), nalmefene (Revex), and naltrexone (ReVia) include: Codeine Morphine Hydrocodone Oxycodone Meperidine (Demerol) Fentanyl

Note: Naltrexone (ReVia) is also used to treat alcohol dependence.

Morphine Opiate receptors in the CNS mediate analgesic activity. Opioid agonists occupy the same receptors as endogenous opioid peptides, and both alter the central release of neurotransmitters from afferent nerves sensitive to noxious stimuli. They decrease presynaptic release of neurotranmitters and increase postsynaptic potential. The opioid receptors all have a common general structure. They are characteristically G protein-linked receptors embedded in the plasma membrane of neurons. Once the receptors are bound, a portion of the G protein tGj) is activated, allowing it to diffuse within the plasma membrane. The G protein moves within the membrane until it reaches its target cither an enzyme or an ion channel. Opioid receptors in the central nervous system are thought to be activated by endogenous chemicals under physiologic conditions. The body contains three types of these chemicals that produce morphine-like effects (reduce pain): Beta-endorphins bind to opioid receptors in the brain and have potent analgesic activity. Enkephalins bind to opioid receptors in the brain and are more widely distributed in the brain than the beta-endorphins. Seem to play a role in pain perception, movement, mood, and behavior. Dynorphins are the most powerful of these chemicals and are found throughout the central and peripheral nervous systems. Some research supports the theory that they regulate pain at the spinal cord level, influence feeding behavior at the hypothalamic level and function with other endogenous opioids to regulate the cardiovascular system. Opioid receptors: 1. mu ( p ) - mediate morphine-like supraspinal analgesia, mitosis, respiratory depression, euphoria, physical dependence, and supression of opiate withdrawal. Note: The prototypical opioid agonist for this receptor is morphine, and its analgesic activity is considered to depend on its binding to this receptor. 2. delta (6) - mediate antagonist activity. Note: The enkephalins are considered to be the typical agonist for this receptor. 3. kappa (K) - mediate spinal analgesia, respiratory depression, and sedation. Note: The dynorphins are thought to be the typical agonist for this receptor. Sites of analgesic action of opioids: The opioid drugs produce analgesia by actions at several levels of the nervous system, in particular, inhibition of neurotransmitter release from the primary afferent terminals in the spinal cord and activation of descending inhibitory controls in the midbrain. *** Opioid analgesics (i.e.. morphine, codeine, meperidine, propoxyphene, etc.) mimic endogenous opioids at CNS opiate receptors, raising the pain threshold and increasing pain tolerance.

TM

PDF Editor

(PHARMACOLOGY

Narcotic Analgesics)

Which narcotic is not used in dentistry?

Morphine
1 Hydrocodone (component of Vicodin)

Oxycodone (component of'Percocet) Meperidine (Demerol) Codeine Fentanyl

208 Copyright 2011-2012 - Dental Decks

(PHARMACOLOGY

Narcotic Analgesics)

Which TWO of the following analgesics may interact with warfarin (Coumadin) to cause increased bleeding?

Acetaminophen Ibuprofen (Motrin; Advil) Hydrocodone with acetaminophen Aspirin

209 Copyright 2011-2012 - Dental Decks

TM

PDF Editor

Morphine

Therapeutic indication for opioids (narcotic analgesics) are the relief of moderate-tosevere pain, as preanesthetic medications, as analgesic adjuncts during anesthesia, as antitussives, and as antidiarrheals. Note: They are administered with caution to patients with head injury or those with a history of drug abuse and dependency.

/Notes1

1. Morphine is not used in dentistry because of its high addictive liability. 2. Hydrocodone in combination with acetaminophen is known as Vicodin, Lorcet, Lortab, Maxidone, and Zydone. 3. Hydrocodone in combination with ibuprofen is known as Vicoprofen. 4. Oxycodone in combination with acetaminophen is known as Roxicet, Percocet, and Tylox. 5. Oxycodone in combination with ibuprofen is known as Combunox. 6. Oxycodone alone is known as OxyContin. It is more potent than codeine. 7. Meperidine (Demerol) in combination with promethazine is known as Mepergan Fortis. 8. Codeine in combination with acetaminophen is known as Tylenol #3. 9. Fentanyl is available as a transmucosal preparation known as Actiq, a transdermal patch formulation known as Duragesic and as an intravenous preparation known as Sublimaze.

Among the opiates available for use in dentistry, hydrocodone products are commonly the drugs of choice.

Ibuprofen (Motrin; Advil) Aspirin Ibuprofen and other non-selective NSAIDs (inhibitors of both cyclo-oxygenase 1 and cyclo-oxygenase 2 enzymes) such as naproxen (Anaprox) and flurbiprofen (Ansaid) inhibit platelet aggregation. This action would enhance the anti-coagulant effect of warfarin (Coumadin) to increase the risk of bleeding. Aspirin inhibits platelet aggregation to potentiate the anticoagulant effects of warfarin (Coumadin) and increase the risk of bleeding. Acetaminophen is a non-narcotic analgesic that does not affect platelet aggregation nor does it affect the coagulation pathway. Thus, it will not affect the anticoagulant nature of warfarin (Coumadin). Acetaminophen can be given safely to patients taking warfarin (Coumadin). Hyrocodone with acetaminophen is a combination of a narcotic analgesic with acetaminophen. Common brand names for this combination are Vicodin, Lorcet and Lortab. Hydrocodone and the entire drug class of narcotic analgesics do not affect blood clotting and will not enhance the anticoagulant effects of warfarin (Coumadin). Narcotics with acetaminophen can be given safely to patients taking warfarin (Coumadin).

TM

PDF Editor

(PHARMACOLOGY

Narcotic Analgesics)

Your patient has a history of neuropathic pain. They are currently taking oxycodone (OxyContin), which is an opiate analgesic. Which TWO of the following are adverse effects of oxycodone?

1 Nausea 1

Peptic ulcers

1 Insomnia

Constipation

210 Copyright 2011-2012 - Dental Decks

(PHARMACOLOGY

Narcotic Analgesics)

Which drug is a synthetic opioid analgesic used as an intravenous sedative and is more potent than morphine?

Meperidine (Demerol) Pentazocine (Talwin) Propoxyphene (Darvon) Fentanyl

211 Copyright 2011-2012 - Dental Decks

TM

PDF Editor

Nausea Constipation The most common side effect of the narcotic (opiate) analgesics is nausea. The most serious side effect of the narcotic analgesics is respiratory depression. The cause of death from overdose of narcotics is respiratory depression and shut down of the respiratory system. Narcotic analgesics do not cause peptic ulcers. Nor do they cause insomnia, but rather would cause drowsiness and sedation since narcotics depress the conscious centers of the brain.

Fentanvl Fentanyl (Stiblimaze)is a potent narcotic analgesic used primarily as an intravenous supplement during conscious sedation procedures or procedures requiring general anesthesia. Fentanyl is 80100 times more potent than morphine. Fentanyl is also available as a lollipop-type lozenge (brand name Actiq) for transmucosal absorption and as a transdermal patch (brand name Duragesic) for delivery through a patch applied to the skin. Note: Fentanyl congeners include Alfcntanil, Sufentanil, and Remifentanil. Meperidine (Demerol) is used as an intravenous supplement during conscious sedation procedures. However, it is less potent than morphine and much less potent than fentanyl. Meperidine (Demerol) is also used as an oral medication for pain control after dental surgery. Note: A metabolite, normeperidine, is a CNS stimulant. Important: Contraindicated with MAO inhibitors (e.g., phenylzine and tranylcypromine) Pentazocine (Talwin) is chemically related to morphine and has weak analgesic properties. It is a mixed agonist-antagonist drug, having agonist activity at some receptors and antagonist activity at other receptors. It is not used intravenously to produce conscious sedation. Pentazocine has abuse liability. Note: Talwin NX tablets contain naloxone which is added to deter misuse. Propoxyphene (Darvon) in the form of propoxyphene napsylate with acetaminophen is known as Darvocet-N 100 and is useful for pain control after dental surgery. It is taken orally and not used intravenously. Darvocet-N 100 has low abuse liability. Note: It is less potent than codeine and a metabolite, norpropoxyphene, is a CNS stimulant. Tramadol (Ultram) is an opioid partial agonist. It is not a scheduled drug. Use great caution in combination with MAOIs (e.g., phenylzine and tranylcypromine).

TM

PDF Editor

(PHARMACOLOGY

Narcotic Analgesics)

AH of the following analgesics can produce drug dependence and addiction EXCEPT one. Which one is the EXCEPTION?

Codeine Morphine Hydrocodone Meperidine (Demerol) Oxycodone Voltaren

212 Copyright 2011-2012 - Dental Decks

(PHARMACOLOGY

Terms/Def)

All of the following sites are generally accepted for IM injections EXCEPT one. Which one is the EXCEPTION?

The buttocks The biceps muscle The deltoid muscle The anterior thigh

213 Copyright 2011-2012 - Dental Decks

TM

PDF Editor

Voltaren *** Voltaren is a NSAID which do not produce drug dependence and addiction. The other analgesics belong to the opiate class of drugs. They are also known as narcotic analgesics since the actions of this family is to cause drowsiness and sleep as a side effect. The opiates produce drug dependence leading to addiction. Psychic dependence, physical dependence, and tolerance can develop upon repeated administration. Psychic dependence is unlikely if an opiate is taken for a short period for pain relief Physical dependence is a condition in which continued administration of the drug is required to prevent unpleasant withdrawal symptoms. Tolerance occurs when increasingly large doses of opiate are required in order to produce the same degree of analgesia. Opiate drugs used in dentistry to provide pain relief after dental surgery include: Codeine Hydrocodone (Vicodin) Oxycodone (Percocet; Tylox) Meperidine (Demerol)

The biceps muscle *** For young children, the anterior thigh is usually the place to give IM injections. Absorption from an intramuscular injection is often faster and there is a higher bioavailability than with oral administration. Proper depth of needle in muscle: In big muscle (adult) go in one inch; in children go in one-quarter of an inch. Never go beyond two-thirds of the needle length.

TM

PDF Editor

(PHARMACOLOGY

Terms/Def)

Which term refers to a drug that binds to physiological receptors to result in specific cellular effects producing a predictable pharmacologic response?

Pharmacologic agonist Pharmacologic antagonist

214 Copyright 2011-2012 - Dental Decks

(PHARMACOLOGY

Terms/Def)

Which of the following routes of drug administration is generally used for local drug effects?

Topical administration Oral ingestion Subcutaneous injection Intravenous injection

215 Copyright 2011-2012 - Dental Decks

TM

PDF Editor

Pharmacologic agonist Drugs that bind to physiologic receptors and mimic the regulatory effects of endogenous signaling compounds will produce a pharmacologic effect as a result of binding to the receptor. A drug that elicits a full response through this process is referred to as a pharmacologic agonist. A partial agonist is a drug which acts on the physiologic receptor but elicits an effect which is only partly as effective as an agonist drug. An antagonist is a drug which binds to the physiologic receptor but does not trigger an effect. When antagonist is present, the agonist drug cannot reach the receptor site to produce an effect. 1. Competitive antagonism occurs when a response can be achieved by increasing the dose of agonist in the presence of antagonist. 2. Noncompetitive antagonism occurs when a response cannot be achieved with increasing doses of agonist in the presence of antagonist.

Topical administration Common routes of administration of drugs: 1. Parenteral administration (not by way of intestine or GI tract) Intravenous (IV) - directly into the bloodstream. Acts very rapidly. Intramuscular (IM) - injected into a muscle area, where it is promptly absorbed Subcutaneous - injection beneath the skin. Absorption may be less rapid. 2. Enteral administration (by way of the intestine or GI tract) Buccal or sublingual - a tablet is placed under the tongue or in the cheek Oral - (most common route), the drug is swallowed. It is the most convenient for safe drug administration. It is safe, painless and economical. Rectal - the drug in solution (enema) or suppository form is inserted into the rectum 3. Inhalation - the drug is given as an aerosol into the respiratory tract. 4. Topical administration - the drug is placed on the skin for a local effect. 5. Transdermal - the drug is placed within a "patch" and placed on the skin to be absorbed into the systemic circulation.

TM

PDF Editor

(PHARMACOLOGY

Terms/Def)

The most important enzyme systems for the biotransformation of drug molecules are found in:

1 Lungs 1 Liver 1 Brain

Kidneys Gastrointestinal tract

216 Copyright C 2011-2012 - Dental Decks

(PHARMACOLOGY

Terms/Def)

Implicit in the interaction of a drug with its physiological receptor is that the drug create effects, but rather, modulates physiological cell functions.

Does; extrinsic Does not; extrinsic Does; intrinsic Does not; intrinsic

217 Copyright 2011-2012 - Dental Decks

TM

PDF Editor

Hepatic metabolism of drugs occurs in Phase I reactions catalyzed by a microsomal mixed-function oxidase system (also known as the P-450 system) and in Phase II reactions known as conjugation reactions. Phase I reactions: Occur in the liver microsomal enzyme system (mixed-function oxidase system orP450 system). In this system, drug metabolism occurs in three basic patterns. First, the active parent drug can be converted to the inactive metabolite. Second, an active parent drug may be converted to a second active compound which is subsequently converted to an inactive compound; and thirdly, an inactive parent drug may be transformed to an active compound. The most common reaction in drug metabolism is an oxidation reaction in which oxygen in the form of hydroxyl group is attached to the drug molecule. There are at least eight distinct groups of microsomal drug metabolizing enzymes: These enzymes "families" are identified as a cytochrome (CYP prefix) followed by their numerical digestion (e.g., 1A2). Thus the enzyme CYP 1A2 is a distinct drug metabolizing enzyme that converts a variety of drugs to the oxidized product. Phase II reactions: Conjugation reactions involve coupling the drug with an acid present in cells (usually glucuronic acid). When coupled to glucuronic acid, the process is known as glucuronide conjugation with the resulting metabolite referred to as the "glucm-cnide." Conjugations occur in the liver, kidney and to a lesser extent in other tissues. Conjugation of drugs results in polar, water-soluble compounds that are rapidly excreted in urine. Thus, the parent drug is effectively rendered inactive and transported out of the body by this process.

Does not; intrinsic There are four major families of physiological receptors that drugs can bind to produce effects: Receptors as enzymes (i.e., cell surface protein kinases): These kinases exert their regulatory effects by phosphorylating proteins within the cell which alters the cellular biochemical activities. By binding to these kinases, drugs can also cause the alteration in biochemical activities resulting in a drug effect. Ion channels: Drugs can bind to ion channels in cell membranes to cause opening or closing. This alters the cell's membrane potential to result in a drug effect. G protein-coupled receptors: When drug binds to these receptors, second messengers are produced such as cyclic AMP to produce an effect within the cell. This results in a drug effect. Receptors in cell nucleus: Receptors for steroid hormones are soluble DNA-transcription factors within the nucleus that regulate the transcription of specific genes. Modification of the transcriptions of these genes results in a drug effect.

TM

PDF Editor

(PHARMACOLOGY

Terms/Def)

The safest and easiest route for drug administration is:

Oral IV Rectal Inhalation

218 Copyright 6 2011-2012 - Dental Decks

(PHARMACOLOGY

Terms/Def)

The majority of drugs penetrate biomembranes by through membrane phospholipids.

Active transport Facilitated diffusion Filtration Simple diffusion

219 Copyright 2011 -2012 - Dental Decks

TM

PDF Editor

Oral *** abbreviated P.O. However, it is also the most unpredictable and least effective route available. Drugs taken by mouth have to be absorbed (usually from the small intestine) before they can be transported to their site of action. Absorption may be slow, unpredictable and irregular due to the presence of variable amounts of food in different stages of digestion and to the varying degrees of acidity and alkalinity of the digestive juices. Moreover, blood from the intestinal tract passes first to the liver: some drugs are metabolized in the liver and others may be stored there to be released only slowly. These considerations make it clear that oral administration is usually unsuitable in emergencies or on other occasions when a rapid effect is needed. Note: After oral administration, drugs will generally be absorbed best from the duodenum. The duodenum has a large surface area due to the presence of villi and microvilli. Note: A major advantage of IV administration of a drug is it allows for titration of the drug. Other advantages of IV administration include: Rapid onset Drugs that cause irritation when administered subcutaneously can be given IV with no irritation In case of emergency, there is an open line through which emergency drugs can be injected One major disadvantage of IV injection is that since it has such a rapid onset of action, overdosage may have effects so immediate that it is impossible to reverse them.

Simple diffusion The mechanism of drug transfer across biological membranes is by: 1. Passive transfer is essential to various processes of metabolism Simple diffusion: lipid-soluble substances move across the lipoprotein membrane by this process. The majority of drugs penetrate biomembranes by this process through membrane phospholipids. The amount of drug dissolving in the membrane at any time is directly proportional to the concentration gradient and its degree of lipid solubility (Note: Only nonionized drugs are soluble in lipid). Filtration: water-soluble molecules small enough to pass through membrane channels may be carried through the pores by the bulk flow of water. Drugs of molecular weights of 60,000 or less can "filter" through capillary membranes. 2. Specialized transport Active transport: lipid-insoluble substances (for example glucose) are shuttled across plasma membranes by forming complexes with specific membrane constituents called carriers. These carrier molecules within a cell furnish energy for transportation of the drug to regions of higher concentration. Facilitated diffusion is the term given to carrier-based transfer when the driving force is simply the concentration difference of the drug across the membrane. Note: Osmosis is the movement of a pure solvent, as water, through a semipermeable membrane from a solution that has a lower solute concentration to one that has a higher concentration. The membrane is impermeable to the solute but is permeable to the solvent. Remember: The physiochemical properties of drugs that influence their passage across biologic membranes are: lipid solubility, degree of ionization, and molecular size and shape.

TM

PDF Editor

PHARMACOLOGY

Terms/Def
>

A teaspoon can hold what volume of liquid?

5 milliliters 10 milliliters 15 milliliters 20 milliliters

220 Copyright 0 2011-2012 - Dental Decks

PHARMACOLOGY

Terms/Def

AH of the following drugs are known to produce orthostatic hypotension as an adverse reaction EXCEPT one. Which one is the EXCEPTION?

Levodopa (Larodopa, Dopar) Prazosin (Minipress) Indomethacin (Indocin) Morphine

221 Copyright 2011 -2012 - Dental Decks

TM

PDF Editor

5 milliliters

Approximate Household Measures and Weights Teaspoons 1 3 48 1 16 1 Tablespoons Cups or Glasses Fluid Ounces 0.166 0.50 8 Milliliters 5 15 240 5 15 240 Grams

Indomethacin (Indocin) *** Indomethacin (Indocin) is an NSAID which may cause GI bleeding, ulcers and possible stomach perforation. Drugs that may produce orthostatic hypotension: Antihypertensives: for example prazosin (Minipress) Phenothiazines: for example chlorpromazine and thioridazine (Mellaril) Tricyclic antidepressants: for example doxepin (Sinequan), amitriptyline (Elavil) and imipramine (Tofranil) Narcotics: for example meperidine (Demerol) and morphine Antiparkinson drugs: for example levodopa (Larodopa. Dopar) and carbidopa + levodopa (Sinemet) Orthostatic hypotension (also called postural hypotension) is abnormally low blood pressure occurring when an individual assumes the standing posture. Important: Following vasovagal syncope, orthostatic hypotension is the most likely cause of transient unconsciousness in the dental office. Many factors have been identified that may be responsible for the development of orthostatic hypotension, including several which are important to the practice of dentistry. They include the administration and ingestion of drugs, prolonged recumbency and convalescence, an inadequate postural reflex, pregnancy, various defects in the legs, Addison's disease, physical exhaustion and starvation and chronic orthostatic hypotension (Shy-Drager syndrome). The incidence increases with age.

TM

PDF Editor

(PHARMACOLOGY

Terms/Def)

Pain that has no organic basis and is fixed upon some portion of the anatomy is referred to as:

Intractable pain ' Referred pain Psychogenic pain Phantom pain

222 Copyright 2011-2012 - Dental Decks

(PHARMACOLOGY

Terms/Def)

Which schedule of drugs would include a drug that is considered to have a strong potential for abuse or addiction, but which has legitimate medical use?

Schedule I Schedule II Schedule III Schedule IV Schedule V

223 Copyright 2011-2012 - Dental Decks

TM

PDF Editor

Phantom pain

*** For example, the sensation of pain felt in a limb, although that limb has been amputated. Other terms to know: Intractable pain: is pain that is resistant or refractory to ordinary analgesic agents Referred pain: is pain felt in an area other than the site of origin, such as pain near the shoulder associated with biliary disease Psychogenic pain: is pain produced or caused by psychic or mental factors rather than organic factors Remember: Pain threshold refers to the lowest level of pain a patient will detect.

Schedule II

Schedule I - a category of drugs not considered legitimate for medicinal use. Among the substances so classified by the DEA are mescaline, LSD, heroin, and marijuana. Special licensing procedures must be followed to use these or other Schedule I substances. Schedule II - a category of drugs considered to have a strong potential for abuse or addiction, but which have legitimate medical use. Among the substances so classified by the DEA are Morphine, Cocaine, Pentobarbital, Oxycodone, Methadone, and straight Codeine. Schedule III - a category of drugs considered to have less potential for abuse or addiction than Schedule I or II drugs. Among the substances so classified by the DEA are various analgesic combination compounds containing codeine (i.e., acetaminophen and codeine - Tylenol #3) and various analgesic combination compounds containing hydrocodone (i.e., hydrocodone and acetaminophen Vicodin; Lorcet). Schedule IV - a category of drugs that have less potential for abuse or addiction than those of Schedules I to III. Among the substances so classified by the DEA are diazepam (Valium), lorazepam (Ativan), triazolam (Halcion), alprazolam (Xanax), and chloral hydrate. Schedule V - a category of drugs that have a small potential for abuse or addiction. Among the substances so classified by the DEA are many commonly prescribed medications that contain a small amount of Codeine. Note: Schedule II and III must have a written prescription signed by the health professional (laws vary from state to state). The FDA determines which drugs are to be sold by prescription only. The prescription must have the address of the patient and dentist as well as the DEA number of the dentist.

TM

PDF Editor

(PHARMACOLOGY

Terms/Def)

A drug with a high LDCA and a low EDS0 has a

High therapeutic index and is, therefore, very dangerous High therapeutic index and is, therefore, relatively safe Low therapeutic index and is, therefore, very dangerous Low therapeutic index and is, therefore, relatively safe

224 Copyright 2011 -2012 - Dental Decks

(PHARMACOLOGY

Terms/Def)

Which route of administration of a drug listed below is most known for its significant hepatic "first pass" metabolism?

Intramuscular Inhalation Sublingual Oral

225 Copyright 2011-2012 - Denial Decks

TM

PDF Editor

High therapeutic index and is, therefore, relatively safe Explanation: The purpose of an acute toxicity test is to determine the nature and extent of the untoward reactions which might follow the administration of a single dose (or an overdose) of a drug. A quantitative aspect of acute toxicity testing is the determination of the drug's lethal dose using mice. This is usually expressed as the LD 5 u . Standing alone, it conveys less information than does the ratio of the lethal to the effective doses (LD5(/ED5Q), a quantity which is often known as the therapeutic index (a measure of drug safety). The greater a drug's therapeutic index, the less likely that fatalities will follow an accidental overdose. 1. EDQ is the effective dose at which 50% of people will respond. 2. LD 5 Q is the lethal dose at which, in theory, 50% of people will die. *** Lethal doses are always determined in mice, not people 3. The concept of therapeutic window is not the same as therapeutic index. The therapeutic window describes the range between the lowest therapeutic concentration and the beginning of toxicity. Some drugs have a very narrow therapeutic window, making patient monitoring especially important. 4. Toxicity results when the dose of the drug is excessive for the particular patient. 5. Side effect: an adverse effect that occurs within the therapeutic dose range of the drug. 6. Idiosyncratic reaction: a reaction to a medication that is unusual and unpredictable, specific to a particular person. Unlike allergy, it can occur on first exposure to the medication, unlike a side effect, it affects only very few individuals.

The oral administration of a drug is the one most acceptable to the patient. It is convenient because drugs can be given in the form of tablets or capsules which contain an exact dose. It is easy and the patient can take the drug without help from anyone else. One of the disadvantages of drugs taken by mouth is that they have to be absorbed (usually from the small intestine) before they can be transported to their site of action. Blood from the intestinal tract passes first to the liver: some drugs are metabolized in the liver ("first-pass effect") and others may be stored there to be released slowly. This consideration makes it clear that oral administration in usually unsuitable in emergencies or on other occasions when a rapid effect is needed. Intramuscular injection is an injection made into a large muscle. The advantages of IM injection are that it results in uniform absorption and that it can be used for solutions too irritant for subcutaneous injection. The speed of absorption of drugs given by IM injection depends on the vehicle in which they are dissolved: absorption is rapid from aqueous solutions and slow from oily solutions.

TM

PDF Editor

(PHARMACOLOGY

Terms/Def)

The term bioavailability of a drug refers to:

JJ

The movement of a drug into the body tissues over time The dissolution of a drug in the GI tract The measurement of the rate and amount of therapeutically active drug that reaches the systemic circulation The relationship between the physical and chemical properties of a drug and the systemic absorption of the drug The amount of drug destroyed by the liver prior to systemic absorption from the GI tract

226 Copyright 2011-2012 - Dental Decks

(PHARMACOLOGY

Terms/Def)

I Factors influencing hepatic drug metabolism include which of the following?

Microsomal enzyme inhibition Microsomal enzyme induction Plasma protein binding Genetic factors ' Liver disease 'All of the above

227 Copyright 2011-2012 - Dental Decks

TM

PDF Editor

The measurement of the rate and amount of therapeutically active drug that reaches the systemic circulation
The bioavailability of a drug is affected by: The dissolution of a drug in the GI tract The destruction of a drug by the liver Pharmacokinetics focuses on the processes concerned with absorption, distribution, biotransformation (metabolism), and excretion (elimination) of drugs. Absorption: there arc four primary factors that must be considered in evaluating drug absorption: 1. Drug characteristics: Formulation of the drug Concentration of the drug: the higher the concentration, the more quickly the drug is absorbed Lipophilic drug formulations are more readily absorbable: nonionized drugs are more lipid soluble and may readily diffuse across cell membranes Acidic drugs become nonionized in the acidity of the stomach and then diffuse across membranes: basic drugs tend to ionize and arc not well absorbed in the stomach 2. Routes of administration: the most common routes for giving medications include oral, topical, subcutaneous, intramuscular, inttavenously, and rectal. 3. Blood flow: circulation at the site of administration is important in the drug absorption process. Decreased circulation (as seen in congestive heart failure) will result in decreased drug absorption. 4. Cell membrane characteristics: drugs cross membranes via passive diffusion or active transport. Distribution: is the transport of a drug in body fluids from the bloodstream (at the site of absorption) to various tissues in the body Biotransformation (metabolism): is the chemical inactivation of a drug through conversion to a more water-soluble compound that can be excreted from the body. It occurs primarily in the liver. Involves two major steps: Phase I makes the drug more hydrophilic through oxidation, reduction, or hydrolysis. Phase II is called glucuronidation, it involves conjugation. Elimination: is the process by which drugs and their metabolites are removed from the body. The liver and the kidney are two major organs responsible for elimination. Most elimination occurs through excretion by the kidneys. The processes involved in renal elimination consist of glomerular filtration, tubular secretion, and partial reabsorption.

All of the above Factors Influencing Hepatic Drug Metabolism: Microsomal enyzme inhibition: many drugs and environmental agents can inhibit many of the CYP isoforms of the P-450 microsomal drug metabolizing system. Thus many drugs which ordinarily are metabolized by the particular CYP inhibited will not be effectively metabolized and will achieve higher than expected blood levels. Microsomal enzyme induction: agents which induce higher levels of the microsomal drug metabolizing enzymes may cause a more rapid metabolism of other drugs thus resulting in lower than expected blood levels of a drug. Plasma protein binding: drugs highly bound to plasma proteins will not enter the liver to be metabolized, thus resulting in a longer plasma half-life of the drug. Genetic factors: there is individual variance through genetic factors which contribute to differing rates of drug metabolism in the hepatic microsomal enzyme system. Liver disease: hepatic impairment and liver disease most often will result in impairment of the microsomal drug metabolizing system. This most often results in elevated levels of unmetabolized drug. Note: Most drugs are given at lower doses in hepatically impaired individuals.

TM

PDF Editor

(PHARMACOLOGY

Terms/Def)

The urinary elimination of drugs is controlled by all of the following EXCEPT one. Which one is the EXCEPTION?

Glomerular filtration Microsomal enzyme induction Renal tubular reabsorption Active transport through renal tubular cells

228 Copyright 0 2011-2012 - Dental Decks

(PHARMACOLOGY

Terms/Def)

The average time for the onset of drug effect after subcutaneous administration is:

Immediate 5 minutes 15 minutes 30 minutes

229 Copyright 2011-2012 - Dental Decks

TM

PDF Editor

Microsomal enzyme induction Glomerular filtration: all drugs are filtered through the glomerulus to enter the renal tubules. The amount of drug varies according to the degree of plasma protein binding, and bound drugs are not subjected to filtration. Tubular reabsorption: once they enter the renal tubules, drugs may be reabsorbed back into the blood stream through the renal tubular cells. Reabsorption favors the highly lipid soluble agents; the converse is that highly polar compounds are not effectively reabsorbed and are effectively excreted from the renal tubules. Active transport through renal tubular cells: the rate of renal elimination also depends on whether active transport into or out of the tubular fluid occurs. Other excretory pathways for drugs: The gastrointestinal tract excretes some drugs through the feces. This is not as prevalent as urinary excretion. Most drugs can be detected in saliva after administration, but the salivary glands are not considered a route of drug excretion since the drug is re-swallowed along with the saliva. Lungs excrete volatile compounds that were originally inhaled into the system. Nitrous oxide and the volatile general anesthetics are excreted by this route. Some drugs are excreted in the sweat, but this route accounts for only a small percentage of drug excretion.

15 minutes Routes of Drug Administration: Oral route: It takes approximately 30 minutes for onset of drug effect after drug is swallowed. Oral route allows for use of many different dosage forms including tablets, capsules and liquids. Intramuscular injection: Onset of action of drugs injected into muscle occurs rapidly (approximately 5 minutes) because of high blood flow through the muscles. Subcutaneous injection: Onset of action of drugs injected under the skin takes about 15 minutes. Inhalation: Gases such as nitrous oxide are absorbed rapidly through the lungs and gain access to the general circulation within 5 minutes. Topical: This route includes ointments and creams applied to the skin and mucous membranes. It is not intended for systemic drug administration. Patch delivery: Onset of action of drugs in skin patches is about 15 minutes and similar to subcutaneous injection. Skin patches release drug into the blood stream over a 12 to 24 hour period.

TM

PDF Editor

QPHARMACOLOGY

Terms/Def

Which term is a compulsive, uncontrollable dependence on a substance, habit or practice to such a degree that cessation causes severe emotional, mental or physiological reactions?

Habituation Tolerance Addiction None of the above

230 Copyright 2011-2012 - Dental Decks

PHARMACOLOGY

Terms/Def

Which route of drug administration will give the most rapid onset of pharmacological effect?

Oral administration Subcutaneous injection Intravenous injection Intramuscular injection

231 Copyright 2011-2012 - Dental Decks

TM

PDF Editor

Addiction

Habituation is an acquired tolerance from repeated exposure to a particular stimulus. Psychological and emotional dependence on a drug, tobacco or alcohol result from the repeated use of the substance but without the addictive, physiological need to increase dosage. Tolerance is the phenomenon of decreased responsiveness to a drug following chronic administration. Note: Physiological dependence is common to all forms of drug dependence and abuse. These drugs of abuse all have the ability to change one's mood and sensory perception.

Intravenous injection When a drug is given intravenously, it is placed directly into the systemic circulation. The drug is delivered rapidly to all tissues, including the drug receptor sites. For all other routes of drug administration (with the exception ofintra-arterial injection), the drug must be systemically absorbed prior to distribution to the drug receptor sites, and therefore the onset of pharmacological effects is slower. Also, when a drug is given by IV injection there is a complete (100%) bioavailability. The entire dose is placed into the systemic circulation. With other routes of administration, the drug may be lost prior to reaching the systemic circulation. For example, with first-pass effects, a portion of an orally administered drug is eliminated, usually through degradation by liver enzymes, before the drug reaches its receptor sites. Remember: The initial distribution of a drug into the tissues is determined chiefly by the rate of blood flow to the tissue, whereas drug affinity for the tissue will determine whether the drug will concentrate at that site. Note: Gastric emptying time and degree of plasma protein binding also have an effect on drug distribution but are less important than the rate of blood flow to the tissues.

TM

PDF Editor

(PHARMACOLOGY

Terms/Def)

When a drug is administered repeatedly, a higher concentration of the drug than is desired may be achieved. The effect of this excessive accumulation is known as:

Additive effect Synergistic response Cumulative action Idiosyncrasy

232 Copyright C5 2011-2012 - Dental Decks

(PHARMACOLOGY

Terms/Def)

The Controlled Substance Act of 1970 uses which criteria for inclusion of a drug into one of the five schedules?

Potential for abuse Medical usefulness Degree to which it produces physiological dependence Degree to which it produces physical dependence All of the above

233 Copyright 2011-2012 - Dental Decks

TM

PDF Editor

Cumulative action An additive effect occurs when additive drugs are administered. The response is no greater than that which would be expected had the drugs been given one at a time. There is no enhancement of potential of the individual drugs as a result of being used in combination. A synergistic response occurs when the combined action of two drugs is greater than the sum of their individual actions. Examples of synergism: Alcohol is synergistic with diazepam (Valium), narcotics, barbiturates and phenothiazines. It should be avoided if taking these drugs. Idiosyncrasy is a response to a drug that is unusual or abnormal or one that grossly deviates from the routine reaction.

All of the above


The Controlled Substance Act of 1970 uses the following criteria for inclusion of a drug into one of the five schedules: 1. Potential for abuse (most important) 2. Medical usefulness 3. Degree to which it produces physiological dependence 4. Degree to which it produces physical dependence Remember: Schedule I drugs (LSD, heroin, etc.) cannot be prescribed and arc made available only for specific approved research projects. Schedule II drugs (amphetamines, morphine, codeine, oxycodone etc.) can be prescribed but can not be refilled. A new prescription must be written for refills. Prescriptions for Schedule II drugs can not be called into the pharmacy over the telephone. Schedule III drugs (hydrocodone with acetaminophen, codeine with acetaminophen, etc.) may be called in to the pharmacy over the telephone. The prescriber can authorize refills without need of a new written prescription. *** The prescriber must have a Drug Enforcement Agency authorization number (DEA U) in order to prescribe scheduled drugs. Clinical testing of drugs: Before a drug can be approved for sale to the public there is a set of clinical tests that must be performed. There is the Pre-Clinical Research Stage. Here the drug is synthesized and purified. Animal tests arc performed, and institutional review boards assess the studies and make recommendations on how to proceed. If the recommendations are positive, then an application to the FDA occurs and clinical tests begin. Phase 1: clinical studies in this phase represent the first time that an investigational new drug is tested on humans cither healthy volunteers or sometimes patients. The purpose of these studies is study in a clinical setting the metabolism, structure-reactivity relationships, mechanism of action, and side effects of the drug in humans. If possible, phase 1 studies arc used to determine how effective the drug is. Phase 1 studies arc usually conducted on 20 to 80 subjects. The purpose of phase 2 clinical trials is to determine the efficacy of a drug to treat patients with a specific disease or condition, as well as learn about common short-term side effects or risks. These studies are conducted on a larger scale than phase 1 studies and typically involve several hundred patients. Phase 3 clinical trials provide more information about the effects and safety of the drug and they allow scientists to extrapolate the results of clinical studies to the general population. Phase 3 studies generally involve several hundred to several thousand people. *** After a successful phase III, the drug company submits to the Food and Drug Administration (FDA) a New Drug Application (NDA) to market the drug.

TM

PDF Editor

QPHARMACOLOGY
i

Terms/Def

Which of the following refers to the efficacy of a drug?

The relative concentrations of two or more drugs that produce the same drug effect The ability of a drug to produce a desired therapeutic effect regardless of dosage The dose of a drug that will kill a patient

234 Copyright 2011-2012 - Dental Decks

PHARMACOLOGY

Terms/Def <)

When two drugs having similar effects are administered in combination, and the resulting response is the sum of the individual actions of each drug when given alone, then this result is referred to as:

An additive effect A synergistic response An antagonistic effect Cumulative action

235 Copyright 2011-2012 - Denial Decks

TM

PDF Editor

The ability of a drug to produce a desired therapeutic effect regardless of dosage *** Efficacy refers to the number of receptors that must be activated to yield a maximal response. A drug with high efficacy needs to stimulate only a small percentage of receptors, whereas a drug with lesser efficacy (but still considered to be a full agonist) has to activate a larger proportion of receptors. Factors Governing Drug Action: Two factors that determine the effect of a drug on physiologic processes are affinity and intrinsic activity. Affinity is a measure of the tightness that a drug binds to the receptor. Intrinsic activity is a measure of the ability of a drug once bound to the receptor to generate an effect activating stimulus and producing a change in cellular activity. Potency is the relative concentrations of two or more drugs that produce the same drug effect. The effect usually chosen is 50% of the maximal effect and the dose causing this effect is called the EC 50 . Potency is determined mainly by the affinity of the receptor for the drug. Note: The smaller the EC 50 , the greater the potency of the drug. Potency is a comparative term (one drug is more potent than another drug). Example: Drug #1 in a dose of 10 mg produces the same magnitude of response as Drug #2 in a dose of 50 mg. The following is true: Drug #1 is five times as potent as Drug #2. Also: If Drug #1 has a greater efficacy than Drug #2, then Drug #1 is capable of producing a greater maximum effect than Drug #2. When comparing drugs with respect to intensity of response, the drug that produces the greatest maximum effect is the one with the highest efficacy.

An additive effect An additive effect occurs when additive drugs are administered. The response is no greater than the sum of the individual actions of each drug when given alone. There is no enhancement of potential of the individual drugs as a result of being used in combination. A synergistic response occurs when the combined action of two drugs having similar pharmacological effects is greater than the sum of the individual actions. Alcohol is synergistic with the Valium family of drugs (i.e., Valium, Xanax, Halcion, etc), narcotics and barbiturates. Alcohol should be avoided when taking these medications.

TM

PDF Editor

You might also like